0% found this document useful (0 votes)
632 views

Cma 1

1. A bureaucratic organizational structure has advantages such as clearly defined routine activities and performance methods related to tasks. 2. A disadvantage is that creativity within the organization is low due to inflexible standards and impersonal surroundings. 3. A likely effect on employee motivation is that employee performance will be mediocre.

Uploaded by

Frue Love
Copyright
© © All Rights Reserved
We take content rights seriously. If you suspect this is your content, claim it here.
Available Formats
Download as DOC, PDF, TXT or read online on Scribd
0% found this document useful (0 votes)
632 views

Cma 1

1. A bureaucratic organizational structure has advantages such as clearly defined routine activities and performance methods related to tasks. 2. A disadvantage is that creativity within the organization is low due to inflexible standards and impersonal surroundings. 3. A likely effect on employee motivation is that employee performance will be mediocre.

Uploaded by

Frue Love
Copyright
© © All Rights Reserved
We take content rights seriously. If you suspect this is your content, claim it here.
Available Formats
Download as DOC, PDF, TXT or read online on Scribd
You are on page 1/ 83

C. Product.

CMA PART 1 G
Organizational Structures, D. Matrix.

Management
[5] Source: Publisher
And Communication If a company makes a fundamental change in its
organizational structure, resistance by employees
280 Questions
A. Is likely to be minimal if affected persons suffer no
economic loss.
[1] Source: Publisher
Departmentation is the grouping of organizational B. Can be minimized by adopting a participative
subsystems. The greatest advantage of functional management approach.
departmentation is that it
C. Can be minimized by adopting an autocratic
A. Provides the benefits of specialization. management approach.

B. Facilitates communication between primary D. Will be greatest if they receive notice long before
functions. the change is implemented.

C. Helps to focus on achievement of organizational


goals. [6] Source: Publisher
An adhocracy is most likely to be found in what kind of
D. Is appropriate for geographically dispersed organizational structure?
companies.
A. Mechanistic.

[2] Source: CIA 0596 II-32 B. Organic.


The most effective way for a supervisor to delegate a task
to an employee would be to C. Bureaucratic.

A. Define the desired outcome and the approach D. Classical.


precisely and in writing.

B. Define the desired outcome precisely, discuss [7] Source: Publisher


possible approaches with the employee, and reach The term for the organizational design strategy favored by
agreement on the approach to be taken. the classical school of management is

C. Let the employee try to perform the task for a A. Mechanistic.


defined period of time and then meet to critique the
approach, clarify the assignment as needed. B. Organic.

D. Give the assignment in very general terms, have C. Project management.


the employee develop the desired outcome and
approach, and then review and critique the D. Contingency.
employee's decisions.

[8] Source: Publisher


[3] Source: Publisher A large company uses assembly line techniques to
The form of departmentation that most readily lends itself to manufacture a single product. Its choice of relatively
use of profit centers is mechanistic organizational design was more likely based on
its
A. Project.
A. Need for rapid response to environmental change.
B. Functional.
B. Primary concern for operational efficiency.
C. Product.
C. Low fixed technology requirements.
D. Matrix.
D. Project management emphasis.

[4] Source: Publisher


Dual reporting is most characteristic of which method of [9] Source: Publisher
departmentation? A small engineering consulting firm accepts special projects
from customers in a variety of fields. Its choice of an
A. Territorial. organic organizational design was most likely based on its

B. Functional. A. Lack of concern for self-fulfillment of employees.


B. Preference for formal communication. employee's involvement in developing and adapting
his/her job description and required tasks.
C. Adoption of directive leadership style.

D. Participative decision making processes. [14] Source: Publisher


A line position

[10] Source: Publisher A. Is indirectly responsible for achieving the


A characteristic of a bureaucratic organizational structure is organization's basic objectives.
that
B. Makes administrative and operating decisions.
A. It is small and simple.
C. Advises and assists staff positions.
B. Work assignments are rarely delegated.
D. Is indirectly involved with the organization's
C. There is no hierarchy to follow when product(s) and/or services.
communicating.

D. The divisions of labor are distinct. [15] Source: Publisher


Line and staff positions are most likely to be in conflict
because
[11] Source: Publisher
Which of the following is an advantage of bureaucratic A. Line managers have no authority over staff
organizational structures? employees.

A. The routine activities and performance methods B. Staff managers consider line managers' functional
related to a particular task are invariable and clearly authority threatening to their own authority.
defined to the employee.
C. Line managers believe that staff managers are
B. Open positions are filled by individuals recruited resistant to their advice.
from outside the firm to bring in new perspectives and
backgrounds. D. Staff managers dislike relying on line expertise.

C. The organizations are typically decentralized to


allow a subunit manager to make more timely [16] Source: Publisher
decisions. Which of the following is an advantage of a tall
organizational structure?
D. The minimization of the opportunity for social
interaction enhances efficiency. A. The flow of communications from top to bottom
and bottom to top is improved.

[12] Source: Publisher B. The organization's overall objectives are familiar to


A disadvantage of a bureaucratic organizational structure is all employees.
that
C. Employees have more opportunity to advance.
A. The quantity of paperwork prepared is minimal.
D. Labor expenses are reduced.
B. Employee behavior is not controlled because the
organization is too flexible.
[17] Source: Publisher
C. Creativity within the organization is low. Which of the following is a disadvantage of a flat
organizational structure?
D. Organizational goals are de-emphasized because
the work environment is personalized. A. Employees are not encouraged to be creative.

B. The input of fresh ideas from outside the company


[13] Source: Publisher is limited because employee turnover is low.
A likely effect of a bureaucratic structure on employee
motivation is that C. Managers spend too much time training individuals
and not enough time supervising.
A. Employee performance will be mediocre because
of inflexible standards and impersonal surroundings. D. Employees may not be performing work tasks
properly.
B. An employee will pursue organizational goals
because of his/her participation in establishing them.
[18] Source: Publisher
C. Morale will be high because of each employee's Which of the following is a likely effect of a narrow span of
feeling of belonging and ownership. control?

D. Job enrichment will be high because of each A. The manager closely supervises employees.
A. The head of the company is aware of and can
B. Each employee is given more responsibility. influence all decisions before they are made.

C. The organizational structure is flexible. B. The company is operated as one unit.

D. The manager's control over employees is limited. C. Specialists for a particular product have no
authority but advise the company's top management.

[19] Source: Publisher D. Decisions are made on a more timely basis.


When determining the appropriate span of control, the
most important consideration is
[24] Source: CMA 1293 1-22
A. The set of policies and procedures currently in The organizational chart
effect.
A. Is used only in centralized organizations.
B. The typical span of control used by other entities.
B. Is applicable only to profit-oriented companies.
C. The preference of the company's creditors.
C. Depicts only line functions.
D. That all departments will be evaluated, whether or
not they will be affected. D. Depicts the lines of authority linking various
positions.

[20] Source: Publisher


A company's decisions are made solely by Ed Smith, the [25] Source: CMA 1293 1-23
president and major shareholder. Which of the following Which one of the following is generally not beneficial to
powers is Smith least likely to have over the other group decision making in an organization?
shareholders to whom he has delegated some authority?
A. More information.
A. Coercive power.
B. Acceptance.
B. Legitimate power.
C. More knowledge.
C. Referent power.
D. Dominant members.
D. Reward power.

[26] Source: CMA 1293 1-24


[21] Source: Publisher Which one of the following is least likely to contribute to
Which of the following is a reason for delegating? group cohesiveness in an organization?

A. The manager wants to make more decisions. A. Goal consensus.

B. Subordinates lack initiative. B. Subgroup formation.

C. The manager wants to remain the sole expert in C. Leadership acceptance.


his/her field.
D. Interpersonal trust.
D. Subordinates have too many responsibilities.

[27] Source: CIA 0590 III-5


[22] Source: Publisher Henry Mintzberg has suggested that managers at all levels
Which of the following is a reason that a manager may be have roles that can be classified into ten basic types. Three
reluctant to delegate? of these roles can be categorized as interpersonal. Identify
the interpersonal roles.
A. The manager fears being held accountable for the
subordinate's performance. A. Figurehead, leader, and liaison.

B. The manager fears (s)he may fail because of some B. Entrepreneur, resource allocator, and negotiator.
lack of knowledge.
C. Nerve center, disseminator, and spokesperson.
C. The manager did not monitor the subordinate's
work. D. Entrepreneur, disseminator, and liaison.

D. The manager fears unknown goals.


[28] Source: CIA 0591 III-1
A manager participating in negotiating sessions with other
[23] Source: Publisher parties to make sure the organization's interests are
Which of the following is a benefit of decentralization? adequately represented is assuming which of the following
managerial roles as defined by Mintzberg?
D. Functional management, such as the financial
A. Interpersonal role. officer.

B. Informational role.
[33] Source: Publisher
C. External role. The primary difference between centralization and
decentralization is
D. Decisional role.
A. Separate offices for all managers.

[29] Source: CIA 0592 III-3 B. Geographical separation of divisional headquarters


Successful managers come from a variety of backgrounds and central headquarters.
and possess a wide range of traits and skills. However, it is
possible to isolate at least three key preconditions for C. The extent of freedom of decision making by
achieving success as a manager. Those three preconditions many levels of management.
for success can best be expressed by the following:
D. The relative size of the firm.
A. Success = Valence x Expectancy x Effort.

B. Success = Ability x Motivation x Opportunity. [34] Source: CIA 1185 III-5


Which of the following is most likely to be a disadvantage
C. Success is a function of the leader, follower, and of decentralization?
the situation.
A. Lower-level employees will develop less rapidly
D. Success is a function of self-actualization, status, than in a centralized organization.
and acceptance.
B. Top management will have less time available to
devote to unique problems.
[30] Source: CIA 0592 III-6
In a comprehensive study of management reflected in the C. Lower-level managers may make conflicting
book In Search of Excellence, Peters and Waterman decisions.
identify eight attributes of excellence in successful
companies. These attributes encourage focusing on D. Lower-level managers may lose motivation.

A. Customers, employees, and new ideas.


[35] Source: Publisher
B. Planning, organizing, and controlling. Which of the following is not a cost of decentralization?

C. Interpersonal, informational, and decisional roles. A. Dysfunctional decision making owing to


disagreements of managers regarding overall goals
D. Leading, motivating, and communicating. and subgoals of the individual decision makers.

B. A decreased understanding of the overall goals of


[31] Source: CIA 0590 III-4 the organization.
Which level of management would be most involved in the
operational planning of specific tasks? C. Increased costs for developing the information
system.
A. Functional manager.
D. Decreased costs of corporate-level staff services
B. First-line manager. and management talent.

C. Department manager.
[36] Source: CIA 0586 III-5
D. Middle manager. The CEO of a rapidly growing high-technology firm has
exercised centralized authority over all corporate functions.
Because the company now operates in four geographically
[32] Source: CIA 0587 III-19 dispersed locations, the CEO is considering the advisability
Decisions regarding the assignment of employees to of decentralizing operational control over production and
specific jobs on an assembly line rest with which level of sales. Which of the following conditions probably will result
management? from and be a valid reason for decentralizing?

A. Low-level management, such as the first line A. Greater local control over compliance with
supervisor. governmental regulations.

B. Middle management, such as the department B. More efficient use of headquarters staff officials
manager. and specialists.

C. Administrative management (support), such as the C. Less overall operating costs.


personnel director.
D. Quicker and better operating decisions.
D. Formalization of jobs.

[37] Source: CIA 0595 III-34


Which of the following management practices involves [41] Source: CIA 1196 III-1
concentrating on areas that deserve attention and placing When an organization depends to a great extent on its
less attention on areas operating as expected? environment, which of the following statements best
characterizes the relationship among an organization's
A. Management-by-objectives. environment, the level of uncertainty it faces, and its
structure? The more dynamic and complex the
B. Responsibility accounting. environment, the

C. Benchmarking. A. More uncertainty the organization will face and the


more organic the structure should be.
D. Management-by-exception.
B. More uncertainty the organization will face and the
more mechanistic the structure should be.
[38] Source: CIA 1196 III-20
Organizational productivity can be defined as the ratio of an C. Less uncertainty the organization will face and the
organization's total output to its total input, adjusted for more autocratic the structure should be.
inflation, for a specified period of time. For a number of
years, Japan's productivity has been held out as an example D. Less uncertainty the organization will face and the
to emulate. Japan's higher productivity has been mostly more organic the structure should be.
attributed to

A. Abundant raw materials and excellent human and [42] Source: CIA 0595 III-9
financial resources. In what form of organization does an employee report to
multiple managers?
B. Better management and the ability to do more with
less. A. Bureaucracy.

C. Constant refurbishing of the country's B. Matrix.


infrastructure.
C. Departmental.
D. A superior educational system that emphasizes
creativity. D. Mechanistic.

[39] Source: CIA 1196 III-22 [43] Source: CIA 1196 III-3
A new manager of a production department has been The relationship between organizational structure and
asked to assess the effectiveness of that department. The technology suggests that, in an organization using mass
organization needs to satisfy both internal and external production technology (for example, automobile
constituents and takes a broad approach to effectiveness. manufacturing), the best structure is
In order to complete the assignment successfully, the
manager should A. Organic, emphasizing loose controls and flexibility.

A. Measure the daily productivity of the department. B. Matrix, in which individuals report to both product
and functional area managers.
B. Do a survey of employee morale, as it is often a
major underlying factor in productivity. C. Mechanistic, that is, highly formalized, with tight
controls.
C. Compare the past year's production against annual
goals. D. Integrated, emphasizing cooperation among
departments.
D. Consider short-, medium-, and long-term
effectiveness.
[44] Source: CIA 0594 II-22
Which particular type of organization structure will likely
[40] Source: CIA 0596 III-14 have unity-of-command problems unless there is frequent
For the past several years, many organizations have and comprehensive communication between the various
attempted to reduce administrative costs and respond more functional and project managers?
rapidly to customer and competitive demands. One method
is to eliminate layers of middle management. The element of A. Line and staff.
organizational structure affected by such reductions is
B. Strategic business unit.
A. Spatial differentiation.
C. Centralized.
B. Formalization.
D. Matrix.
C. Vertical differentiation.
[45] Source: CIA 0596 III-15 An organization that combines strict adherence to the unity
The structure of an organization generally follows its overall of command with high division of labor may cause
strategy. At one end are loosely structured, organic problems for customers trying to obtain information. Of the
organizations. At the other end are highly centralized, tightly following, which is the most probable type of internal
controlled, mechanistic organizations. Consider the overall environment this structure creates?
strategy of a company that is a pioneer in the combination
of laser and robotic technologies. The company's scientists A. Networked and formal.
and engineers hold many patents. They are continually
looking for ways to improve their products as well as to B. Compartmentalized and informal.
introduce new ones. Identify the most appropriate
structural option for this organization. C. Networked and informal.

A. Mechanistic. D. Compartmentalized and formal.

B. Imitative.
[50] Source: CIA 1195 III-26
C. Organic. With the shift in some countries' economies toward service
industries, a new form of organization has developed. This
D. Bureaucratic. organization structure is referred to as the professional
bureaucracy. While this structure resembles the machine
bureaucracy (which relies on standardized work processes)
[46] Source: CIA 1196 III-7 in several respects, it is different in one key aspect. This
Discount stores and sellers of generic grocery products significant difference is that in a professional bureaucracy
keep prices low and innovate only when there are low-risk,
high-payback projects. They are pursuing a(n) A. Senior management has had to give up a
substantial amount of control.
A. Innovation-minimization strategy.
B. Tasks are accomplished with a high degree of
B. Imitation strategy. efficiency.

C. Cost-minimization strategy. C. There is strict adherence to rules.

D. Initiation strategy. D. There is a tendency for subunit conflicts to


develop.

[47] Source: CIA 1195 III-24


As an organization increases the number of employees, its [51] Source: CIA 1195 III-25
structure becomes more complex. Rules become more A substantial duplication of functions characterizes which of
formalized and more supervisors are hired to direct the the following structures?
increased numbers of subordinates. What is the nature of
the size-structure relationship? A. Simple structure.

A. The size-structure relationship is linear. B. Divisional structure.

B. The structure becomes fixed once an organization C. Machine bureaucracy.


attains a level of about 200 employees.
D. Professional bureaucracy.

C. The size-structure relationship is concave.


[52] Source: CIA 0595 III-27
D. None of the answers are correct. The president of an organization assigned the Chief
Information Officer (CIO) the task of developing a disaster
recovery plan for the organization. The recovery plan was
[48] Source: CIA 1196 III-6 to encompass all activities of the organization, not merely
In general, as organizations grow in size, their strategies information systems. Therefore, the CIO needed to draw
on the expertise of specialists from all over the organization.
A. Become more ambitious, and they often expand Identify the type of organizational structure that would be
their activities within their industry. best to successfully complete this project.

B. Focus on vertical integration, and their structures A. Focus group.


consequently must become more centralized.
B. Reengineering process team.
C. Change from a focus on a diverse set of products
to a focus on a single product line. C. Matrix organization.

D. Follow and are determined by their internal D. Ad hoc committee.


structures.

[53] Source: CIA 1196 III-4


[49] Source: CIA 1195 III-21 Routine tasks, which have few exceptions and problems
that are easy to analyze, are conducive to B. Narrow.

A. Formalized structure, in which procedures C. Moderate.


manuals and job descriptions are common.
D. Wide.
B. Decentralized decision making, in which decisions
are pushed downward in the organization.
[58] Source: CIA 1191 III-3
C. Organic structures that emphasize adaptability and Which of the following factors is least likely to affect a
flexibility to changing circumstances. manager's direct span of control?

D. High degrees of job satisfaction on the part of the A. Frequency of supervisor-subordinate contact.
employees performing them.
B. The manager's willingness to delegate authority.

[54] Source: CIA 1194 III-9 C. The manager's training and communication skills.
Centralization and decentralization are defined according to
the relative delegation of decision-making authority by top D. Number of people in the corporation.
management. Many managers believe that decentralized
organizations have significant advantages over centralized
organizations. A major advantage of a decentralized [59] Source: CIA 0594 III-79
organization is that A flat organization structure is one with relatively few levels
of hierarchy and is characterized by wide spans of
A. Decentralized organizations are easier to control. management control. A tall organization has many levels of
hierarchy and narrow spans of control. Which of the
B. Decentralized structures streamline organizations following situations is consistent with a flat organization
and eliminate duplication of resources. structure?

C. Decentralized organizations have fewer managers A. Tasks require little direction and control of
than centralized organizations. subordinates.

D. Decentralized organizations encourage increased B. Work areas are geographically dispersed.


initiative among employees.
C. Tasks are highly complex and varied.

[55] Source: CIA 1188 III-8 D. Subordinates perform distinctly different tasks.
A claimed advantage of decentralizing is

A. Concentration of authority. [60] Source: CIA 1192 III-9


The optimal span of control of a manager is contingent
B. Manager development. upon several situational variables. For instance, a manager
supervising workers within the same work area who are
C. Elimination of duplication of effort. performing identical tasks that are simple and repetitive
would best be able to supervise
D. Departmentalization.
A. An unlimited number of employees.

[56] Source: CIA 0585 III-8 B. Only a few workers (a narrow span of control).
The difference between a tall organization structure and a
flat organization structure is that in the former C. A relatively large number of employees (a wide
span of control).
A. The communication process takes longer and is of
poorer quality. D. Fewer workers than if the workers were
geographically dispersed.
B. Maintenance of the organization is less costly.

C. The morale of lower-level employees is generally [61] Source: CIA 0596 III-19
higher. A printing company changes its type of ink to a nontoxic
variety due to unfavorable publicity by a local
D. A higher degree of coordination and cooperation environmental group. This is an example of an
is created. organizational change called

A. Anticipatory.
[57] Source: CIA 1188 III-9
The most likely span of control to apply over 14 data-entry B. Reactive.
clerks who do essentially the same job and work in the
same office would be C. Incremental.

A. Close. D. Strategic.
B. Group decision making is almost always less
[62] Source: Publisher efficient than individual decision making.
Power is synonymous with leadership. Simply, it is the
ability to influence other people. The sources of power are C. Although group members frequently have diverse
various. For example, the kind of power arising from the views about a decision, each member's desire to be
strength of the leader's personality is known as accepted by the group often restrains open
disagreement.
A. Coercive power.
D. Group decision making tends to be less creative
B. Legitimate power. than individual decision making.

C. Expert power.
[67] Source: CIA 1195 II-3
D. Referent power. An operational audit of your organization's budget process
has revealed that department heads frequently circumvent
budget controls by actions such as padding their budgets
[63] Source: Publisher and concealing opportunities to cut costs. One of the audit
The director of internal auditing for a large company has recommendations involves forming a group comprised of
established an excellent reputation because of her strong department heads to find a solution for this problem. Which
professional credentials and tactful but firm handling of one of the following methods is likely to generate a high
auditor-auditee relationships. With regard to auditees, she level of commitment to the solution among members of the
must rely upon what sources of power? group?

A. Expert, coercive. A. The brainstorming technique in which group


members orally identify as many alternatives as
B. Referent, reward. possible without criticism from other members.

C. Referent, expert. B. A mandate from top management.

D. Legitimate, coercive. C. The Delphi technique in which a series of


questionnaires is used to arrive at a consensus.

[64] Source: Publisher D. An open discussion of the problem and potential


The weakness of using the Golden Rule as a management solutions until a consensus is reached.
philosophy is that

A. It does not work any more. [68] Source: CIA 1195 II-33
Which one of the following statements about quality circles
B. There is an assumption that the workers want is false?
what the leader wants.
A. A quality circle is typically comprised of a group
C. Workers fear the consequences of disobeying. of 8 to 10 subordinates and supervisors.

D. There is only one best way to perform a job. B. Part of the quality circle concept includes teaching
participants communication skills, quality strategies,
and problem analysis techniques.
[65] Source: Publisher
The Hawthorne studies in the 1920s and early 1930s C. Quality circles meet on the company premises and
showed that on company time.

A. Workers did not respond directly to a physical D. The quality circle has the final control over
change but rather to their perception of the change. implementation of recommended solutions.

B. Wages were found to be more important than


social acceptance is determining individual output. [69] Source: CIA 1196 II-37
Which of the following is not an advantage of group
C. An industrial engineering approach to management decision making as compared to individual decision
was preferable to classical management theory. making?

D. Workers responded directly to physical changes A. Groups obtain an increased degree of acceptance
in their environment. of a solution so that it may be more easily
implemented.

[66] Source: CIA 1195 II-1 B. Group decision making is consistent with
Which of the following statements about group decision democratic methods.
making is generally considered false?
C. Group members bring more complete information
A. There is a lack of responsibility for group and knowledge into the decision process.
decisions.
D. Group members avoid expressing opinions that
deviate from what appears to be the group developed a system based on managerial roles. The three
consensus. basic role categories defined by Mintzberg are

A. Planning, organizing, and controlling.


[70] Source: CIA 1194 III-8
A manager can use power and authority to accomplish B. Interpersonal, informational, and decisional.
objectives. The relationship between these two important
concepts is best explained as follows: C. Decision making, staffing, and communicating.

A. Power is the right to do things, while authority is D. Motivation, leadership, and delegation.
the ability to do things.

B. Authority is the right to do things, while power is [75] Source: CIA 0593 III-3
the ability to do things. An example of an appropriate interpersonal role carried out
by a manager in an organization would be
C. Power and authority are both required to
accomplish a task. A. Designing and initiating changes within the
organization.
D. Power and authority are simply two words that
describe the same concept - - how to get things done B. Transmitting selected information to subordinates.
in organizations.
C. Participating in negotiating sessions with other
parties (vendors, unions, etc.).
[71] Source: CIA 1193 III-3
"The process of working with and through others to D. Motivating subordinates to get the job done
achieve organizational objectives in a changing properly.
environment" is a good definition of

A. Referent power. [76] Source: CIA 0596 III-26


All of the following are examples of the management
B. Management. function of decision making except

C. Informal leadership. A. Participating in negotiating contracts with vendors.

D. Motivation. B. Motivating subordinates to get the job done


properly.

[72] Source: CIA 1193 III-1 C. Taking corrective action in nonroutine situations.
Which level of management is most concerned with
determining how specific tasks can be accomplished with D. Allocating organizational resources.
available resources by a given date?

A. First-line manager. [77] Source: CIA 0595 III-31


The managerial role of the CEO in communicating selected
B. Department head. information regarding the cost-cutting measures to the
stock market analysts would be categorized as
C. General manager.
A. Liaison.
D. Chief executive officer.
B. Spokesperson.

[73] Source: CIA 0592 III-9 C. Leader.


A lumber mill must carefully plan production for 160
different varieties of plywood. To do this, a linear D. Figurehead.
programming model is used to generate a 6-month
production schedule that is adapted to a weekly plan. The
level of management responsible for this type of planning is [78] Source: Publisher
(Refer to Figure 7.) Which of the following is a staff
A. The board of directors. position in the Fisher Bank?

B. Top management. A. The manager of customer service-checking, the


department that handles checking account inquiries
C. Middle management. and transactions.

D. First-line supervision. B. The director of personnel, the department that


handles the hiring, firing, promotion, etc., of all
employees.
[74] Source: CIA 0590 III-6
Henry Mintzberg believed that the functional approach for C. The chief loan officer, who is in charge of final
describing what managers do was inadequate. He approval of all loans.
B. Job rotation.
D. The vice president in charge of the mortgage
department, which handles mortgages on business C. Job enrichment.
and residential property.
D. Job simplification.

[79] Source: Publisher


(Refer to Figure 7.) The manager of customer service [83] Source: Publisher
checking at Fisher Bank is most likely to encounter According to management literature, worker absenteeism,
conflict(s) because of turnover, and error rates are likely to be greatest

A. The department's reliance on the savings manager A. When workers alternate in performing different
jobs.
and the money market manager.
B. In simplified assembly-line jobs.
B. His/her providing services for customers who do
not have checking accounts. C. In job sharing situations.

C. The requirement that (s)he must report directly to D. When flexible working hours are provided.
both the president and the vice president for
customer accounts.
[84] Source: Publisher
D. A potential disagreement with the director of Job enrichment is a motivational approach used by
personnel about a subordinate's performance management that
appraisal.
A. Emphasizes the need for close supervision.

[80] Source: Publisher B. Is based on Maslow's analysis of survival needs.


Which ethical standard is most clearly violated if a financial
manager/management accountant knows of a problem that C. Is based on Herzberg's analysis of factors extrinsic
could mislead users but does nothing about it? to the work.

A. Competence. D. Applies the principle of worker participation.

B. Legality.
[85] Source: Publisher
C. Objectivity. Job enlargement is typified by

D. Confidentiality. A. Horizontal loading of the job.

B. Vertical loading of the job.


[81] Source: Publisher
The IMA Code of Ethics includes a competence standard, C. Increased worker control of tasks.
which requires the financial manager/ management
accountant to D. More rapid performance feedback.

A. Report information, whether favorable or


unfavorable. [86] Source: Publisher
A company that wishes to improve its rate of retention of
B. Develop his/her professional proficiency on a its experienced employees might
continual basis.
A. Abolish its hire-from-within policy.
C. Discuss ethical conflicts and possible courses of
action with an unbiased counselor. B. Improve its fringe benefit package.

D. Discuss, with subordinates, their responsibilities C. Initiate job simplification programs.


regarding the disclosure of information about the firm.
D. Set a mandatory retirement age.

[82] Source: Publisher


Job design involves not only the intentional planning of the [87] Source: Publisher
characteristics of a job but also of the work environment. The factor that would not contribute to retention of
The purpose is to increase workers' intrinsic job motivation experienced employees would be
so as to improve performance. It is an approach that
explicitly considers workers' ego and social needs. The A. Initiation of job enrichment programs.
aspect of job design that should have the greatest
motivational effect is B. Adherence to federal regulations regarding sex
discrimination in paying and promoting workers.
A. Job enlargement.
C. Paying new employees more than older ones.
D. Adoption of flextime rules. [93] Source: Publisher
Victor Vroom's expectancy theory is based on the
intuitively appealing idea that people have expectations of
[88] Source: Publisher rewards derived from their unique personal motive
The value of retaining employees is determined by structure, from their beliefs as to what are important
rewards to them, and from their expectations of getting
A. Human asset accounting. these incentives if they exert effort. The perceived equity of
rewards leads to satisfaction or dissatisfaction and thus
B. Financial accounting. feeds into the next cycle's expectancies. For example, an
employee who believes (s)he is compensated less well than
C. Cost accounting. another worker performing the same task with the same
degree of skill may become dissatisfied. An important
D. Human resource planning. concept in the expectancy theory is

A. Theory X and Theory Y.


[89] Source: Publisher
Leadership situations vary with regard to the degree to B. Motivation = Valence x Expectancy.
which the leader can determine what subordinates will do,
how they will do it, and what the results will be. According C. The two-factor theory of behavior.
to Fiedler's contingency theory, a leader with a
relationship-oriented management style will be most D. The management grid.
effective when (s)he exerts

A. Great control. [94] Source: Publisher


A company that formerly paid certain management
B. Moderate control. employees on a salary plus commission basis decided to
compensate a test group solely with commissions.
C. Little control. Performance of these employees declined. The most likely
explanation for this result is that
D. Great or little control.
A. The employee received special attention.

[90] Source: Publisher B. Compensation was a motivational factor.


According to the contingency theory of leadership, a
manager will be most effective when (s)he C. Compensation was not a hygiene factor.

A. Consistently initiates structure. D. Increased concerns about security made the


employees risk averse.
B. Adapts his/her style to specific circumstances.

C. Is task-oriented. [95] Source: Publisher


A company has a compensation system for its managers
D. Is relationship-oriented. based on a management-by-objectives (MBO) approach.
The essential premise of MBO is that

[91] Source: Publisher A. Compensation should be based on qualitative


Maslow's theory of motivation is based on a hierarchy of factors.
human needs. The need satisfied by greater income is
B. Employees should be concerned with routine
A. Self-actualization. matters, and managers should attend to exceptions.

B. Safety. C. Employees should participate in setting goals.

C. Participation. D. Managers should establish goals for their


employees.
D. Authority.

[96] Source: Publisher


[92] Source: Publisher A Theory X manager most likely believes that employees
According to the behavioral theory of management,
A. Require little supervision.
A. Employees are motivated to fulfill needs.
B. Are creative and imaginative.
B. Morale problems are not goal related.
C. Need direction and security.
C. Compensation is a universal motivator.
D. Solve problems outside their immediate control.
D. Productivity is not correlated with job satisfaction.
[97] Source: Publisher D. An objective evaluation.
Which of the following is a weakness of using MBO for
motivating employees?
[102] Source: Publisher
A. Employees may believe that they control situations A disadvantage of separating performance evaluations from
that arise in the company. wage-increase decisions is that

B. Employees may be forced to place too much A. Not enough emphasis is placed on short-run
emphasis on quantitative factors. performance.

C. Employees may participate too much in the B. Financial rewards may lose their motivational
goal-setting process. effect.

D. Employees may become too trusting and too C. Employees may not be motivated by good
dependent on the "team" to get things done. appraisals.

D. The employee's performance evaluation does not


[98] Source: Publisher consider the financial status of the company overall.
MBO managers are most likely to believe that employees

A. Dislike their work. [103] Source: Publisher


A company allows each of its departments to develop its
B. Avoid responsibility whenever possible. own system for evaluating performance. Linda Ward, the
personnel director, should communicate appraisal
C. Work best when threatened with punishment. information to a new employee by

D. Are self-motivated. A. Presenting anything that is pertinent to the listener's


situation.

[99] Source: Publisher B. Providing an overview of all systems within the


What is the first step to take when implementing MBO in a company.
department accustomed to Theory X?
C. Describing how her own performance is
A. Teach the employees about MBO. evaluated.

B. Set up individual goals with each employee on a D. Discussing each department's evaluation system in
one-to-one basis. detail.

C. Discuss overall organizational goals with the


[104] Source: Publisher
employees. Rupert is a manager who believes that his department's
most valuable resource is the employees' time. He enforces
D. Verify that top management supports MBO. a set of rigid rules for employees. A characteristic of this
leadership style is that

[100] Source: Publisher A. Employees are encouraged to participate in


Evaluating performance is NOT done to decision making.

A. Determine the amount of nondiscriminatory B. Personal interaction among employees is limited.


benefits that each employee deserves.
C. Organizational objectives are coordinated with
B. Assess the available human resources of the firm. each employee's goals.

C. Motivate the employees. D. The manager's perceptions are similar to


McGregor's Theory Y.
D. Determine which employees deserve salary
increases.
[105] Source: Publisher
Marianne is a manager who believes that positive employee
[101] Source: Publisher attitudes are extremely important. She cooperates with
When a manager generalizes from the evaluation of one or employees in solving problems. A likely effect on employee
a few traits to the employee's total performance, (s)he has behavior of this leadership style is
made
A. Mistrust of the manager.
A. A judgmental evaluation.
B. A lack of extraordinary performance.
B. An evaluation subject to the halo effect.
C. High employee turnover.
C. A projection.
D. Increased employee creativity.
[110] Source: Publisher
A company's compensation program does not allow for
[106] Source: Publisher salary increases based on above-average performance
A manager implementing the directive leader approach after an employee reaches the top of a position's salary
should range. It also pays some employees higher salaries because
of their educational qualifications. This company could
A. Closely supervise each employee. improve the program by

B. Display confidence in each employee's ability. A. Allowing each manager to set up his/her own
system of salary increases.
C. Work with the employee when developing goals.
B. Providing only monetary compensation.
D. Clearly signal that the employee is expected to be
successful. C. Developing a system that equates years of
experience with education.

[107] Source: Publisher D. Decreasing the annual percentage increase in each


Which of the following is a benefit of implementing the salary range.
achievement-oriented leader approach rather than the
directive leader approach?
[111] Source: Publisher
A. Employee development is enhanced. Which of the following conditions would motivate
employees to improve performance under a merit pay
B. The structured environment allows employees to system?
better achieve the organization's goals.

C. Closer supervision is provided for those who A. Ideal goals are stressed.
perform better in a structured work atmosphere.
B. The company's culture emphasizes outstanding
D. Employees have more opportunities to develop performance.
creativity and meet challenges.
C. Different levels of performance have minimal
differences in pay.
[108] Source: Publisher
An employee with little educational background but many D. Job descriptions are invariably set with the
years of experience knows that other employees at the required tasks clearly outlined.
same level receive higher salaries because of their
educational qualifications. (S)he is often asked for help by
others because of his/her years of experience. The most [112] Source: CIA 0593 III-11
likely behavioral effect is that the employee will A leader who is able to gain compliance from a group
based solely on personal attraction is said to have
A. Become less productive.
A. Reward power.
B. Continue to help the other, more educated
employees. B. Coercive power.

C. Request a transfer. C. Referent power.

D. Seek a new job. D. Legitimate power.

[109] Source: Publisher [113] Source: CIA 0591 III-14


An employee with a good background and years of A production worker in a plant often speaks for the entire
experience earns a salary at the top of his/her range. Under work force when problems arise between labor and
the company's compensation program, the employee can management. Although this individual has the same level of
increase his/her salary above the usual annual increase only authority and expertise as his/her co-workers, the worker
by earning a promotion. Which of the following is most seems to possess a degree of power that others do not
likely to be an effect on his/her behavior? have. What type of power does this individual apparently
have?
A. The employee may refuse new duties or tasks.
A. Coercive.
B. The employee may become less productive.
B. Referent.
C. The employee may not be motivated to improve
performance. C. Legitimate.

D. The employee may seek a position with another D. Reward.


company.
[114] Source: CIA 0594 III-57 are met.
Which of the following is an example of an efficiency
measure? C. Job esteem improves as physiological needs are
met.
A. The rate of absenteeism.
D. Job esteem improves as job satisfaction increases.
B. The goal of becoming a leading manufacturer.

C. The number of insurance claims processed per [119] Source: CIA 0594 III-49
day. Alternative work schedules for employees are said to
increase the efficiency of business operations. They are
D. The goal of increasing market share. consistent with the underlying concepts of which theory?

A. Motivation-Hygiene theory.
[115] Source: CIA 1193 III-11
If you were designing a new position in an organization, B. Theory X.
which of the following design techniques would you use to
increase the motivation of the person filling the position by C. Equity theory.
adding responsibility and authority?
D. Cognitive Evaluation theory.
A. Job enlargement.

B. Job rotation. [120] Source: CIA 0594 III-48


Alternative work schedules for employees are said to
C. Job enrichment. increase the efficiency of business operations. Which of the
following is not a type of alternative work schedule?
D. Job significance.
A. A worker works 40 hours within a four-day
period.
[116] Source: CIA 0592 III-15
Frederick Herzberg postulated a two-factor theory of B. Workers are allowed discretion over when they
human behavior that included satisfiers and dissatisfiers. arrive and leave work.
Which of the following is a dissatisfier?
C. A worker rotates to a job at the same level with
A. Promotion to another position. similar skill requirements.

B. Salary. D. A worker performs tasks at home using a


computer and a modem to access the company's
C. Challenging work. information system.

D. Responsibility.
[121] Source: CIA 0591 III-11
During an audit of the personnel function, it was noted that
[117] Source: CIA 0592 III-16 numerous questions were asked of potential hirees. Which
Which of the following is not correct concerning job of the following questions would normally be appropriate
motivation? when interviewing an inexperienced person applying for an
entry-level internal auditing position?
A. Increased planning and decision making in a job is
a positive motivator only if accompanied by a salary A. How have you kept up your internal auditing
increase. education?

B. Recognition of achievement motivates B. What kind of reports have you written in previous
performance. jobs?

C. Poor working conditions create dissatisfaction C. What are your career goals?
with the job.
D. What is your marital status?
D. Poor interpersonal relations create dissatisfaction
with the job.
[122] Source: CIA 1192 III-10
When faced with the problem of filling a newly created or
[118] Source: CIA 1192 III-13 recently vacated executive position, organizations must
Both Maslow and Herzberg have developed popular decide whether to promote from within or to hire an
motivational theories. Which statement best distinguishes outsider. One of the disadvantages of promoting from
Herzberg's theory? within is that

A. Job performance improves as job satisfaction A. Internal promotions can have negative motivational
increases. effect on the employees of the firm.

B. Job performance improves as physiological needs B. Internal promotions are more expensive to the
organization than hiring an outsider.
[127] Source: CIA 0596 II-33
C. It is difficult to identify proven performers among A manager discovers by chance that a newly hired
internal candidates. employee has strong beliefs that are very different from the
manager's and from those of most of the other employees.
D. Hiring an insider leads to the possibility of social The manager's best course of action would be to
inbreeding within the firm.
A. Facilitate the reassignment of the new hire as
quickly as possible before this situation becomes
[123] Source: Publisher disruptive.
Motivation is
B. Ask the rest of the team for their reaction and act
A. The extent to which goal-specific performance is according to the group consensus.
recognized by supervisors.
C. Take no action unless the new hire's behavior is
B. The extent to which individuals have the authority likely to cause harm to the organization.
to make decisions.
D. Try to counsel the new hire into more reasonable
C. The extent of the attempt to accomplish a specific beliefs.
goal.

D. The desire and the commitment to achieve a [128] Source: CIA 0595 II-35
specific goal. The internal auditor faces two potentially conflicting trends
when formulating the presentation of audit reports: (A) the
information age and the increase in information available for
[124] Source: CIA 0594 III-88 decision making; and (B) the concept of "bounded
People with a high need for achievement usually do not: rationality," which pertains to the ability of the individuals to
process data and make decisions. Which of the following
A. Perform best when they perceive risks are high. approaches to developing an audit report would be
consistent with the notion of bounded rationality?
B. Seek situations in which they can attain personal
responsibility for problem solving. A. Identify the information most available to
encourage the user to make an optimal decision.
C. Function effectively as salespeople.
B. Identify the problem, but let the decision maker
D. Prefer frequent and unambiguous feedback. independently search the databases for the problem
solution.

[125] Source: CIA 1194 II-30 C. Present data in graphic form to focus on the
As a manager, you should be striving for a high level of job major, more simple aspects of complex problems.
satisfaction for your staff for all the following reasons
except D. Suggest that a short time limit be placed on the
time to make a decision so that the decision maker
A. A happy, satisfied worker is always a more will concentrate on taking effective action in a timely
productive worker. fashion.

B. High job satisfaction usually results in lower


turnover. [129] Source: CIA 0594 III-50
Which of the following is not an advantage of teamwork
C. Dissatisfied employees are often less healthy. compared with work performed by individuals?

D. Many people feel job satisfaction is as important A. Teams provide support to the team members.
as remuneration.
B. Teams make decisions that are more easily
accepted.
[126] Source: CIA 1196 II-38
Which of the following statements is correct with respect to C. Teams provide a clear link between effort and
a change in values? outcome.

A. Values are neither stable nor enduring. D. Teams control and discipline members.

B. The process of questioning values will result in a


change. [130] Source: CIA 1196 II-26
If a supervisor uses a supportive management approach,
C. Values are not fixed, and when they change, they evidenced by positive feelings and concern for
change quickly. subordinates, a problem might result because

D. Values are established in early years and are A. An approach based on pure power makes it
unlikely to change. difficult to motivate staff.
B. This approach depends on material rewards for [135] Source: CIA 0594 III-89
the worker. The punishing of employees is made less effective by:

C. This approach depends on people who want to A. Stating the offending behavior specifically.
work, grow, and achieve.
B. Postponing the start of disciplinary procedures.
D. The manager must believe in the teamwork
approach. C. Permitting employees to challenge their culpability.

D. Focusing the discussion on the offending behavior


[131] Source: CIA 1196 II-27 instead of the offender.
Some behavioral models stress employee participation as a
key to motivation. A limitation of the participative approach
is [136] Source: CIA 1195 II-40
When supervising employees, the behavior most likely to
A. Workers are intrinsically lazy and must be driven. attain long-term positive results for a manager would be to

B. A number of dissatisfiers must be present in order A. Discipline employees immediately for undesirable
for the approach to work. behaviors, using oral reprimands, written warnings,
and temporary suspensions.
C. It is difficult to elicit the participation of all
employees. B. Hold weekly meetings during which employees are
reminded of work procedures and are praised for the
D. Unresolvable conflicts arise when a mature, week's accomplishments.
capable, creative person joins a structured,
demanding, and limiting organization. C. Praise employees on a random schedule and link
rewards to performance.

[132] Source: CIA 1193 III-2 D. Tell employees that working overtime now will
Which of the following is most likely to provide for the result in a better performance review in 6 months.
continual development of managerial skills?

A. Organizational orientation sessions. [137] Source: CIA 0596 II-36


During an exit conference, the most effective way to
B. Job rotation. motivate an auditee to adopt a recommendation on a
control weakness would be to
C. Role playing.
A. Remind the auditee that audit reports are read by
D. Liaison committees. senior management.

B. Inform the auditee that other line managers with


[133] Source: CIA 1194 II-28 fewer resources have been able to remedy similar
The least effective method for an employee to learn situations.
computer skills is
C. Try to provide conclusive proof that the findings
A. Classroom training using equipment. are correct and the conclusions are sound.

B. Video training. D. Compliment the auditee on improvements already


adopted and encourage further changes.
C. Apprenticeships.

D. Case analysis. [138] Source: CIA 0595 III-25


A manager has a small team of employees, but each
individual is self-motivated and could be termed a "high
[134] Source: CIA 0594 II-9 achiever." The manager has been given a particularly
Which of the following is not an example of positive difficult assignment. Even for a high achiever, the
reinforcement of behavior? probability that this job can be completed by one individual

A. Paying a bonus to employees who had no by the required deadline is low. Select the best course for
absences for any four-week period. the audit manager.

B. Giving written warnings after only every other A. Assign one individual since high achievers thrive
absence. on high risks.

C. Assigning a mentor to each employee. B. Assign two employees to moderate the risk of
failure.
D. Having a lottery every month where 10% of the
employees with no absences receive a $200 bonus. C. Assign all employees to ensure the risk of failure is
low.
D. Ask company management to cancel the job. other manager felt that specific, difficult goals produce the
best results. As the discussion continued, other methods of
goal setting were identified. Select the best method for
[139] Source: CIA 1196 III-28 setting goals.
In many jobs, excessive specialization can eventually lead
to poor motivation, boredom, and alienation. In order to A. The manager should provide generalized goals.
cope with the potential problems in such a situation,
managers should B. The manager should select specific, difficult goals.

A. Focus on their employees' higher-level needs in C. The employee should develop generalized goals
order to help them achieve self-actualization. and obtain management concurrence.

B. Remove dissatisfiers such as low salary, bad D. The employee should develop specific, difficult
supervision, lack of job security, and poor working goals and obtain management concurrence.
conditions.

C. Implement an optimal organizational rewards [143] Source: CIA 1195 III-23


system and provide all needed training to keep A production manager is working on a plan to increase
employees up to date on technology. employee motivation at a local plant by comparing results
of employee interviews to those from employees at a higher
D. Change the jobs to fit the employees' needs or performing plant. The manager is puzzled about the results
rotate employees to jobs that satisfy their needs. of two sets of employee interviews. The employees
interviewed performed identical jobs. Local employees
thought their jobs were boring and unfulfilling while the
[140] Source: CIA 0596 III-34 other plant's employees were very satisfied. Even new
The basic underlying principle of the quality-of-work-life employees at the local plant indicated they quickly lost
view of motivation suggests that an organization should enthusiasm. Identify the primary element the manager must
unlock the creative potential of its people by overcome to enhance performance.

A. Involving them in decisions affecting their lives and A. Negative job perception.
providing them with more control over their work.
B. Inadequate job autonomy.
B. Providing fair and equitable reward systems that
are clearly linked to the employees' effort and C. Lack of skill variety.
performance.
D. Low task significance.
C. Focusing on employees' higher-level needs.

D. Using job enrichment techniques that increase skill [144] Source: CIA 1195 III-2
variety, task identity and significance, autonomy, and A new production team has been formed by taking
feedback. experienced high achievers from existing teams within the
factory. The members of the new team have not been
required to learn any new skills, and the machines used are
[141] Source: CIA 1195 III-22 identical to those used in their former teams. The team's
The human resource department of an organization production supervisor is a longtime employee of the
observed that accounting staff turnover was unusually high. organization but has not previously worked with any
Exit interviews indicated that the accounting department members of the new team. Despite the abilities and
work schedule was highly restrictive for accountants who previous individual achievements of the individual team
had young children. To improve the retention of skilled members, management is surprised by the mediocre
employees in the accounting department, the best solution performance of the new team. The best approach for the
would be to production supervisor to improve performance would be to

A. Implement a program of job rotation within the A. Increase pressure on the team through higher
accounting department. goals and reprimands.

B. Promote job enlargement for the positions B. Replace these individuals on the team.
experiencing the greatest turnover.
C. Provide opportunities for the team members to
C. Provide job sharing and flextime opportunities for socialize with each other.
accounting department employees.
D. Do nothing now because it is too soon to draw
D. Enrich the jobs of accounting department any conclusions.
employees.

[145] Source: CIA 0595 III-7


[142] Source: CIA 0596 III-33 There are numerous factors that enhance or detract from a
Two managers were discussing the merits of goal setting to group's attractiveness and cohesiveness. Select the factor
improve employee performance. One manager felt that that enhances cohesiveness.
specific goals should not be established and that, to provide
for flexibility, only generalized goals should be used. The A. Favorable view from the outside-looking-in.
A. Directive.
B. Wide variety of members.
B. Supportive.
C. A common threat in the environment.
C. Participative.
D. Moderately large size.
D. Achievement-oriented.

[146] Source: CIA 0595 III-8


The key ingredient to group effectiveness is [149] Source: CIA 0596 III-6
(Refers to Fact Pattern #1)
A. Challenge. The manager of a team of actuaries has been asked to
develop the basic pricing structure for a new health
B. Trust. insurance product. The team has successfully designed
other pricing structures in recent years. The manager was
C. Norms. assigned to the team 6 months ago. What is the best
leadership style for the manager of this team?
D. Roles.
A. Directive.

[Fact Pattern #1] B. Supportive.


The following question presents a scenario in which a
manager needs to decide what leadership style to use to C. Participative.
obtain employee satisfaction and effective employee
performance. For the purposes of this question, the D. Achievement-oriented.
manager has a choice of four styles.

The directive leader lets subordinates know what is [150] Source: CIA 0596 III-7
expected of them, schedules work to be done, and gives (Refers to Fact Pattern #1)
specific guidance on how to accomplish tasks. A manager in a government agency supervises a section of
clerical employees who review license applications for
The supportive leader is friendly and shows concern for the approval or denial. The clerical jobs are well defined
needs of the subordinates. procedurally and are covered by government regulations.
In this case, what is the best leadership style for the
The participative leader consults with subordinates and manager?
uses their suggestions before making a decision.
A. Directive.
The achievement-oriented leader sets challenging goals and
expects subordinates to perform at their highest level. B. Supportive.

[147] Source: CIA 0596 III-4 C. Participative.


(Refers to Fact Pattern #1)
The workers in a factory have been told that their machines D. Achievement-oriented.
are obsolete and will be replaced by new,
computer-assisted machines. The workers must be
retrained and are eager to learn everything about the new [151] Source: CMA 0696 1-20
machines. The manager was recently hired from a company The protected group under the Age Discrimination in
where the new machines were extensively used and is very Employment Act of 1967, as amended, is defined as
familiar with them. In this case, what is the best leadership anyone in which age group?
style for the manager?
A. 35 and 60.
A. Directive.
B. 40 and 75.
B. Supportive.
C. 40 and older.
C. Participative.
D. 45 and older.
D. Achievement-oriented.

[152] Source: CIA 0595 II-34


[148] Source: CIA 0596 III-5 There has been an increased emphasis on group decision
(Refers to Fact Pattern #1) making in organizations. Which of the following statements
A production team has been together for several years and has been found to hold true in studies of individual decision
has worked well together. However, severe arguments making as compared to group decision making? Individual
have recently occurred between two members of the decision making tends to
group, and other members have begun to take sides. This
problem has had a negative effect on production A. Be more conservative.
performance. The best leadership style for the manager in
this situation is B. Evaluate more complete information.
C. Generate more alternatives. accordance with relevant standards is acting contrary to
which one of the following standards?
D. Increase the perceived legitimacy of the decision.
A. Competency.

[153] Source: CIA 1196 II-34 B. Confidentiality.


A manager believes that positive reinforcement is the most
appropriate way to deal with employees. Which of the C. Integrity.
following actions demonstrates the principle of positive
reinforcement? D. Objectivity.

A. Employees are given 2-day suspension without


pay if errors exceed a predefined level. [157] Source: Publisher
The military command structure is an example of what kind
B. Employees are praised when the detected error of communication network?
rate in their work stays below a pre-defined level.
A. Wheel.
C. Time budgets, which have forced employees to
rush and consequently make errors, are eliminated. B. All-channel.

D. Employees are not required to work overtime if C. Star.


errors stay below a predefined level.
D. Chain.

[154] Source: CMA Samp Q1-10


Clear Connections Inc., the largest provider of mental [158] Source: Publisher
health services in its tri-county area, was encountering Which communication network is best suited to complex
personnel problems. Their 25 residences housed many group tasks requiring a high level of creativity?
vulnerable clients, but funding never seemed adequate to
hire quality, live-in staff. A new administrator, Deborah A. Wheel.
Romano, is determined to facilitate long-term employment
of the best possible care-giving staff. Besides paying better B. Circle.
wages, she feels it is important that the staff be strongly
motivated by the work itself. According to Maslow's need C. All-channel.
hierarchy, the best employees would have a need for
D. Star.
A. Esteem.

B. Belongingness. [159] Source: Publisher


Communication is the process by which meaning is
C. Self-actualization. conveyed between people. According to communications
theory
D. Safety and security.
A. The process has three elements: sender, medium,
receiver.
[155] Source: Publisher
Financial managers/management accountants are obligated B. One-directional communication is usually most
to maintain the highest standards of ethical conduct. effective.
Accordingly, the IMA Code of Ethics explicitly requires
that they C. Communication is the message actually received,
not what is transmitted.
A. Obtain sufficient competent evidence when
expressing an opinion. D. Informal communications are usually to be
avoided because of their inaccuracy.
B. Not condone violations by others.

C. Comply with generally accepted auditing [160] Source: Publisher


standards. Communication must occur in various directions within an
organization. The kind of communication that is the slowest
D. Adhere to generally accepted accounting is
principles.
A. Upward.

[156] Source: CMA 3 B. Downward.

In accordance with Statements on Management C. Horizontal.


Accounting Number 1C (SMA 1C) (revised), Standards
of Ethical Conduct for Practitioners of Management D. Diagonal.
Accounting and Financial Management, a management
accountant who fails to perform professional duties in
[161] Source: Publisher
Because communication is vital to effective management, C. Distributing a proposed agenda for the meeting to
managers spend most of their time communicating. all participants.
Consequently, managers
D. Requesting an advance copy of all supporting
A. Who are good communicators will be effective. information to be brought to the meeting by each
participant.
B. Mostly engage in oral communication.

C. Devote most of their time to formal, written [166] Source: Publisher


communication. A special, mandatory meeting was announced in a
memorandum. What action should the chair take during the
D. Are essentially senders of messages. meeting to make it more productive?

A. Direct individual responses offered by


[162] Source: Publisher participants.
Communications networks in organizations may take
various forms. The network in which misunderstanding is B. Take the minutes.
most likely is the
C. Discuss at the beginning of this meeting the
A. All-channel. minutes from the last monthly meeting.

B. Hub. D. Discuss any topics relating to the special purpose


of the meeting in the random order that they are
C. Wheel. introduced by participants.

D. Chain.
[167] Source: Publisher
Which of the following actions should a chair take following
[163] Source: Publisher
Communication is most likely to be effective when an important, mandatory meeting announced in a
memorandum?
A. The sender carefully considers the receiver's
needs. A. Place the minutes and any documented
information shared at the meeting in a file to which
B. Sender and receiver concentrate only on verbal (s)he alone has access.
messages.
B. Distribute a list of any individual assignments made
C. It changes the receiver's attitude. during the meeting to all company personnel.

D. The receiver monitors the sender's behavior. C. If any problems arise, call a follow-up meeting
announced to all participants at a moment's notice.

[164] Source: Publisher D. Request status reports on projects assigned to


A memorandum to announce an important, mandatory individuals during the meeting.
meeting should contain several details, including

A. A list of all of the people who may be affected by [168] Source: Publisher
the meeting. Which of the following devices translates the intended
message into the sender's chosen communication form?
B. The expected length of the meeting.
A. Transmitter.
C. A brief description of the topic of the next monthly
meeting. B. Encoder.

D. The person to contact if the employee has not C. Channels.


received the memorandum.
D. Decoder.

[165] Source: Publisher


After writing a memorandum announcing a meeting but [169] Source: Publisher
before the meeting occurs, the writer/chair should take Which of the following perceptual errors is the tendency for
one party to attribute his/her traits to the other party when
several actions, including communicating?

A. Checking with his/her superior for approval to call A. Expectancy.


the meeting.
B. Projection.
B. Calling the participants to find out if they have any
questions or problems. C. The halo effect.
D. Employee speculation, mistrust, and paranoia
D. Stereotyping. increase.

[170] Source: Publisher [174] Source: Publisher


Which of the following actions is most likely to minimize the Which of the following results is most likely to occur when
effect of perceptual errors? public media are used to resolve disputes between
employees and management?
A. The sender should ensure that the receiver
understands the message by allowing only one-way A. The company's image may be improved.
communication.
B. Public opinion may pressure the parties to resolve
B. The receiver should consider how the sender the dispute.
decodes the message.
C. Communication between employees and
C. The sender alone should minimize noise in the flow management is improved.
of communication.
D. The need for support from a union is increased.
D. The receiver should try to understand the sender's
perspective.
[175] Source: Publisher
The benefits of electronic communication for a company
[171] Source: Publisher and its employees include

Which of the following is a benefit of the communication A. More efficient use of resources to increase
link represented by negotiating employee-supervisor profitability.
differences?
B. Creation of redundant tasks that allow employees
A. Each employee perceives that management values to develop expertise.
his/her accomplishments.
C. Constant development of new procedures.
B. A written communication link between the
employees and management is provided. D. Elimination of simulation models.

C. Each employee is shown that management values


his/her rights and needs. [176] Source: Publisher
When implementing an electronic communication system,
D. The need for an informal grapevine is reduced. the organization should

A. Develop project teams or task forces made up


[172] Source: Publisher entirely of specialists.
Which of the following is a benefit of a holiday party as a
communication link between employees and management? B. Restrict the use of systems manuals and
designated user support to members of the MIS
A. Employees are given the opportunity to discuss department.
issues with management in an informal setting.
C. Create an MIS steering committee to hold
B. Employees become more aware of the confidential discussions about any systems
organizations' objectives. development.

C. Each employee may discuss concerns without fear D. Communicate information about the system to all
of repercussions from management. employees.

D. Outstanding employee performance is


communicated in a formal, public atmosphere. [177] Source: CMA 0692 1-16
All of the following statements about communication are
correct except
[173] Source: Publisher
Which of the following is the most likely effect on other A. Written communication inhibits feedback.
employees when the firing of one employee is not fully
communicated? B. Managers spend more of their workday involved
in oral communication than written communication.
A. Employees will believe that their jobs are more
secure. C. Written communication provides a permanent
record.
B. Employees are discouraged from unifying because
of their fear of repercussions from management. D. Written communication is usually better when the
message is nonroutine and personal.
C. Employees' insecurity is decreased.
[178] Source: CMA 0692 1-21 Telecommuting, working away from the office and
In which one of the following statements is the use of communicating via electronic media, has become more
accounting jargon an impediment to communication widespread as advances in communication devices have
between accountants and nonaccounting professionals? made this more practical. All of the following are problems
that are beginning to be associated with employees using
A. "Labor standards tell us how much time workers telecommuting except that the telecommuting employees
should take to assemble a computer."
A. Fall behind in their fields of specialization.
B. "I am trying to decide whether the million dollar
disbursement should be expensed or capitalized." B. Lack strong working relationships.

C. "In carrying out variance analysis, I am seeking the C. Experience a loss of career opportunities.
possible reasons why the net income shown in the
financial statements for the year 1990 is 20% less D. Lack sufficient communication skills.
than expected."

D. "An accounting department in a factory is [183] Source: CMA 1293 1-13


classified as a service because it allows the Non-verbal communication is a powerful but
departments actually involved in the making of little-understood form of communication in organizations.
products freedom from bookkeeping chores." All of the following represent forms of non-verbal
communication except

[179] Source: CMA 1292 1-26 A. The tone of voice a person selects.
The three managerial roles have been defined as
interpersonal, informational, and decisional, all of which B. The positioning of furniture in a person's office.
require various forms of communication. The most likely
form of communication for a manager fulfilling the C. Prolonged eye contact.
informational role would be
D. A facsimile message from a business associate.
A. Resolving conflict between two subordinates.

B. Encouraging employees to improve productivity. [184] Source: CMA 1293 1-14


Which one of the following statements regarding
C. Attending a ribbon-cutting ceremony for the communication along the grapevine in an organization is
opening of a new plant. incorrect?

D. Scanning industry reports to stay abreast of A. By maintaining open channels of communication,


developments. managers can minimize the damage the grapevine can
do in an organization.

[180] Source: CMA 1292 1-28 B. Attempts to totally eliminate the grapevine are
Noise may disrupt communication during transmission. All unlikely to succeed.
of the following are examples of noise except
C. The grapevine communication network can exist in
A. Selective perception. several patterns.

B. Static on a telephone line. D. The grapevine is rumor-mongering, and managers


should not use it.
C. A letter lost because it was interspersed with junk
mail.
[185] Source: CMA 1293 1-15
D. A participant in a conversation being called away Which one of the following statements about human
for a meeting. perception and its role in the communication process is
incorrect?

[181] Source: CMA 1292 1-29 A. Perception is used to interpret information from
The biggest single drawback of written communication, the environment.
when compared with oral communication, is that it
B. Perception can distort communication only at the
A. Is less time consuming. end of the communication process.

B. Is relatively difficult. C. Selective perception refers to the tendency to


remember what we prefer and to forget what we are
uncomfortable with.
C. Provides a permanent record.
D. Stereotyping is a form of perceptual organization.
D. Inhibits feedback.

[186] Source: CMA 1293 1-16


[182] Source: CMA 1292 1-30 The basic purposes of an organization's internal
communications network include all of the following except ウ
レ--------------+-----------------ソ
A. Obtaining a common focus among employees. ウ ウ ウ
VP Sales Controller VP Manufacturing
B. Informing potential investors about company ウ

operations and financial results.
Production
Supervisor
C. Aiding high-quality decision making. ウ

D. Integrating the efforts of specialists. Foreman,
Grinding Dept.

[187] Source: CMA 1293 1-17 A. Foreman only.


The marketing manager of Oakleaf Corporation wrote an
interdepartmental memorandum to the budget manager B. Foreman, supervisor, and vice president.
concerning corrections to the sales forecast for the coming
year. This is an example of C. Foreman, supervisor, vice president, and
president.
A. Downward communication.
D. Foreman, supervisor, vice president, controller,
B. Upward communication. and president.

C. Informal communication.
[191] Source: Publisher
D. Horizontal communication. The format for internal reports in a responsibility accounting
system is prescribed by

[188] Source: CMA 1293 1-18 A. Generally accepted accounting principles.


At Marshall Inc., accounting clerks log incoming invoices,
and the controller determines the amount and timing of B. The Financial Accounting Standards Board.
invoice payments. For this type of situation, the most
efficient and accurate communication network is the C. The American Institute of Certified Public
Accountants.
A. All-channel network.
D. Management.
B. Circle network.

C. Chain network. [192] Source: Publisher


Rational decision making is a multi-step process. In which
D. Wheel network. stage of this process will effective communication to
persons affected by the decision be most important?

[189] Source: CMA 1291 3-10 A. Evaluating possible solutions.


A segment of an organization is referred to as a service
center if it has B. Defining the problem.

A. Responsibility for developing markets and selling C. Following up.


the output of the organization.
D. Identifying possible solutions.
B. Responsibility for combining the raw materials,
direct labor, and other factors of production into a
[193] Source: Publisher
final output. In which of the following cases is noise not present?

C. Authority to make decisions affecting the major A. The sender receives a telephone call while writing
determinants of profit including the power to choose a memorandum.
its markets and sources of supply.
B. The sender and receiver's telephone conversation
D. Authority to provide specialized support to other is interrupted by another caller.
units within the organization.
C. The receiver is called to an important meeting
while reading the sender's memorandum.
[190] Source: CIA 0588 IV-12
A company, organized as shown below, uses the D. The receiver listens to what (s)he wants to hear in
responsibility accounting concept. Who would receive the telephone conversation with the sender.
copies of the detailed expenditure reports for the grinding
department?
[194] Source: Publisher
Grinding Department Reporting Structure Which of the following is a benefit of implementing an
President electronic communication system?
A. Audience.
A. Relatively low capital expenditures are required.
B. Environment.
B. The company's comparative advantage over
smaller firms that cannot afford such a system will C. Method.
increase.
D. Interpersonal problems.
C. There is little need for additional resource
allocation to facilitate implementation of the system.
[198] Source: CIA 1190 III-20
D. The sophistication of electronic communication The list of modern communication systems extends well
systems eliminates the need for backup files and data beyond the telephone and postal service. These new
recovery systems. systems can be distinguished by the features or capabilities
they provide. Thus, features such as Answer, Edit,
Forward, Send, Read, and Print would indicate a system
[195] Source: Publisher called
Which of the following is a frequent effect on employees of
implementing an electronic communication system? A. Electronic mail.

A. Hardware and software development increase B. Voice store-and-forward.


employees' personal comfort.
C. Desktop publishing.
B. Communication between employees and the
management information systems (MIS) department D. Digital communications.
is improved because of employee participation in the
design of the system.
[199] Source: CIA 0594 II-EX9
C. Employees lack an understanding of what the Tolerating silence, asking open ended questions, and
system can do and how it will assist them. paraphrasing are three aids to more effective

D. Employees are willing and eager to change to the A. Meetings.


new system.
B. Listening.

C. Interviews.
[196] Source: CIA 1192 III-19
To explain a preliminary audit finding to the manager of D. Feedback.
personnel, an auditor comments as follows: "We believe
that our statistical sample documents a breakdown in the
review of updates to the personnel master file by the [200] Source: CIA 0594 II-46
immediate supervisor. The expected error rate of our Accountants must be effective listeners, especially when
attribute sample was 1% but the actual error rate was
between 4% and 6%. The result was a rejection of the asking complex questions. To improve their listening,
sample hypothesis. We therefore recommend that the accountants should take care to do all the following except:
importance of complying strictly with existing review
procedures be reemphasized." The manager of personnel A. Stop talking. It is very difficult to listen and talk at
disputes the validity of the finding. The communications the same time.
barrier presented above is best described as
B. Be patient. Allow the speaker ample time to
A. Poor environment. respond.

B. Technical jargon. C. Avoid all questions until the speaker has


concluded.
C. Inappropriate communication method.
D. Put the speaker at ease. A nervous speaker will
D. Poor timing. be difficult to understand.

[197] Source: CIA 1192 III-20 [201] Source: CIA 1192 III-18
To motivate an auditor-in-charge to improve the work of Which part of the communication process is described by
the staff auditors assigned to an audit, an internal auditing terms such as selectivity, organization, and interpretation?
supervisor sent the following memo: "I want you to initial
and date every working paper. You are to check all work A. Environmental.
and get these people to conform to department
procedures." At the end of the audit, the supervisor was B. Objective.
shocked to discover that the audit budget was grossly
exceeded, working paper quality was poor, and audit focus C. Noise.
was upon low risk and unproductive areas. The failure in
the communication chain was most likely caused by D. Perception.
[202] Source: CIA 1191 III-13 A. Meeting of all employees.
Perception is a vital part of communication and is made up
of which three subprocesses? B. Front-page message in the monthly company
newsletter.
A. Selectivity, organization, and interpretation.
C. Memorandum to each employee.
B. Sending, receiving, and feedback.
D. Board meeting.
C. Listening, writing, and speaking.

D. Sending, listening, and psychosocial. [207] Source: CIA 0578 III-12


Which of the following is an example of upward
communication?
[203] Source: CIA 1191 III-12
A purchasing agent placed a rush telephone order with a A. Management's notices on bulletin boards.
supplier. The clerk in the supplier's office repeated the
order specifications back to the purchasing agent. No B. Grievance actions.
written confirmations were exchanged. When the shipment
arrived, it was late and of the wrong quantity. However, the C. Informational inserts in pay envelopes.
purchasing agent was unable to prove that the shipment
was unsatisfactory. What link of the communication chain D. Personnel policy manuals.
has failed in this scenario?

A. Encoding. [208] Source: CIA 0579 III-26


Which of the following is the best indicator of the
B. Decoding. effectiveness of a communication on a receiver?

C. Medium. A. Understanding of message received.

D. Feedback. B. Clarity of message.

C. Change in receiver's attitude.


[204] Source: CIA 0593 III-29
A manager found that instructions given to a subordinate D. Change in receiver's behavior.
were not followed. A review of the cause of the failure
revealed that the manager was interrupted by several
telephone calls while issuing the instructions. In terms of [209] Source: CIA 0580 III-11
problems in the communications chain, the interruptions are Which of the following is least appropriate with regard to
management's approach to informal group or grapevine
A. Noise. communication? Management should

B. Nonverbal feedback. A. Use it to supplement communication channels of


the formal organization.
C. Examples of closure.
B. Try to suppress it as a possible source of
D. Examples of perceptual selectivity. conflicting information.

C. Take advantage of it as a device to correct


[205] Source: CIA 0594 II-26 misinformation.
"But, I mailed the order 4 weeks ago, giving the supplier
plenty of time," said the parts manager when asked why a D. Make use of it as a means of transmitting
critical part was not available. The most likely reason for information not appropriate for formal communication
this failed communication between the parts manager and channels.
the supplier was

A. Lack of feedback. [210] Source: CIA 0580 III-19


Effective communication is most likely to take place when
B. Confusing language.
A. The sender and receiver share similar frames of
C. Inappropriate medium. reference.

D. Perceptual selectivity. B. The message is stated in general rather than


specific terms.

[206] Source: CIA 0591 III-17 C. The message is delivered as quickly as possible.
The rumor is spreading that a company's key contract has
been lost and 5,000 people will be laid off. Top D. The sender ignores any underlying assumptions.
management knows this is completely untrue and should
strongly deny the rumor in a
[211] Source: CIA 0582 III-11 B. Is often imprecise.
In some organizations, first-line supervisors withhold or
alter unfavorable information that they do not want higher C. Always conveys a more truthful response.
management to know. This selective withholding of
information is widely known as D. Always conveys less information than verbal
communication.
A. Selective reception.

B. Filtering. [216] Source: CIA 0594 II-12


When researching a question about how to account for a
C. Regulating information flow. particular type of transaction, which of the following is most
important?
D. Perceptual defense.
A. Using computer databases to find all relevant
sources.
[212] Source: CIA 0582 III-24
Studies of managerial communications have indicated that B. Providing documentation of the reference sources.

A. Most managers are excellent communicators. C. Presenting only those facts that support the
conclusion.
B. Managers spend most of their time
communicating. D. Presenting all facts that might bear on the issue.

C. Written communication takes more of a manager's


time than oral communication. [217] Source: CIA 0594 II-13
An auditor is considering developing a questionnaire to
D. Most effective communicators will be good research employee attitudes toward control procedures.
managers. Which of the following represents a criterion that should not
be considered in designing the questionnaire?

[213] Source: CIA 1182 III-25 A. Questions must be worded to ensure a valid
Communication plays an important role in the successful interpretation by the respondents.
operation of all organizations. Which of the following
statements concerning organizational communications is B. Questions must be reliably worded so that they
false? measure what was intended to be measured.

A. Communication involves at least two people: a C. Questions should be worded such that a "No"
sender and a receiver. answer indicates a problem.

B. Communication is what the sender says, not what D. Many types of questions can be used.
the receiver understands.

C. Every act of communication influences the [218] Source: CIA 0594 II-16
organization in some way. Checklists used to assess audit risk have been criticized for
all of the following reasons except
D. Management spends the majority of its time
communicating with other members of the A. Providing a false sense of security that all relevant
organization. factors are addressed.

B. Inappropriately implying equal weight to each item


[214] Source: CIA 0594 II-1 on the checklist.
Which of the following is unlikely to cause changes in
attitudes? C. Decreasing the uniformity of data acquisition.

A. Make sure that the message is credible. D. Being incapable of translating the experience or
sound reasoning intended to be captured by each
B. Present many different issues in as short a time as item on the checklist.
possible.

C. Shape the argument to the listener. [219] Source: CIA 0594 II-43
In documenting the procedures used by several interacting
D. Focus the presentation on its ultimate objective. departments the internal auditor will most likely use

A. A horizontal (or systems) flowchart.


[215] Source: CIA 0594 II-45
When evaluating communication, the accountant should be B. A vertical flowchart.
aware that nonverbal communication
C. A Gantt chart.
A. Is independent of a person's cultural background.
D. An internal control questionnaire.
D. Departmental needs and goals.

[220] Source: CIA 0594 II-11


All of the following are true about business memos except [225] Source: Publisher
All of the following statements about communication are
A. They are usually short. false except

B. They should be written in active voice. A. Managers spend more of their workday involved
in written communication than oral communication.
C. The tone should be casual to get the reader's
attention. B. Written communication encourages feedback.

D. Readers should grasp their meaning easily. C. Breakthroughs in electronic technology will blur
the distinctions between written and oral
communications.
[221] Source: CIA 0594 II-EX10
A long-term supplier has asked your company to negotiate D. The grapevine rarely provides accurate
some increased purchases and faster payment. The supplier information.
is having some financial difficulty, but your firm has been
quite profitable lately, having experienced growing sales.
Your firm's best negotiation strategy is [226] Source: Publisher
Managerial roles have been defined as interpersonal,
A. Collaboration. informational, and decisional. Each role requires various
forms of communication. The most likely form of
B. Competition. communication for a manager fulfilling the decisional role
would be
C. Subordination.
A. Encouraging employees to improve productivity.
D. Avoidance.
B. Resolving conflict between two subordinates.

[222] Source: CIA 0594 II-EX11 C. Scanning industry reports to stay abreast of
Two internal auditors have been assigned projects of equal developments.
priority and the same due date. Unfortunately, support
services are limited. The auditors have been directed to D. Attending a ribbon-cutting ceremony for the
negotiate between themselves for the available services. opening of a new plant.
This type of negotiation is called

A. Distributive. [227] Source: Publisher


Which of the following perceptual errors is the
B. Integrative. extrapolation of a judgment about one matter to other,
possibly unrelated, issues?
C. Attitudinal structuring.
A. Projection.
D. Intraorganizational.
B. The halo effect.

[223] Source: Publisher C. Expectancy.


The biggest advantage of oral communication over written
communication is that it D. Stereotyping.

A. Tends to be more accurate.


[228] Source: Publisher
B. Promotes feedback. Which of the following terms does not apply to noise in
communication?
C. Is less time-consuming.
A. Encoding.
D. Does not provide a permanent record.
B. Sending.

[224] Source: Publisher C. Interpretation.


Organizational characteristics that may be barriers to
effective communications include all of the following except D. Decoding.

A. Organizational status differences.


[229] Source: Publisher
B. Lack of formal channels. Which of the following does not describe perception?

C. Listening problems. A. Selectivity.


B. Organization. C. Informal communication and the grapevine,
respectively.
C. Objective.
D. Downward communication and upward
D. Interpretation. communication, respectively.

[230] Source: Publisher [234] Source: CMA 0695 1-27


Which of the following statements is false according to Which one of the following is not an example of formal
communications theory? internal communication?

A. The process has three elements: sender, medium, A. Input for the yearly budget provided by the
receiver. Purchasing Department to the director of budgeting.

B. Two-directional communication is usually most B. Environmental impact statements.


effective.
C. Expense account reports.
C. Communication is the message actually received,
not what is transmitted. D. Safety bulletins.

D. Informal communications are used by effective


managers. [235] Source: CMA 0695 1-28
Which one of the following statements about the nature of
communication is correct?
[231] Source: Publisher
Which of the following is an example of a badly encoded A. Communication occurs only when we deliberately
message? and consciously decide to communicate.

A. Inattention or disinterest in the message. B. Accounting terms such as expense, revenue, net
income, and variance mean the same to all users.
B. Inconsistency between verbal and nonverbal
messages. C. Because managerial accounting reports classify
and summarize vast amounts of data, they do not add
C. Gesturing to someone who cannot see the gesture. to the information overload within an organization.

D. Sender's dislike of receiver. D. Communication between departments is affected


by the level of functional specialization within
departments.
[232] Source: CMA 0695 1-25
Nancy Ashburn is the cost accounting manager for Imperial
Aluminum's extrusion plant. Her job involves coordinating [236] Source: CMA 0695 1-29
and summarizing monthly production department variance Which one of the following is not an example of formal
reports that are prepared by various staff accountants. She communication within an organization?
then provides the relevant variances to her colleagues, the
production departmental managers. The accounting reports A. The grapevine.
communicated by Ashburn are an example of
B. Variance analysis.
A. Downward communication.
C. The performance evaluation system.
B. Hierarchical communication.
D. A budget.
C. Informal communication.

D. Horizontal communication. [237] Source: CMA 0695 1-30


Which one of the following will not help overcome
communication problems between the Accounting
[233] Source: CMA 0695 1-26 Department and other departments?
Arthur Bell is the superintendent of a production
department. Every month, Bell sends copies of his A. Interdepartmental task forces.
department's production variance reports to his supervisory
staff asking them to explain variances that exceed certain B. Cross training and job rotation.
prescribed limits. Bell's communication of variances to his
supervisory staff and their explanation to him of the C. Organization-wide social events.
variances are examples of
D. A performance appraisal prepared by your
A. Horizontal communication and upward immediate supervisor.
communication, respectively.

B. Formal and informal communication, respectively. [238] Source: CIA 0594 II-48
Written communication improves with conscious use of the
active voice. Which of the following statements does not A. Refuting some of their more vulnerable arguments
use the active voice? in a subtle fashion.

A. The inventory count will be conducted by the B. Taking a representative sample of their arguments
inventory clerk. and agreeing that the strong ones are strong and
refuting the weaker ones.
B. Make the appointment for 9:00 a.m. next
Thursday. C. Attacking the character of the people who hold
those opinions.
C. The division controller reports directly to the
corporate controller. D. Refuting arguments at the end of the message to
show that they are the least important.
D. Given the above, we cannot recommend approval
at this time.
[Fact Pattern #2]
A multinational firm was attempting to buy a controlling
[239] Source: CIA 0594 II-49 interest in a medium size ($10 million annual sales) Brazilian
Effective communication avoids overuse of concealed metal working firm. Its negotiator in Brazil sent the
verbs. Which of the following items does not contain a following telegram: "They won't deal unless 51%
concealed verb? ownership." The executive committee of the multinational
firm, not wanting a minority interest, then canceled the deal.
A. The A Division is planning to make an acquisition Upon returning to the multinational firm, the negotiator
of a part supplier. pointed out that the Brazilian firm wanted to sell no more
than 51% ownership, so they could retain at least 49%.
B. Elimination of overtime was effected by the Thus, the deal could have been made.
security department.
[243] Source: CIA 0592 III-19
C. The electronic data processing consulting group (Refers to Fact Pattern #2)
will provide assistance to the internal auditors. The telegram received by the executive committee was
faulty. In terms of the links in the communications process,
D. Unless payment is received by the first of next the error occurred because of
month, we will be forced to cancel your credit line.
A. Noise in the communication chain.

B. The sender's perception.


[240] Source: CIA 1196 II-19
The behavioral science literature identifies diffusion as an C. Message encoding.
effective approach to resolving conflict. An auditor
effectively using diffusion in working with a confrontational D. The choice of transmission medium.
auditee would

A. Set aside critical issues temporarily and try to [244] Source: CIA 0592 III-20
reach agreement on less controversial issues first. (Refers to Fact Pattern #2)
The faulty telegram led to a communications error by the
B. Emphasize differences between the parties. executive committee of the multinational firm. The error
was in
C. Avoid the conflict situation.
A. Decoding of the message.
D. Identify the sources of conflict and address them
directly. B. Choice of transmission medium.

C. Understanding of the message.


[241] Source: CIA 1194 II-25
All of the following are true about good writing styles used D. Response to the message.
in business memos except

A. Draws positive attention to the writing style. [245] Source: CIA 1196 II-31
A company is rumored to be considering downsizing.
B. Treats all receivers with respect. Because a manager stops the use of all temporary
employees, the staff concludes that some jobs will be lost.
C. Suits the method of presentation and delivery. Which of the following is true about the manager's
communication about job losses?
D. Develops ideas without overstatement.
A. The staff decoded the formal communication sent
by the manager correctly.
[242] Source: CIA 1195 II-39
In a persuasive message organized around rational B. The manager properly encoded the idea in a
arguments, a persuasive writer deals with the opposition's message.
arguments by
C. The lack of a formal message had a negative
impact on staff. appropriate source.

D. The channel through which the message was sent


was appropriate. [249] Source: CIA 1196 II-33
During an internal audit, the auditor experienced difficulty
obtaining required information from a specific employee.
[246] Source: CIA 0593 III-29 When this situation continued for one week, the auditor
A manager found that instructions given to a subordinate requested a private meeting with the employee for the
were not followed. A review of the cause of the failure purpose of identifying the problem and resolving the
revealed that the manager was interrupted by several difficulty through open discussion. Which conflict
telephone calls while issuing the instructions. In terms of management technique was the auditor applying?
problems in the communications chain, the interruptions are
A. Problem solving.
A. Noise.
B. Expansion of resources.
B. Nonverbal feedback.
C. Authoritative command.
C. Semantics.
D. Altering the human variable.
D. Closure.

[250] Source: CIA 0596 II-3


[Fact Pattern #3] In a written communication, which of the following is least
The supervisor of purchasing reviewed a memorandum likely to cause changes in attitudes?
prepared for a buyer in the department. The memo read,
"Effective September 30, the corporation has determined A. Make sure that the message is credible.
that your functions will be absorbed into our parent
company's small-unit purchasing function. This will reduce B. Present as many different issues in as short a time
operating costs, improve communications, and facilitate as possible.
production engineering changes. You will be provided with
outplacement support." "That should cover the situation," C. Appeal to the individual's emotions.
thought the supervisor. "It's too bad that I am leaving on
vacation before the buyer returns from vacation, but this D. Shape the argument to the listener.
memo will give the buyer the general idea."

[247] Source: CIA 0593 III-27 [251] Source: CIA 0594 II-EX9
(Refers to Fact Pattern #3) Tolerating silence, asking open ended questions, and
What link in the communications chain is defective? paraphrasing are three aids to more effective

A. The meaning of the message would be unclear to A. Meetings.


the buyer.
B. Listening.
B. The supervisor chose the wrong channel for the
communication. C. Interviews.

C. The supervisor should not be the source of this D. Feedback.


type of communication.

D. The supervisor did not account for the noise in the [252] Source: CIA 1196 II-20
communication chain. Some studies show that managers spend 60 to 70% of
their time communicating and that nearly 60% of that time is
spent listening. Listening effectiveness is best increased by
[248] Source: CIA 0593 III-28
(Refers to Fact Pattern #3) A. Resisting both internal and external distractions.
Select an additional deficiency in the communication chain.
B. Waiting to review key concepts until the speaker
A. The receiver of the communication was is through talking.
inappropriate; the personnel department should have
received the memorandum and then informed the C. Tuning out messages that do not seem to fit the
buyer. meeting purpose.

B. The message is improperly encoded because the D. Factoring in biases to evaluate the information
buyer does not know why the termination is to be being given.
completed.

C. Because the supervisor and buyer have conflicting


vacation schedules, no possibility exists for feedback. [253] Source: CIA 0596 II-15
A supportive behavior that a listener, such as an auditor or
D. The supervisor should not be the sender of the a supervisor, can use to encourage a speaker would be to
memorandum; the personnel department is the
A. Look away from the speaker to avoid any IV. Unconscious actions of the speaker while speaking
intimidation. A. III only.

B. Interject a similar incident or experience. B. IV only.

C. Stop other activity or work while the person is C. I and II only.


talking.
D. I, II, and IV.
D. Not respond verbally until the speaker stops
talking.
[258] Source: CIA 0596 III-17
There are many types of third-party negotiations available
[254] Source: CIA 0596 II-19 to parties facing disagreement. If the goal is to be certain
An advisable strategy for a participant in a meeting of the that settlement is reached, a negotiator with authority to
employees would be to make a decision should be selected. The best negotiator to
select, given this goal, would be a(n)
A. Read the agenda and supporting materials for the
meeting during the early part of the meeting to A. Mediator.
prepare for later discussion.
B. Arbitrator.
B. Present strong opinions on one side of a proposal
right away. C. Consultant.

C. Present views as trial balloons that can be D. Conciliator.


researched later.

D. Consider the opinions and information needs of [259] Source: CIA 1196 III-11
other participants before speaking. A construction manager is using a distributive-bargaining
approach in negotiating the price of lumber with a supplier.
The construction manager will
[255] Source: CIA 0594 II-EX10
A long-term supplier has asked your company to negotiate A. Concede to the supplier's asking price in order to
some increased purchases and faster payment. The supplier maintain a positive working relationship.
is having some financial difficulty, but your firm has been
quite profitable lately, having experienced growing sales. B. Hire a mediator to negotiate the deal on behalf of
Your firm's best negotiation strategy is the manager.

A. Collaboration. C. Attempt to get agreement on a price within the


settlement range (that is, within both the manager's
B. Competition. and supplier's aspiration ranges).

C. Subordination. D. State the resistance point (that is, the highest price
acceptable) and ask the supplier to concede.
D. Avoidance.

[260] Source: CIA 1196 III-12


[256] Source: CIA 1194 II-26 Which of the following would be the best approach for
While conducting fieldwork, a strong conflict arises negotiating the purchase of a large number of
between two of your subordinates regarding possible microcomputers, assuming that both parties follow the
scope expansion. You draw their attention to their shared same approach?
views, downplaying the issues of contention. This technique
for resolving conflict is called A. Review previous demands, concessions, and
settlements (precedents).
A. Superordinate goals.
B. Attempt to get personal information about the
B. Smoothing. opposing negotiators.

C. Problem solving. C. Enter without preconceived ideas about what


should be accomplished.
D. Compromise.
D. Ask as few questions as possible during
negotiations.
[257] Source: CIA 0596 II-18

Nonverbal communication consists of messages conveyed [261] Source: CIA 1196 III-9
by In a situation involving a disagreement between two parties,
when one party's interests are more important than the
I. The physical distance between the sender and the receiver other's (for example, a customer believes one product is
II. The facial expressions used when speaking most suitable while the seller disagrees, yet the seller's
III. Electronic means of communication such as e-mail primary goal is to keep the customer satisfied), the best
conflict-resolution strategy is [265] Source: CIA 1195 III-8
Two managers have been arguing about the distribution of
A. Accommodating. money for capital investment projects affecting their
respective production units. All of the projects are
B. Compromising. worthwhile and significantly exceed the organization's
required rate of return. The approach that would create a
C. Competing. win-win solution for the managers under these
circumstances would be to
D. Challenging.
A. Smooth the differences of the two managers by
emphasizing their common interests.
[262] Source: CIA 0595 III-12
To market effectively the internal auditing function to
management, auditors must recognize that their roles may B. Alter the attitudes and behaviors of the managers
result in varying degrees of conflict. Conflict triggers must so that agreement can be reached.
be understood and managed so that a dysfunctional
situation does not develop. Select the answer that is not a C. Force the managers to compromise by asking
conflict trigger. each of them to give up something.

A. Communication breakdowns. D. Expand the resources available so that both


manager's projects can be funded.
B. Superordinate goals.

C. Personality clashes. [266] Source: CIA 1195 II-16


Which one of the following techniques is not generally
D. Status differentials. recognized as an effective conflict resolution technique for
management to use in a dispute between employees?

[263] Source: CIA 0595 III-4 A. Accommodation - management encourages a high


Many people think that conflict is inherently detrimental to degree of cooperation and a low degree of
an organization. However, studies and actual case assertiveness.
experience demonstrate that some conflict is desirable if an
organization is to thrive. Identify the technique that B. Competition - management encourages the parties
management could use to stimulate conflict. to seek their own interests, regardless of the effect on
each other.
A. Authoritative command.
C. Reorganization - management transfers one of the
B. Restructuring the organization. disputants to another department.

C. Expansion of resources. D. Compromise - management persuades each party


to make concessions.
D. Creation of superordinate goals.

[267] Source: CIA 0594 III-81


[264] Source: CIA 0596 III-10 Which of the following would not be considered a conflict
Two managers have been informed that their units will be trigger?
relocated to a new site. The units are to share space at the
new office location. The managers have been arguing for A. Ambiguous jurisdictions.
several weeks over the allocation of space and the location
of offices. This disagreement is threatening the relocation B. Competition for scarce resources.
schedule and disrupting other projects. The managers'
supervisor has now become involved in the conflict and C. Status differential.
must try to minimize the potential for hurt feelings while
resolving the problem quickly. Identify the supervisor's best D. Superordinate goals.
approach for this situation.

A. Sit down with the managers and determine a [268] Source: CIA 0596 II-22
solution. As a conflict resolution strategy, optimizing (or a win-win
strategy) is most appropriate when
B. Design a floor plan and tell the managers who
occupies what space. A. The benefits being contested cannot be changed.

C. Remind the managers that the company needs B. The relationship between the parties is likely to
their cooperation in this effort so that costs can be continue.
reduced.
C. People are deeply committed to established habits
D. Tell the managers not to worry and that problems and patterns.
like this have a way of working themselves out.
D. Time is scarce and the manager's patience is
wearing thin.
キ Audit programs were pre-set and printed in the audit manual
to
[269] Source: CIA 0596 III-8 facilitate staff productivity.
One division of a large manufacturing company has キ Audit staff could expect the basic corporate salary increase
traditionally performed much better than any of the other percentage if performance met explicitly stated departmental
divisions. The management team of this division has risen rules
through the ranks together and exhibits no signs of conflict. about attendance, meeting scheduled completion dates for
Recently, earnings of the division have begun to decline, assignments,
and market share has eroded. Senior management of the and submitting properly formatted working papers.
parent company has asked the director of internal audit キ The director's predecessor had hired only recent college
whether the introduction of conflict by bringing in outside graduates who
managers might help resolve the deteriorating situation. The met entry-level qualification requirements. Only the audit
most appropriate response would be that supervisor,
who had worked with the previous director for over 20 years,
A. Conflict is dysfunctional and should not be risked had been
under these circumstances. with the company more than 3 years.
キ Audits were classified and assigned to one of three groups in
B. All conflict can be beneficially controlled and the
should be encouraged in this situation. department: financial, operations, or EDP.
キ The previous director had routinely denied all staff requests
C. The management team has been together for a to enroll
long time and should be allowed to work through its in postgraduate degree programs.
problems.
[271] Source: CIA 0596 II-24
D. Varying the management team could introduce (Refers to Fact Pattern #4)
new ideas and be beneficial to the division, and some Which of the following situations calls for conflict-resolution
conflict is not a problem. principles and techniques to be applied in the audit
department?

[270] Source: CIA 0595 II-31 A. Requests to attend postgraduate programs.


Upon completing an audit of a major operation of the
company, the auditor is certain that a proposed B. Performance criteria.
recommendation should be made in the audit report.
However, the auditor also understands that the C. Division of staff into three specialty groups.
recommendation will result in conflict between the auditee
department and the accounting department. The D. Preassigned audit programs.
organization is not bureaucratic and encourages the
development of informal relationships across departments.
Which of the following statements is correct regarding the [272] Source: CIA 0596 II-25
nature of conflict in organizations? (Refers to Fact Pattern #4)
Which of the following are examples of the traditional view
A. Conflict is more likely to be functional in a of motivation that the new director needs to overcome?
bureaucratic organization than in a less formal
(organic-type) organization. A. Use of pre-set and printed audit programs.

B. Conflict reduces the likelihood that an acceptable B. Emphasis on rules for attendance and timely
solution can be implemented in highly structured completion of assignments.
organizations; thus the auditor should consider
revising the recommendation in order to avoid C. Use of compensation rewards based on following
conflict. stated rules.

C. Conflict should be viewed as a healthy way to D. All of the answers are examples of the traditional
facilitate growth in an organization; thus the auditor view of motivation that the new director needs to
should accept conflict that may result from normal overcome.
audit recommendations.

D. Conflict is healthy unless it clearly points out [273] Source: CIA 0596 II-26
differences in the goals and objectives of the (Refers to Fact Pattern #4)
organization's operating units. Which of the following does not describe a structural
condition that creates opportunities for conflict to arise?

[Fact Pattern #4] A. Communication within the department.


A newly appointed director of internal auditing perceived
the need to address, among other things, motivational B. Tenure and age of most department members.
techniques for employees in the department. In a group
session, the new director identified the following situations. C. Written and enforced compensation criteria.

キ Communication within the department needed the most D. Size and grouping of staff into one of three
attention by specializations.
management.
[274] Source: CIA 1196 II-23 [278] Source: Publisher
When an internal auditor encounters active opposition, as A financial manager/management accountant discovers a
when auditees remain unconvinced of the auditor's problem that could mislead users of the firm's financial data
reasonably presented point of view, the most effective way and has informed his/her immediate superior. (S)he should
to gain consensus is to report the circumstances to the audit committee and/or the
board of directors only if
A. Refer the matter to the auditees' superior.
A. The immediate superior, who reports to the chief
B. Wait to discuss with auditees late in the day when executive officer, knows about the situation but
they may be more reasonable. refuses to correct it.

C. Rely on logic and explain the auditor's point again. B. The immediate superior assures the financial
manager/management accountant that the problem
D. Find a point of agreement by letting auditees will be resolved.
explain their position again.
C. The immediate superior reports the situation to
his/her superior.
[275] Source: Publisher
If a financial manager/management accountant has a D. The immediate superior, the firm's chief executive
problem in identifying unethical behavior or resolving an officer, knows about the situation but refuses to
ethical conflict, the first action (s)he should normally take is correct it.
to

A. Consult the board of directors. [279] Source: Publisher


In which situation is a financial manager/ management
B. Discuss the problem with his/her immediate accountant permitted to communicate confidential
superior. information to individuals or authorities outside the firm?

C. Notify the appropriate law enforcement agency. A. There is an ethical conflict and the board has
refused to take action.
D. Resign from the company.
B. Such communication is legally prescribed.

[276] Source: Publisher C. The financial manager/management accountant


If a financial manager/management accountant discovers knowingly communicates the information indirectly
unethical conduct in his/her organization and fails to act, through a subordinate.
(s)he will be in violation of which ethical standard(s)?
D. An officer at the financial manager/ management
A. "Actively or passively subvert the attainment of the accountant's bank has requested information on a
organization's legitimate and ethical objectives." transaction that could influence the firm's stock price.

B. "Communicate unfavorable as well as favorable


information." [280] Source: Publisher
The IMA Code of Ethics includes an integrity standard,
C. "Condone the commission of such acts by others which requires the financial manager/ management
within their organizations." accountant to

D. All of the answers are correct. A. Identify and make known anything that may hinder
his/her judgment or prevent satisfactory completion of
any duties.
[277] Source: Publisher
The IMA Code of Ethics requires a financial B. Report any relevant information that could
manager/management accountant to follow the established influence users of financial statements.
policies of the organization when faced with an ethical
conflict. If these policies do not resolve the conflict, the C. Disclose confidential information when authorized
financial manager/management accountant should by his/her firm or required under the law.

A. Consult the board of directors immediately. D. Refuse gifts from anyone.

B. Discuss the problem with the immediate superior if


(s)he is involved in the conflict.

C. Communicate the problem to authorities outside


the organization.

D. Contact the next higher managerial level if initial


presentation to the immediate superior does not
resolve the conflict.
is growing in importance for multiline, large-scale
CMA PART 1 G enterprises. It is often an outgrowth of functional
Organizational Structures, departmentation and permits extensive authority for a
division executive over a given product or product
Management line. Its advantages include better use of specialized
resources and skills, ease of coordination of the
And Communication activities for a given product, and simpler assignment
of profit responsibility. It is also compatible with a
Answers decentralization strategy and provides, through
product profit centers, a basis for allocating capital
more efficiently.
[1] Source: Publisher
Answer (D) is incorrect because it is used for a
Answer (A) is correct. Departmentation by function specific project or for R&D.
is the most widely used method and is found in almost
every enterprise at some level. The most common
departments are selling, production, and finance [4] Source: Publisher
(though other terms may be used). These often
extend upward in the organizational chart to the level Answer (A) is incorrect because departmentation by
below the chief executive. If persons within a territory does not require violation of the unity of
department have similar knowledge, skills, and command principle.
interests, they can specialize in the solution of
particular problems. Thus, problem solving becomes Answer (B) is incorrect because departmentation by
more efficient. function does not require violation of the unity of
command principle.
Answer (B) is incorrect because departmentation by
function facilitates communication and coordination Answer (C) is incorrect because departmentation by
within rather than between departments. customer or product does not require violation of the
unity of command principle.
Answer (C) is incorrect because the focus may tend
to be on departmental, not organizational, goals. Answer (D) is correct. A matrix organization consists
of a project team formed with people from various
Answer (D) is incorrect because territorial functional areas within the organization. These
departmentation may be preferable for these specialists report simultaneously to the project
companies. manager and the managers of their functional
departments. At the end of the project, the team is

[2] Source: CIA 0596 II-32 disbanded.

Answer (A) is incorrect because defining the


approach decreases both acceptance and [5] Source: Publisher
understanding of the assignment. The employee
should participate in the decision and be able to Answer (A) is incorrect because fear of personal and
discuss and clarify the assignment. social adjustments may lead to resistance.

Answer (B) is correct. The supervisor's expectations Answer (B) is correct. Resistance to change may be
should be clear. The employee should also be caused by fear of the personal adjustments that may
involved in determining how to reach the desired be required. Employees may have a genuine concern
outcome, thereby increasing both acceptance and about the usefulness of the change, perceive a lack of
understanding of the assignment. concern for workers' feelings, fear the outcome,
worry about downgrading of job status, and resent
Answer (C) is incorrect because this approach deviations from past procedures for implementing
almost guarantees that the employee will feel that change (especially if new procedures are less
(s)he failed to perform well. participative than the old). Social adjustments also
may be required that violate the behavioral norms of
Answer (D) is incorrect because the significant risk is informal groups or disrupt the social status quo within
that the employee will not define an outcome and an groups. Economic adjustments may involve potential
approach with which the supervisor agrees. economic loss or insecurity based on perceived
threats to jobs. In general, any perceived
deterioration in the work situation that is seen as a
[3] Source: Publisher threat to economic, social, and/or psychological
needs will produce resistance. The various
Answer (A) is incorrect because a profit center is an adjustments required are most likely to be resisted
organizational unit responsible for costs and revenues when imposed unilaterally by higher authority.
on an ongoing basis, not just for a one-time activity. However, employees who share in finding solutions
to the problems requiring change are less likely to
Answer (B) is incorrect because the profitability of a resist because they will have some responsibility for
single function is difficult to measure. the change.

Answer (C) is correct. Departmentation by product Answer (C) is incorrect because imposed change will
meet the greatest resistance. Answer (A) is incorrect because it is a characteristic
of an organic organization. This arrangement often
Answer (D) is incorrect because adequate notice and works best when the fixed asset base is relatively
effective communication help reduce resistance. small because the company must respond rapidly to
frequent environmental change.

[6] Source: Publisher Answer (B) is correct. Mechanistic organizations


have vertical structures with duties and authority
Answer (A) is incorrect because the mechanistic clearly defined by position in the hierarchy, a stringent
model emphasizes clear definition of duties and set of formal controls, and a centralized staff. Such
authority by position in the hierarchy (vertical organizations tend to emphasize efficiency and
specialization), numerous, well-documented policies, operate in a stable environment with well-established
procedures, and rules, a complex formal control technology. Large firms in basic industries (autos,
structure, and centralized staff. The strong centralized utilities) often adopt this structure.
control in this model is inconsistent with adhocracy.
Answer (C) is incorrect because it is a characteristic
Answer (B) is correct. The organic structure is at the of an organic organization. This arrangement often
opposite end of the design continuum from a works best when the fixed asset base is relatively
mechanistic organization. An organic structure has small because the company must respond rapidly to
groupings of similar persons and resources in subunits frequent environmental change.
(horizontal specialization), relatively few policies,
procedures, and rules, less stringent formal controls, Answer (D) is incorrect because it is a characteristic
decentralized staff, and matrix or divisional of an organic organization. This arrangement often
departmentation. works best when the fixed asset base is relatively
small because the company must respond rapidly to
Answer (C) is incorrect because the bureaucratic frequent environmental change.
model emphasizes clear definition of duties and
authority by position in the hierarchy (vertical
specialization), numerous, well-documented policies, [9] Source: Publisher
procedures, and rules, a complex formal control
structure, and centralized staff. The strong centralized Answer (A) is incorrect because it is characteristic of
control in this model is inconsistent with adhocracy. a mechanistic organization with a rigid vertical or
chain-of-command structure. This design may be best
Answer (D) is incorrect because the structural or when the jobs to be done require little creativity and
classical model emphasizes clear definition of duties personal fulfillment is correspondingly low, leadership
and authority by position in the hierarchy (vertical is directive rather than participative, and
specialization), numerous, well-documented policies,
procedures, and rules, a complex formal control communication is vertical rather than multi-directional.
structure, and centralized staff. The strong centralized
control in this model is inconsistent with adhocracy. Answer (B) is incorrect because it is characteristic of
a mechanistic organization with a rigid vertical or
chain-of-command structure. This design may be best
[7] Source: Publisher when the jobs to be done require little creativity and
personal fulfillment is correspondingly low, leadership
Answer (A) is correct. The traditional approach to is directive rather than participative, and
organizing the structure of roles to facilitate the communication is vertical rather than multi-directional.
achievement of goals emphasizes authority,
responsibility, tasks, hierarchy, etc. Because this Answer (C) is incorrect because it is characteristic of
approach tends to set up prescribed or dictated a mechanistic organization with a rigid vertical or
relationships, it has been called mechanistic. chain-of-command structure. This design may be best
when the jobs to be done require little creativity and
Answer (B) is incorrect because an organic personal fulfillment is correspondingly low, leadership
organization has few hierarchical levels, low reliance is directive rather than participative, and
on positional authority, and few dictated relationships communication is vertical rather than multi-directional.
among members.
Answer (D) is correct. The organic structure is best
Answer (C) is incorrect because the matrix form of suited to a company that emphasizes operational
organization is used for project management since it efficiency less than the flexibility needed to respond
temporarily combines product and functional quickly to changes in its environment and the
structure for the duration of the project. creativity to find solutions to new problems. Decision
making in this context tends to rely on input from
Answer (D) is incorrect because the contingency or many sources and requires multi-directional
modern organizational structure is based on the communication. Hence, a horizontal or flat structure
particular situation, including environmental influences, typified by project or matrix management will be
technical considerations, and the size and age of the appropriate.
organization.

[10] Source: Publisher


[8] Source: Publisher
Answer (A) is incorrect because bureaucratic
organizations are usually large and complex. extraordinary performance is present.

Answer (B) is incorrect because in a bureaucratic Answer (B) is incorrect because employees of a
organization, efficiency is most likely to be achieved bureaucracy do not participate in decision making.
through a high degree of task delegation.
Answer (C) is incorrect because employees are
Answer (C) is incorrect because a bureaucratic usually unsatisfied since they lack a sense of
organization is characterized by a hierarchy of ownership and belonging.
authority, which is typically adhered to when
communicating. Answer (D) is incorrect because each job's
description and tasks are invariable. The employee
Answer (D) is correct. Organizational structure is the has no input in the development of his/her function.
order and disposition of lines of responsibility. A
bureaucracy is an organizational structure in which
[14] Source: Publisher
tasks are specialized under a given set of rules and a
hierarchy of authority. Division of labor is the Answer (A) is incorrect because a line position is
separation of work loads into small segments to be directly responsible for achieving objectives.
performed by one or more people.
Answer (B) is correct. A line position is in the direct
chain of command and is responsible for
[11] Source: Publisher administering policy and making operating decisions.
People in line positions have direct responsibility for
Answer (A) is correct. Each employee's job achieving the organization's basic objectives and thus
description defines the related activities and have formal or legitimate authority.
procedures, which do not vary over time. The result
is certainty and consistency with regard to the Answer (C) is incorrect because a line position
routines and procedures of a job function. receives advice and assistance from staff positions in
the organization.
Answer (B) is incorrect because open positions are
usually filled by employees of the firm through Answer (D) is incorrect because a line position is
promotion. directly related to the organization's product(s) and/or
service(s).
Answer (C) is incorrect because centralization is
typical of bureaucratic organizations.
[15] Source: Publisher
Answer (D) is incorrect because the impersonality of
bureaucracy reduces the human and social processes Answer (A) is correct. Line managers are directly
that promote individual employee growth. responsible for achieving the organization's
objectives, but staff managers are not directly
accountable. However, line managers may have no
[12] Source: Publisher authority to influence staff behavior when it is
inconsistent with the achievement of objectives.
Answer (A) is incorrect because the system of
checks and balances within a bureaucracy requires Answer (B) is incorrect because staff managers may
have functional authority that line managers perceive
elaborate paperwork that limits the action and as threatening.
reaction time of the organization.
Answer (C) is incorrect because staff managers give
Answer (B) is incorrect because a bureaucracy's line managers advice, not vice versa.
inflexibility promotes control but inhibits creativity.
Answer (D) is incorrect because line managers are
Answer (C) is correct. In a bureaucracy, tasks are likely to become reliant on staff expertise.
assigned through the division of labor. A set of
outlined procedures exists for each job. Because
these procedures are invariable, the tasks assigned [16] Source: Publisher
for each job become routine for the employee.
Answer (A) is incorrect because it is an advantage of
Answer (D) is incorrect because the work a flat organizational structure, which concentrates
atmosphere of a bureaucracy is impersonal, which decision-making power at one level.
limits individual employee growth.
Answer (B) is incorrect because it is an advantage of
a flat organizational structure, which concentrates
[13] Source: Publisher decision-making power at one level.

Answer (A) is correct. In a bureaucracy, standards Answer (C) is correct. The organizational structure
for evaluating job performance do not fluctuate consists of the order, disposition, and relationships of
because required tasks never change. The work positions and lines of responsibility within an entity. In
environment is impersonal, which limits individual a tall organizational structure, decision-making
growth of employees because no incentive for responsibility is shared by many levels.
Answer (A) is incorrect because coercive power is
Answer (D) is incorrect because it is an advantage of the ability of the individual to make others cooperate
a flat organizational structure, which concentrates by applying pressure.
decision-making power at one level.
Answer (B) is incorrect because legitimate power is
the leader's right to expect cooperation from others.
[17] Source: Publisher
Answer (C) is correct. A person who is the head of a
Answer (A) is incorrect because the lack of company may exert influence through five types of
supervision increases employee flexibility. power. Referent power is the capacity for the
individual's personality and style to cause others to
Answer (B) is incorrect because the number of identify with or like him/her.
management levels limits the opportunity for
advancement, which may cause high employee Answer (D) is incorrect because reward power is the
turnover. individual's ability to influence others because they
expect good behavior to be rewarded.
Answer (C) is incorrect because managers spend too
much time supervising, and employees do not receive
enough training. [21] Source: Publisher

Answer (D) is correct. A flat organizational structure Answer (A) is incorrect because delegation passes
concentrates decision-making authority at one level. some decision-making power to subordinates.
Tasks and performance objectives may be unclear to
employees because of a lack of supervision. Answer (B) is correct. Delegation is the assignment
of a manager's authority and/or workload to his/her
subordinates. By increasing subordinates'
[18] Source: Publisher responsibilities, the manager gives them more
opportunity to exercise judgment and become more
Answer (A) is correct. The span of control is the confident.
number of subordinates for which a manager or
supervisor is responsible. When the span of control is Answer (C) is incorrect because delegation gives
narrow, a few subordinates are tightly controlled. A subordinates the opportunity to increase their
narrow span of control is typical of tall organizational expertise.
structures.
Answer (D) is incorrect because delegation gives
Answer (B) is incorrect because it is a likely effect of subordinates more responsibilities.
a wide span of control, which is likely to be found in
flat organizational structures.
[22] Source: Publisher
Answer (C) is incorrect because it is a likely effect of
a wide span of control, which is likely to be found in Answer (A) is correct. Managers fear delegating
flat organizational structures. because of insecurity, mistrust, insufficient planning,
and/or aversion to the risk of being responsible for
Answer (D) is incorrect because it is a likely effect of subordinates' actions.
a wide span of control, which is likely to be found in
flat organizational structures. Answer (B) is incorrect because this is a fear that
subordinates have about delegation.

[19] Source: Publisher Answer (C) is incorrect because it states a potential


implementation problem.
Answer (A) is correct. The most important factors to
consider are the employees' and manager's Answer (D) is incorrect because this is a fear that
preferences and skills, the firm's culture, the tasks subordinates have about delegation.
involved, physical location of the department, and
established policies and procedures.
[23] Source: Publisher
Answer (B) is incorrect because, although examining
similar entities may be useful, it is not one of the most Answer (A) is incorrect because this typifies a highly
important considerations. centralized entity.

Answer (C) is incorrect because this consideration Answer (B) is incorrect because this typifies a highly
would only be necessary if required by an agreement centralized entity.
with the creditors.
Answer (C) is incorrect because decentralization
Answer (D) is incorrect because this is not normally would give these specialists more authority.
considered in a span-of-control decision.
Answer (D) is correct. Decentralization is the extent
to which decision-making power is delegated within
[20] Source: Publisher an entity. When approval from upper-level
management is not required, lower-level managers
can make more timely decisions because they are acceptance contributes to group cohesiveness by
closer to the necessary sources of information. helping to define roles within the group.

Answer (D) is incorrect because interpersonal trust


[24] Source: CMA 1293 1-22 enhances the ease with which a group is formed and
functions.
Answer (A) is incorrect because an organizational
chart can be used in decentralized as well as
centralized organizations.
[27] Source: CIA 0590 III-5
Answer (B) is incorrect because not-for-profit
agencies use organizational charts for the same Answer (A) is correct. By reason of their status and
reasons as profit-oriented companies. authority, managers have substantial interpersonal
contact. A figurehead plays a symbolic role; for
Answer (C) is incorrect because both staff and line example, by performing such ceremonial functions as
functions are depicted on organizational charts. signing documents and receiving visitors. A leader
motivates employees to perform jobs properly. A
Answer (D) is correct. An organizational chart is liaison serves as a link in both vertical and horizontal
used to represent the organizational structure of an chains of communication.
entity. It normally resembles a pyramid, with the chief
executive on top and the operating work force on the Answer (B) is incorrect because these are decisional
bottom. Lines show reporting relationships, lines of roles. Disturbance handler is the fourth decisional
authority, and task groupings. An organizational chart role, according to Mintzberg.
depicts promotional or career tracks and illustrates
the span of control and the number of organizational Answer (C) is incorrect because these are
levels. informational roles.

Answer (D) is incorrect because an entrepreneur has


[25] Source: CMA 1293 1-23 a decisional role and a disseminator has an
informational role.
Answer (A) is incorrect because a group decision,
like an individual decision, is better when based on
sufficient information. [28] Source: CIA 0591 III-1

Answer (B) is incorrect because acceptance of each Answer (A) is incorrect because the interpersonal
member of the committee and the goals of the role primarily concerns superior-subordinate
committee is essential. relationships.

Answer (C) is incorrect because the more knowledge Answer (B) is incorrect because the informational
that group members possess, the better the ultimate role is a communication role.
decision is apt to be.
Answer (C) is incorrect because the external role is
Answer (D) is correct. Group decision making, such not among Mintzberg's three classifications of
as by a committee, works better when the committee managerial roles.
is small and the members accept each other as
contributing parts of the group. A dominant member Answer (D) is correct. Mintzberg describes three
is not desirable. The chair should act as a moderator categories of managerial roles: interpersonal
and not be considered threatening or overbearing by (figurehead, leader, liaison), informational (nerve
the other members. center, disseminator, spokesperson), and decisional
(entrepreneur, disturbance handler, resource
allocator, negotiator). The manager described is
[26] Source: CMA 1293 1-24 performing the decisional sub-role of negotiator.

Answer (A) is incorrect because agreement about


goals facilitates group harmony. [29] Source: CIA 0592 III-3

Answer (B) is correct. People naturally seek Answer (A) is incorrect because, in Victor Vroom's
association and group acceptance. Members of valence-expectancy theory, the strength of motivation
equals valence (strength of preference for an
groups react to pressures of the group. Groups are
often complex, develop their own leaders, exist to fill outcome) times expectancy (probability that an act
the needs of the members, and result from the will lead to the desired result).
frequent interaction among individuals in the course of
their work. In a sense, group members typically have Answer (B) is correct. Kreitner states that the
an experience of togetherness. Members develop variables in the basic formula for effective
interpersonal trust for other group members. The management are ability, motivation, and opportunity.
formation of small subgroups, however, can splinter Ability is the "capacity to achieve organizational
the harmony of the large group. objectives both effectively and efficiently." Motivation
is "a persistent desire to move ahead." Opportunity
Answer (C) is incorrect because leadership has two components according to Kreitner: a suitable
job in management and a supportive climate. would not make operating decisions for a production
department. The term administrative management
Answer (C) is incorrect because, according to may also apply to those middle managers who
situational theorists, the appropriate leadership style establish objectives and strategies for organizational
can be selected by considering the leader, the subunits consistent with strategic plans developed by
follower, and the situation. top management.

Answer (D) is incorrect because self-actualization, Answer (D) is incorrect because middle- and
status, and acceptance are categories in Maslow's upper-level managers must attend to issues broader
hierarchy of needs. than the details of everyday operations.

[30] Source: CIA 0592 III-6 [33] Source: Publisher

Answer (A) is correct. Peters and Waterman found Answer (A) is incorrect because the concept of
that the basics of customer priority, employee centralization and decentralization involves levels of
development and participation, and fostering of new decision making, not whether all managers have
ideas led to excellence in the marketplace. Customer separate offices.
input and satisfaction are emphasized, risk taking and
innovation are encouraged, and individuals are Answer (B) is incorrect because geographical
treated with respect. separation of divisional and central headquarters can
support either a centralized or decentralized
Answer (B) is incorrect because planning, organizing, environment.
and controlling are management functions.
Answer (C) is correct. The primary distinction
Answer (C) is incorrect because interpersonal, between centralization and decentralization is in the
informational, and decisional are the managerial role degree of freedom of decision making by managers at
categories defined by Mintzberg. many levels. In decentralization, decision making is at
as low a level as possible. The premise is that the
local manager can make better (more informed)
Answer (D) is incorrect because leading, motivating, decisions than a centralized manager. Centralization is
and communicating are management functions. based on the theory that decision making must be
consolidated so that activities throughout the
organization may be more effectively coordinated. In
[31] Source: CIA 0590 III-4 most organizations, a mixture of these approaches is
found to be best.
Answer (A) is incorrect because functional managers
have authority over a specific operational area (e.g., Answer (D) is incorrect because the relative size of
finance). They are middle managers who are the firm may be a factor in determining whether to
concerned primarily with intermediate planning. maintain a centralized or decentralized environment,
but relative size is not a primary difference between
Answer (B) is correct. A first-line or lower-level centralization and decentralization.
manager directly supervises employees who are not
managers. Operational planning is a type of
short-range or tactical planning performed by lower [34] Source: CIA 1185 III-5
management. It involves the development of action
plans by which strategies and policies are executed. Answer (A) is incorrect because decentralization
encourages development of lower-level managers.
Answer (C) is incorrect because department They will have greater responsibilities and authority.
managers are middle managers.
Answer (B) is incorrect because top managers will be
Answer (D) is incorrect because middle managers freed from operating problems.
are concerned primarily with intermediate planning.
Answer (C) is correct. The disadvantages of
decentralization include a tendency to focus on
[32] Source: CIA 0587 III-19 short-run results to the detriment of the long-term
health of the entity, an increased risk of loss of
Answer (A) is correct. Middle- and upper-level control by top management, the increased difficulty of
managers do not have the time or the expertise to coordinating interdependent units, and less
make every decision in a complex organization. Thus, cooperation and communication among competing
a first-line supervisor would normally be given the decentralized unit managers.
authority and held responsible for detailed operating
tasks. Answer (D) is incorrect because decision-making
power should motivate lower-level managers.
Answer (B) is incorrect because middle- and
upper-level managers must attend to issues broader
than the details of everyday operations. [35] Source: Publisher

Answer (C) is incorrect because a staff or Answer (A) is incorrect because dysfunctional
support-level manager, such as a personnel director, decision-making is a cost of decentralization.
the greatest returns from supervisory effort may be
Answer (B) is incorrect because a decreased achieved.
understanding of the overall goals of an organization
is a cost of decentralization.
[38] Source: CIA 1196 III-20
Answer (C) is incorrect because increased costs for
developing the information system is a cost of Answer (A) is incorrect because Japan has very
decentralization. limited raw materials.

Answer (D) is correct. The costs of centralized staff Answer (B) is correct. Much of the credit for Japan's
may actually decrease under decentralization. On the success has been given to its management systems
other hand, the corporate staff and the various and the ability to be efficient with limited resources.
services they provide may have to be duplicated in Japanese society and behavior is highly structured,
various divisions, thereby increasing overall costs. which is conducive to effectiveness and efficiency.
Suboptimal decisions may result from disharmony
among organizational goals, subgoals of the division, Answer (C) is incorrect because the infrastructure
and the individual goals of managers. The overall has not been constantly refurbished, and frequent
goals of the firm may more easily be misunderstood change is not conducive to efficiency.
because individual managers may not see the larger
picture. Moreover, the information system necessary Answer (D) is incorrect because the Japanese
for adequate reporting in a decentralized mode will educational system does not emphasize creativity.
tend toward redundancy, which increases costs.

[39] Source: CIA 1196 III-22


[36] Source: CIA 0586 III-5
Answer (A) is incorrect because daily productivity
Answer (A) is incorrect because compliance with relates to short-term effectiveness only.
governmental regulations is probably more easily
achieved by centralization. A disadvantage of Answer (B) is incorrect because a survey of
decentralization is the difficulty of assuring uniform employee morale may contribute to assessing
action by units of the entity that have substantial effectiveness, but it is not sufficient for assessing
autonomy. overall effectiveness.

Answer (B) is incorrect because decentralization may Answer (C) is incorrect because comparing
result in duplication of efforts, resulting in less efficient production against goals is a measure of short-term
use of headquarters staff officials and specialists. effectiveness.

Answer (C) is incorrect because decentralization may Answer (D) is correct. Kreitner (6th ed., pages
result in duplication of efforts, thereby increasing 279-80) states, "Organizational effectiveness can be
overall costs. defined as meeting organizational objectives and
prevailing societal expectations in the near future,
Answer (D) is correct. Decentralization results in adapting and developing in the intermediate future,
greater speed in making operating decisions because and surviving in the distant future. In the near term
they are made by lower level managers instead of (about one year), it should be effective in achieving its
being referred to top management. The quality of goals, efficient in its use of resources, and a source of
operating decisions should also be enhanced, satisfaction to its constituencies (owners, employees,
assuming proper training of managers, because those customers, and society). In the intermediate term (2
closest to the problems should be the most to 4 years), it should adapt to new possibilities and
knowledgeable about them. obstacles and develop its abilities and those of its
members. In the long term (5+ years), the
organization should be able to survive in an uncertain
[37] Source: CIA 0595 III-34 world.

Answer (A) is incorrect because, in MBO, a


manager and his/her subordinates jointly formulate the [40] Source: CIA 0596 III-14
subordinates' objectives and the plans for attaining
them. Answer (A) is incorrect because spatial differentiation
refers to the degree of geographic dispersion of
Answer (B) is incorrect because, in a responsibility facilities and personnel.
accounting system, managers are evaluated only on
the basis of factors they control. Answer (B) is incorrect because formalization refers
to the degree of job standardization through
Answer (C) is incorrect because benchmarking is the descriptions and rules.
practice of identifying, studying, and building upon the
best practices in the industry or in the world. Answer (C) is correct. Vertical differentiation
concerns the depth of the organizational hierarchy.
Answer (D) is correct. Management-by-exception The greater the number of levels, the more complex
gives significant attention only to those areas in which the organization, the greater the potential for
material variances from expectations occur. information distortion, the more difficult the
Consequently, management focuses resources where coordination of management activities, and the slower
and less effective the response to changing vertical differentiation, high horizontal differentiation,
conditions. and high formalization. This structure is one in which
tasks are well-defined, most communication is
Answer (D) is incorrect because formalization refers downward, and control is tight.
to the degree of job standardization through
descriptions and rules. Answer (D) is incorrect because an integrated
structure is a nonsense term in this context.

[41] Source: CIA 1196 III-1


[44] Source: CIA 0594 II-22
Answer (A) is correct. A dynamic and complex
organizational environment faces constant change, so Answer (A) is incorrect because a line and staff
the level of uncertainty increases. The more structure is designed to maximize unity-of-command
uncertainty an organization faces, the more organic by giving only line managers the authority to make
the structure should be. Organic organizations tend to decisions affecting those in their chain of command.
be flexible and adaptive to change.
Answer (B) is incorrect because a strategic business
Answer (B) is incorrect because a dynamic and unit is a subunit that is treated as an independent
complex environment is more uncertain, so it requires business. Thus, unity-of-command is not an issue for
a flexible structure. a strategic business unit.

Answer (C) is incorrect because a dynamic and Answer (C) is incorrect because a centralized
complex environment is more uncertain, so it requires structure need not have unity-of-command problems
a flexible structure. if management is organized in a line and staff fashion.

Answer (D) is incorrect because a dynamic and Answer (D) is correct. The matrix allows authority to
complex environment is more uncertain, so it requires flow both vertically and horizontally. A manager is
a flexible structure. appointed for each project and draws on personnel
who are organized by function and report to a
manager for each function. This violates the principle
[42] Source: CIA 0595 III-9 of unity of command, which states that each
subordinate should have only one superior.
Answer (A) is incorrect because, in a bureaucracy,
each subordinate reports to a single manager.
[45] Source: CIA 0596 III-15
Answer (B) is correct. A matrix organization (project
management) is characterized by vertical and Answer (A) is incorrect because a mechanistic
horizontal lines of authority. The project manager structure is appropriate for organizations focusing on
borrows specialists from line functions as needed. cost minimization through tight controls, extensive
This manager's authority is limited to the project, and division of labor, high formalization, and
the specialists will otherwise report to the line centralization.
managers.
Answer (B) is incorrect because an imitation strategy
Answer (C) is incorrect because departmental is not adopted by true innovators but rather by
organization structures represent the typical companies that move into new markets only after
organization with unified and clear single lines of smaller competitors have demonstrated the potential
authority. for success. Imitation strategies are best suited to a
structure that combines mechanistic and organic
Answer (D) is incorrect because mechanistic components.
organization structure is another term for a
bureaucracy. Answer (C) is correct. Organizational structure
depends on the company's overall strategy. This
organization has adopted an innovation strategy. It
[43] Source: CIA 1196 III-3 introduces major new products or services. The
structure that provides the flexibility required for
Answer (A) is incorrect because an organic structure major innovation is organic, a form of organization
is flexible and therefore not suited to mass characterized by a loose structure, low division of
production. labor, low formalization, and decentralization.

Answer (B) is incorrect because matrix is not a type Answer (D) is incorrect because bureaucracies are
of structure but rather a type of departmentation. mechanistic, not organic.

Answer (C) is correct. According to Joan


Woodward's work on the relationship of technology [46] Source: CIA 1196 III-7
and structure in manufacturing, companies may be
categorized as engaged in unit production (units or Answer (A) is incorrect because innovation
small batches), mass production (large batches), or minimization is not a type of strategy. An innovation
process production (continuous processing). Mass strategy emphasizes the introduction of new products
production is most effective if the entity has a or services.
mechanistic structure characterized by moderate
Answer (B) is incorrect because an imitation strategy
seeks to introduce new products or enter new Answer (B) is incorrect because the structure is
markets only after their viability has been proven. formal and compartmentalized.

Answer (C) is correct. Research suggests that Answer (C) is incorrect because the structure is
organizational structure follows strategy. A formal and compartmentalized.
cost-minimization strategy tightly controls costs,
avoids unnecessary innovation or marketing Answer (D) is correct. A high division of labor results
expenses, and cuts prices in selling a discount in specialization. Unity of command clarifies authority
product. The sellers of generic grocery products and responsibility relationships. An organizational
pursue this strategy. structure with a high division of labor and a strict unity
of command is a classic bureaucracy. Thus, it is
Answer (D) is incorrect because there is no initiation compartmentalized and formal.
strategy.

[50] Source: CIA 1195 III-26


[47] Source: CIA 1195 III-24
Answer (A) is correct. A professional bureaucracy
Answer (A) is incorrect because the size-structure (e.g., a university or library) is a complex and formal
relationship is likely to be linear but only within a but decentralized organization in which highly trained
narrow range. specialists have great autonomy. The jobs performed
by these professionals require years of education and
Answer (B) is incorrect because the size-structure training. By their nature, such jobs can be performed
relationship is likely to be linear but only within a effectively only if the employees are afforded
narrow range. substantial autonomy.

Answer (C) is incorrect because the size-structure Answer (B) is incorrect because both a machine
relationship is likely to be linear but only within a bureaucracy and a professional bureaucracy can
narrow range. accomplish routine tasks in a highly efficient manner.

Answer (D) is correct. As an organization increases Answer (C) is incorrect because both organizational
in size, its structure tends to become more formal and structures thrive on rules.
mechanistic. More policies and procedures are
necessary to coordinate the increased number of Answer (D) is incorrect because subunit conflicts are
employees, and more managers must be hired. typical of bureaucracies. Specialization tends to result
However, the relationship between size and changes in a focus on subunit rather than organizational
in structure is linear only within a certain range. For objectives.
example, adding 100 employees to a company with
100 employees is likely to cause significant structural
change, but adding the same number to a workforce [51] Source: CIA 1195 III-25
of 10,000 is unlikely to have little impact. By the time
a company reaches a certain size (1,500 to 2,000 or Answer (A) is incorrect because a simple structure
more), it usually has most of the qualities of a has low complexity and formality, and authority is
mechanistic structure. centralized. Its small size and simplicity usually
precludes significant inefficiency in the use of
resources.
[48] Source: CIA 1196 III-6
Answer (B) is correct. A division is essentially a
Answer (A) is correct. The strategies of larger self-contained organization. Hence, it must perform
organizations tend to be more ambitious and all or most of the functions of the overall organization
complex. Thus, a company may expand from one of which it is a part. It is characterized by substantial
product line to many. The result is a need for more duplication of functions when compared with more
complicated coordination methods. centralized structures.

Answer (B) is incorrect because a company with Answer (C) is incorrect because a machine
diversified products may decide to create bureaucracy is a complex, formal, and centralized
independent, decentralized divisions. organization that performs highly routine tasks,
groups activities into functional departments, has a
Answer (C) is incorrect because focusing on a single strict chain of command, and distinguishes between
product line rather than a diverse set of products is line and staff relationships.
not a strategy for a growing company.
Answer (D) is incorrect because a professional
Answer (D) is incorrect because structure follows bureaucracy (e.g., a university or library) is a
strategy. complex and formal but decentralized organization in
which highly trained specialists have great autonomy.
Duplication of functions is minimized. For example, a
[49] Source: CIA 1195 III-21 university would have only one history department.

Answer (A) is incorrect because the structure is


formal and compartmentalized. [52] Source: CIA 0595 III-27
Answer (A) is incorrect because a focus group [55] Source: CIA 1188 III-8
includes a small number of consumers brought
together by marketing researchers to discuss a given Answer (A) is incorrect because authority is more
topic. concentrated in centralized management structures.

Answer (B) is incorrect because a reengineering Answer (B) is correct. When an organization changes
process team is assembled to search for and from a centralized to a decentralized structure, top
implement a radical redesign of basic business management is delegating more authority to middle
activities. and lower levels. Thus, managers at these lower
levels are usually hired and developed more
Answer (C) is incorrect because a matrix rigorously than under the centralized structure.
organization is characterized by dual lines of authority
and a combination of functional and product Answer (C) is incorrect because some effort will
departmentation. It puts similar specialists together. inevitably be duplicated under decentralization, of
which departmentalization is a moderate form.
Answer (D) is correct. An ad hoc committee is the
structure best suited for specialized, one-of-a-kind Answer (D) is incorrect because departments are
projects that require diverse specialists to work in formed when one manager can no longer supervise
small teams. It is simple, informal, decentralized, and the entire organization; departmentalization is
temporary. It has no hierarchy, permanent therefore a characteristic of centralized as well as
departments, standard procedures for accomplishing decentralized organizations.
routine tasks, or formal rules. Consequently, an ad
hoc organization is flexible and responsive.
[56] Source: CIA 0585 III-8

[53] Source: CIA 1196 III-4 Answer (A) is correct. "Flat" organizational structures
have relatively few levels from top to bottom. "Tall"
Answer (A) is correct. Technology has been viewed organizational structures have many levels between
in terms of two dimensions: task variability and top and bottom. Flat structures have the advantages
problem analyzability, or the exceptions encountered of fast information flow from top to bottom of the
in work and the search procedures for responding to organization and increased employee satisfaction. Tall
the exceptions, respectively. Routine tasks have few structures are faster and more effective at problem
exceptions, and the search for solutions is well resolution because of the increased frequency of
defined. Because routine tasks change little over time, interaction between superior and subordinate, and the
they are well suited to a mechanistic structure. One greater order imposed by the hierarchy. Studies do
characteristic of such a structure is high formalization. not indicate great advantages for either flat or tall
structures.
Answer (B) is incorrect because decentralized
decision making is not suited to routine tasks but to Answer (B) is incorrect because it is an advantage of
organic structures. a flat structure.

Answer (C) is incorrect because organic structures Answer (C) is incorrect because it is an advantage of
are best when problems are poorly defined. a flat structure.

Answer (D) is incorrect because job satisfaction is Answer (D) is incorrect because it is an advantage of
often low in routine and repetitive tasks. a flat structure.

[54] Source: CIA 1194 III-9 [57] Source: CIA 1188 III-9

Answer (A) is incorrect because decentralized Answer (A) is incorrect because close is not a
organizations are more difficult to control. span-of-control category.

Answer (B) is incorrect because centralized Answer (B) is incorrect because a narrow span of
structures streamline organizations and eliminate control is useful where jobs are dissimilar, procedures
duplication of resources. are not standardized, and subordinates are more
dispersed.
Answer (C) is incorrect because the number of
managers is not related to the degree of centralization Answer (C) is incorrect because a moderate span of
or decentralization but is a function of the span of control is useful in situations that have some
control. characteristics of both extremes, wide and narrow.

Answer (D) is correct. A decentralized organization Answer (D) is correct. In any situation, there are
allows lower level employees to participate in underlying variables that influence the number of
decision making. This increased involvement subordinates a manager can supervise. In general, if
encourages initiative and creative thinking and is jobs are similar, procedures are standardized, and
especially appropriate in complex and rapidly physical dispersion is minimized, a wide span of
changing environments. control is most effective.
control is most effective.
[58] Source: CIA 1191 III-3
Answer (D) is incorrect because geographical
Answer (A) is incorrect because managers who can dispersion would decrease rather than increase the
contact subordinates frequently are able to control span of control.
more people than those who have relatively
infrequent contact with subordinates.
[61] Source: CIA 0596 III-19
Answer (B) is incorrect because managers who
delegate authority have more time to control the Answer (A) is incorrect because anticipatory changes
subordinates who report to them. These individuals are systematically planned changes intended to take
can therefore supervise more people than managers advantage of expected situations. In the
who prefer not to delegate authority. Nadler-Tushman model, tuning is an incremental
anticipatory change, and re-orientation is a strategic
Answer (C) is incorrect because managers who have anticipatory change.
received effective training and are skillful
communicators are equipped to control more Answer (B) is correct. Reactive changes are
necessitated by unexpected environmental events or
individuals than managers who are untrained and/or pressures. They may be incremental or strategic. In
have deficient communication skills. the Nadler-Tushman model (Kreitner, Robert,
Management, 6th ed., Houghton Mifflin, 1995, pp.
Answer (D) is correct. The optimal span of control is 498-499), adaptation is an incremental reactive
the number of subordinates that a given manager can change, and re-creation is a strategic (and risky)
effectively supervise. It is a function of many reactive change.
situational factors. However, the total number of
people in an organization has no bearing on the Answer (C) is incorrect because incremental changes
optimal span of control of a particular manager. involve subsystem adjustments needed to keep the
organization on its chosen path.

[59] Source: CIA 0594 III-79 Answer (D) is incorrect because strategic changes
alter the overall shape or direction of the organization.
Answer (A) is correct. Flat structures have the
advantages of fast information flow from top to
bottom of the organization and increased employee [62] Source: Publisher
satisfaction. Tall structures are faster and more
effective at problem resolution because of the Answer (A) is incorrect because it relies on
increased frequency of interaction between superior something more than personality as the source of
and subordinate and the greater order imposed by power.
the hierarchy. For a flat structure to be successful,
employees must be able to work without supervision Answer (B) is incorrect because it relies on
much of the time because a manager with many something more than personality as the source of
employees has little time for each one. power.

Answer (B) is incorrect because geographically Answer (C) is incorrect because it relies on
dispersed work areas are very difficult for a manager something more than personality as the source of
with many subordinates to control. power.

Answer (C) is incorrect because tasks that are highly Answer (D) is correct. Power may be classified as
complex and varied are more appropriate for a reward power (the leader controls resources),
narrow span of control. coercive power (the leader may punish the
subordinate), legitimate power (the leader has the
Answer (D) is incorrect because a narrow span of right to lead), referent power (the leader has fame,
control (a tall structure) is more appropriate when charisma, etc.), and expert power (the leader has
subordinates perform distinctly different tasks. specialized ability or knowledge).

[60] Source: CIA 1192 III-9 [63] Source: Publisher

Answer (A) is incorrect because, although a manager Answer (A) is incorrect because she does not have
under these conditions would be able to supervise a the power to coerce others.
large number of employees, an upper limit must exist.
Answer (B) is incorrect because she has no power to
Answer (B) is incorrect because the conditions reward others.
described support a wide rather than a narrow span.
Answer (C) is correct. The internal audit director has
Answer (C) is correct. In any situation, there are no formal (legitimate or position) power over
underlying variables that influence the number of auditees. Nor does she have the power to coerce
subordinates a manager can supervise. In general, if (punish) or reward them. Rather, her ability to exert
jobs are similar, procedures are standardized, and power (influence others) must derive from her
physical dispersion is minimized, a wide span of specialized ability and knowledge and the force of her
personal qualities. experiences, and abilities usually results in considering

Answer (D) is incorrect because she does not have more solutions to a problem.
the power to coerce others.

[67] Source: CIA 1195 II-3


[64] Source: Publisher
Answer (A) is incorrect because brainstorming
Answer (A) is incorrect because the Golden Rule is generates a large number of ideas. It helps group
still an effective philosophy in certain cases. members overcome the pressure to conform while
the group is identifying options. It has no predictable
Answer (B) is correct. Consideration of subordinates effect, however, on the group's commitment to the
and their behavioral needs is an outgrowth of the solution.
behavioral school of thought. The Golden Rule was
an early attempt to meet the needs of the individual in Answer (B) is incorrect because top management
human relationships. The weakness of this approach mandates are unlikely to result in a high level of
is the assumption that all others want what the leader commitment, and they make group formation
wants, i.e., that all people are alike. pointless.

Answer (C) is incorrect because the fear of Answer (C) is incorrect because the Delphi technique
disobeying is a traditional approach to management uses a series of questionnaires, the results of which
that has nothing to do with the Golden Rule. are compiled and distributed to group members. This
process continues until a solution is reached. It is
Answer (D) is incorrect because the Golden Rule can effective for generating a large number of ideas and
be applied in any circumstance, regardless of the for arriving at a consensus. However, because
number of ways to perform a job. members are anonymous and do not interact, the
method results in a low level of commitment to the
solution.
[65] Source: Publisher
Answer (D) is correct. Interacting groups, that is,
Answer (A) is correct. The Hawthorne studies traditional groups in which individuals meet
showed that workers did not respond directly to a face-to-face and interact in customary ways,
physical change but rather to their perception of the potentially foster the greatest commitment to the
change. Feelings resulting from change determined solution reached. When individuals who must
employee response to change. Social acceptance implement the solution participate in the decision
was found to be more important than wages in through open discussion and arrive at a consensus,
determining individual output. they are usually inclined to accept the decision.

Answer (B) is incorrect because wages were found


to be less important than social acceptance. [68] Source: CIA 1195 II-33

Answer (C) is incorrect because an industrial Answer (A) is incorrect because a quality circle is a
engineering approach is a part of classical small group of subordinates and supervisors, usually
management theory. eight to 10 people.

Answer (D) is incorrect because workers did not Answer (B) is incorrect because each member is
respond directly to physical changes, but to their responsible for the success of the circle, and success
perception of the change. depends on the ability of members to analyze and
solve problems.

[66] Source: CIA 1195 II-1 Answer (C) is incorrect because quality circles are
used by companies to accomplish objectives.
Answer (A) is incorrect because responsibility for Participation is part of each worker's job.
group decisions is more diffuse than when individuals
make decisions. Answer (D) is correct. Use of quality circles is a form
of participative management. A quality circle is a
Answer (B) is incorrect because group decision group of up to 10 individuals (managers and
making almost always takes more time than individual subordinates) who do similar work and who
decision making, except when the need for diverse volunteer to meet weekly to discuss and solve
views is so great that an individual decision maker work-related problems. However, management
needs to consult many people or perform research. retains the right to make the final decisions.

Answer (C) is incorrect because group members


usually have diverse views, but their common need to [69] Source: CIA 1196 II-37
be accepted and respected by the group often
restrains the full, open expression of their views when Answer (A) is incorrect because, if members of the
they fear strong disagreement. group are responsible for the decision making, their
participation in the implementation process will
Answer (D) is correct. Groups tend to be more increase the ease with which the decisions are carried
creative than individuals. Diversity of member views, out.
Answer (A) is correct. A first-line or lower-level
Answer (B) is incorrect because group decision manager directly supervises employees who are not
making adds legitimacy to the solution by following managers. Operational planning is a type of
democratic methods. short-range or tactical planning performed by lower
management. It involves the development of action
Answer (C) is incorrect because a group possesses plans by which strategies and policies are executed.
greater resources than an individual. According to Kreitner, it is "the process of
determining how specific tasks can best be
Answer (D) is correct. The groupthink phenomenon accomplished on time with available resources."
is undesirable. Groupthink occurs when group
members accept what appears to be the group Answer (B) is incorrect because a department head
consensus rather than giving their honest input. The is a middle manager and is concerned with
result may be decisions with which some members of intermediate planning.
the group are not happy.
Answer (C) is incorrect because a general manager is
a member of top management and is concerned with
[70] Source: CIA 1194 III-8 strategic planning.

Answer (A) is incorrect because authority is the right Answer (D) is incorrect because the chief executive
to do things, and power is the ability to do things. officer is the most senior manager in an organization
and focuses on strategic planning.
Answer (B) is correct. Authority is the officially
sanctioned privilege to direct others. A clear
hierarchy of authority enhances coordination and [73] Source: CIA 0592 III-9
accountability. Power is the ability to marshal
organizational resources to obtain results. A manager Answer (A) is incorrect because the board of
may have both authority and power, or one without directors is not involved in operations.
the other.
Answer (B) is incorrect because top management is
Answer (C) is incorrect because a manager may responsible for strategic planning.
accomplish a task without having formal authority.
Answer (C) is correct. Middle managers, such as
Answer (D) is incorrect because authority is the right department heads and functional managers, are
to do things, and power is the ability to do things. responsible for intermediate planning with a time line
of approximately 6 months to 2 years. Thus, the
manager of the production function should be
[71] Source: CIA 1193 III-3 responsible for the 6-month production schedule.

Answer (A) is incorrect because referent power is Answer (D) is incorrect because first-line supervision
the influence that a person has over those who is involved in day-to-day operations.
identify with him/her if they comply on that basis
alone.
[74] Source: CIA 0590 III-6
Answer (B) is correct. In Management, 6th ed.
(Boston: Houghton Mifflin Company, 1995, p. 4), Answer (A) is incorrect because it lists functional
Robert Kreitner defines management as "the process categories.
of working with and through others to achieve
organizational objectives in a changing environment. Answer (B) is correct. The interpersonal roles
Central to this process is the effective and efficient (figurehead, leader, liaison) are necessary because,
use of limited resources." Thus, management is a given his/her authority and status, a manager has
social process that attempts to use scarce resources substantial interpersonal contacts, particularly with
to achieve organizational objectives while anticipating peers and subordinates. The informational roles
environmental changes and balancing effectiveness (nerve center, disseminator, spokesperson) reflect the
against efficiency. The physical, social, political, importance of information to organizational activity.
moral, and informational environments external to the Managers must receive and transmit information to
organization are the "sources of change" that parties both within and outside the organization. The
managers must anticipate. decisional roles require managers to make choices
and balance divergent interests. Decisions involve
Answer (C) is incorrect because informal leadership developing strategies and implementing them.
is influencing others to pursue unofficial objectives
that may or may not serve the organization's interest. Answer (C) is incorrect because it lists functional
categories.
Answer (D) is incorrect because motivation, an
important part of the manager's job, is the Answer (D) is incorrect because it lists functional
psychological process that gives behavior purpose categories.
and direction.

[75] Source: CIA 0593 III-3


[72] Source: CIA 1193 III-1
Answer (A) is incorrect because designing and
initiating change is an example of the positions are directly related to the service(s) and/or
entrepreneurial-decisional role of a manager. product(s) offered by the firm.

Answer (B) is incorrect because transmitting Answer (B) is correct. Staff positions in any
information to subordinates is an example of the organization advise and support the line positions.
disseminator-informational role of a manager. They indirectly help to achieve the organization's
basic objective. Each staff position's authority is, at
Answer (C) is incorrect because participating in most, functional. Such authority is exercised only over
negotiations is an example of the activities related to the staff's function or specialty.
negotiator-decisional role of a manager.
Answer (C) is incorrect because they are examples
Answer (D) is correct. Mintzberg describes three of line positions that are directly involved in the
categories of managerial roles: interpersonal, achievement of the organization's objectives. Line
informational, and decisional. By reason of their positions are directly related to the service(s) and/or
status and authority, managers have substantial product(s) offered by the firm.
interpersonal contact. A figurehead plays a symbolic
role, for example, by performing such ceremonial Answer (D) is incorrect because they are examples
functions as signing documents and receiving visitors. of line positions that are directly involved in the
A leader motivates employees to perform jobs achievement of the organization's objectives. Line
properly. A liaison serves as a link in both vertical positions are directly related to the service(s) and/or
and horizontal chains of communication. product(s) offered by the firm.

[76] Source: CIA 0596 III-26 [79] Source: Publisher

Answer (A) is incorrect because participating in Answer (A) is incorrect because the manager of
negotiating contracts with vendors is a decisional role. checking is a line position which does not rely on the
other customer service managers.
Answer (B) is correct. According to Mintzberg, the
decisional roles of managers include entrepreneur, Answer (B) is incorrect because the department only
disturbance handler, resource allocator, and provides services for the customers with checking
negotiator. However, the leader role of motivating accounts.
subordinates to get the job done properly is one of
the interpersonal roles. Answer (C) is incorrect because the manager of
checking reports to the president indirectly through a
Answer (C) is incorrect because taking corrective vice president.
action in nonroutine situations is a decisional role.
Answer (D) is correct. The director of personnel
Answer (D) is incorrect because allocating (DP) must coordinate all employee evaluations.
organizational resources is a decisional role. Although the manager of checking services is in a line
position that is not under the DP, (s)he works with
the DP when hiring, dismissing, transferring, or
[77] Source: CIA 0595 III-31 evaluating employees.

Answer (A) is incorrect because the director is a


liaison who provides the link in the vertical chain of [80] Source: Publisher
communication, i.e., from the highest level (CEO)
down to the lowest level (staff level). Answer (A) is incorrect because the competence
standard pertains to the financial
Answer (B) is correct. According to Mintzberg, manager/management accountant's responsibility to
managerial roles may be categorized as interpersonal, maintain his/her professional skills and knowledge. It
informational, and decisional. The informational role also pertains to the performance of activities in a
of the CEO in providing selected information to
outsiders, i.e., to stock market analysts, is that of a professional manner.
spokesperson.
Answer (B) is incorrect because legality is not
Answer (C) is incorrect because the interpersonal addressed in the IMA Code of Ethics.
role of the leader is to motivate subordinates.
Answer (C) is correct. Objectivity is the fourth part
Answer (D) is incorrect because the interpersonal of the IMA Code of Ethics. It requires that
role of the figurehead is the symbol of legal authority information be communicated "fairly and objectively,"
and the one who performs certain ceremonial duties, and that all information that could reasonably
e.g., signing documents and receiving visitors. influence users be fully disclosed.

Answer (D) is incorrect because the confidentiality


[78] Source: Publisher standard concerns the financial manager/management
accountant's responsibility not to disclose or use the
Answer (A) is incorrect because they are examples firm's confidential information.
of line positions that are directly involved in the
achievement of the organization's objectives. Line
[81] Source: Publisher the theory that, while job satisfaction has an
economic importance only indirectly related to
Answer (A) is incorrect because the integrity productivity, satisfied workers exhibit lower rates of
standard requires the financial manager/management absenteeism, turnover, tardiness, apathy, and
accountant to "communicate unfavorable as well as sabotage. Organizations with more organic or flexible
favorable information and professional judgments or organization structures may use more of the worker's
opinions." skills and present a greater variety of work
challenges, thus improving morale. Mass production
Answer (B) is correct. One of the responsibilities of technology usually involves repetitious, boring tasks.
the financial manager/management accountant under
the competence standard is to "maintain an Answer (C) is incorrect because it should improve
appropriate level of professional competence by job satisfaction.
ongoing development of his/her knowledge and
skills." Answer (D) is incorrect because it should improve
job satisfaction.
Answer (C) is incorrect because one of the
suggestions from the "Resolution of Ethical Conflict"
paragraph is to "clarify relevant ethical issues by [84] Source: Publisher
confidential discussion with an objective advisor (e.g.,
IMA Ethics Counseling Service) to obtain a better Answer (A) is incorrect because job enrichment is
understanding of possible courses of action." based on the assumption that employees who have
qualitatively improved jobs need less supervision.
Answer (D) is incorrect because the confidentiality
standard requires the financial manager/management Answer (B) is incorrect because survival needs are at
accountant to "inform subordinates as appropriate the bottom of Maslow's hierarchy. Job enrichment
regarding the confidentiality of information acquired in seeks to meet the higher-level needs (affiliation,
the course of their work and monitor their activities to esteem, self-actualization).
assure the maintenance of that confidentiality."
Answer (C) is incorrect because the intrinsic factors
(challenge, growth, responsibility, etc.) are relevant to
[82] Source: Publisher job enrichment.

Answer (A) is incorrect because job enlargement Answer (D) is correct. Job enrichment increases the
increases the number of tasks assigned to one worker scope of boring, repetitive tasks by using more of the
but does not necessarily make those tasks more employee's skills and allowing the employee more
challenging or interesting. power to make decisions concerning the job, such as
order of tasks, etc. Thus, it encourages worker
Answer (B) is incorrect because job rotation merely participation in decisions previously made by
shifts workers among different jobs. management.

Answer (C) is correct. Job enrichment is an attempt


to apply the findings of Herzberg (two-factor theory [85] Source: Publisher
of behavior), Maslow (hierarchy of human needs),
Likert (participation), and McGregor (Theories X Answer (A) is correct. Job enlargement is a
and Y) by structuring the job so that each worker quantitative (horizontal) extension of the job. It does
participates in the planning and controlling so as to not necessarily involve any greater worker control,
maximize the satisfaction of both social and ego responsibility, or challenge. An example is job
needs and avoid the disadvantages of routine, highly rotation through assignment to duties requiring similar
specialized work. Job enrichment includes allowing skills.
and encouraging more worker discretion in deciding
work methods, work sequence, and work pace. It Answer (B) is incorrect because vertical loading is a
encourages interactions between workers, gives term used in management literature to describe
workers a feeling of personal responsibility for their qualitative job enhancement (enrichment). It entails
tasks, makes sure workers understand how their greater control and responsibility and increased
tasks contribute to the finished product, provides chances for growth.
feedback on job performance, and involves workers
in changes in the physical aspects of the work Answer (C) is incorrect because it is typical of job
environment. enrichment.

Answer (D) is incorrect because job simplification Answer (D) is incorrect because it is typical of job
clearly defines and may reduce the complexity of enrichment.
tasks through standardization of the job.

[86] Source: Publisher


[83] Source: Publisher
Answer (A) is incorrect because reduced opportunity
Answer (A) is incorrect because it should improve for advancement may motivate employees to find
job satisfaction. jobs elsewhere.

Answer (B) is correct. The literature tends to support Answer (B) is correct. Fringe benefits (e.g., pensions
and profit-sharing plans) may be contingent on leader is least effective in this situation.
duration of employment. Accordingly, they motivate
employees to remain with the company. Answer (B) is correct. A relationship-oriented
manager is employee centered. His/her self-esteem is
Answer (C) is incorrect because job simplification strongly affected by personal interactions with
may result in boredom, lessened job satisfaction, and subordinates. Fiedler indicated that such a manager is
a higher turnover rate. most effective when not faced with the extremes of
high or low control situations. High control follows
Answer (D) is incorrect because abolition of a from strong position power, a structured task, and
mandatory retirement age would be more good leader-member relations. A low control
appropriate. situation has just the opposite characteristics. In a
high-control environment, a concern for personal
relations may be unimportant. In a low-control
[87] Source: Publisher situation, the relationship-oriented leader may be
unable to provide the needed task structuring. Thus,
Answer (A) is incorrect because a more intrinsically the moderate control situation is best. An example is
satisfying job will improve the retention rate. an assembly-line situation (a structured task) in which
leader-member relations are poor.
Answer (B) is incorrect because employees subject
to discrimination may be strongly motivated to find Answer (C) is incorrect because the task-oriented
other jobs. leader is least effective in this situation.

Answer (C) is correct. When old employees discover Answer (D) is incorrect because the task-oriented
that newcomers are being hired at higher salaries, the leader is least effective in this situation.
group hired first will be unhappy, feeling that their
greater experience should warrant greater reward. In
the short run, the newcomers will be satisfied. But it is [90] Source: Publisher
possible that next year's newcomers will be hired at
still higher salaries, contributing to the overall Answer (A) is incorrect because a relationship
dissatisfaction. Thus, such a policy will provide (employee)-oriented approach may be preferable
incentives for experienced workers to leave the when tasks are highly structured.
company.
Answer (B) is correct. Fred E. Fiedler's contingency
Answer (D) is incorrect because greater freedom in theory of management holds that no single style of
determining his/her hours may enable a worker to directing is best for all occasions. A successful
remain with the company when other responsibilities director (leader) must, for each situation, balance
(e.g., child care) might have compelled him/her to his/her formal authority, the task structure, and the
leave. leader's relationships with the pertinent group
members.

[88] Source: Publisher Answer (C) is incorrect because a relationship


(employee)-oriented approach may be preferable
Answer (A) is correct. Human resource or human when tasks are highly structured.
asset accounting attempts to measure the value, and
the changes in value, of the organization's investment Answer (D) is incorrect because, when tasks are
in human assets. Although this "asset" is enormously ill-defined, the more effective manager may be one
valuable (sometimes estimated at two or three times who concentrates on defining and organizing the jobs
the annual payroll), it is not shown in balance sheets to be done rather than on motivating employees.
or accounted for in earnings statements. One
experimental measurement approach is a sort of
"present value" of human resources. Another is a [91] Source: Publisher
"cost" approach, with dollar investments (training,
customer goodwill, etc.) offset by reductions (e.g., Answer (A) is incorrect because, according to
retirement). Maslow, the highest need in the hierarchy is
self-actualization. It is the desire to become what one
Answer (B) is incorrect because traditional is capable of becoming, to realize one's potential, and
accounting methods do not value the organization's to accomplish to the limit of one's ability. Challenging
human assets. work helps to satisfy this need.

Answer (C) is incorrect because traditional Answer (B) is correct. Maslow saw human needs as
accounting methods do not value the organization's a hierarchy and held that lower-level needs
human assets. (physiological and safety needs) must be satisfied
before higher-level needs (acceptance by peers,
Answer (D) is incorrect because it merely audits the esteem, self-actualization) can influence the individual.
skills of the current employees and forecasts needs. Safety needs involve protection not only from
physical harm but also from economic insecurity.
Greater job and income security help to satisfy the
[89] Source: Publisher need for safety.

Answer (A) is incorrect because the task-oriented Answer (C) is incorrect because it is not part of
Maslow's hierarchy. diminish performance but whose presence will be a
motivator.
Answer (D) is incorrect because it is not part of
Maslow's hierarchy. Answer (C) is incorrect because a performance
decline followed a reduction in the security of
compensation; hence, money was probably a hygiene
[92] Source: Publisher (maintenance) factor according to Herzberg's
two-factor theory of motivation.
Answer (A) is correct. The behavioral theory of
management holds that all people (including Answer (D) is correct. A straight salary with
employees) have complex needs, desires, and commissions rewarded increased efforts while
attitudes. The fulfillment of needs is the goal toward providing greater security and a reduction in anxiety.
which employees are motivated. Effective leadership Satisfaction of the need for security may have
matches need-fulfillment rewards with desired permitted the employees to adopt promising and
behavior (tasks) that accomplishes organizational profitable but more risky strategies. Since people
goals. concerned about security needs tend to be risk
averse, the change in compensation method probably
Answer (B) is incorrect because management's role caused the performance decline.
in the directing process is to motivate people to
contribute toward accomplishment of organizational
goals. [95] Source: Publisher

Answer (C) is incorrect because, although Answer (A) is incorrect because MBO goals may be
management theories differ as to the motivational set in terms of quantitative measures (such as sales
value of wages, compensation is not a motivator for dollars) or qualitative ones (such as improved
all persons at all times. service).

Answer (D) is incorrect because while research has Answer (B) is incorrect because it describes
indicated that satisfaction and productivity are not management by exception.
directly related, behavioral theorists believe that they
must have some relationship. Answer (C) is correct. The hallmark of MBO is the
mutual setting of goals by the superior and the
subordinate as a basis for performance evaluation.
[93] Source: Publisher Based on the Theory Y philosophy that employees
want to work hard if they know what is expected,
Answer (A) is incorrect because Theory X and MBO requires top management participation and
Theory Y were used by McGregor to signify the commitment to the program, integration of the
extremes of managers' beliefs about employee objectives for all subunits into a compatible system
conduct. directed toward accomplishment of overall goals,
provision for regular reporting of performance, and
Answer (B) is correct. Victor Vroom's expectancy free and honest communication between superior and
theory holds that the force with which an individual is subordinates. Subordinates must make careful
motivated is a function of the valence of a motivator assessments of their abilities and their interests, and
(whether it has a positive or negative value to the managers must "coach" subordinates rather than
individual) and the expectancy, which is that dictate their proper goals. Both sides must maintain
individual's estimation of the probability that a flexibility to accommodate unforeseen changes, and
particular action will lead to a desired outcome. Thus, the review and analysis of results before setting the
motivation equals the valence times the expectancy. next round of goals is a vital part of the process.

Answer (C) is incorrect because Herzberg believed Answer (D) is incorrect because goal setting should
that maintenance factors (such as wages) must be be participative.
present to avoid diminished performance but that
motivational factors (such as challenge and
recognition) are necessary to stimulate better [96] Source: Publisher
performance.
Answer (A) is incorrect because it is a characteristic
Answer (D) is incorrect because the grid approach that McGregor's Theory Y and MBO managers
arrays two dimensions of managing (concern for attribute to their employees.
production and concern for people) on a 9 by 9 grid.
Answer (B) is incorrect because it is a characteristic
that McGregor's Theory Y and MBO managers
[94] Source: Publisher attribute to their employees.

Answer (A) is incorrect because the famous Answer (C) is correct. In the 1960s, Douglas
Hawthorne studies suggest that the employees' McGregor developed two theories to describe the
knowledge of their special status would improve their extremes of managers' views of human nature.
output. Theory X is a negative view of people and the way
they interact with the organization. Theory X assumes
Answer (B) is incorrect because Herzberg defines a that people dislike work, must be threatened with
motivational factor as one whose absence will not punishment to work toward organizational objectives,
require close supervision, avoid responsibility, and about MBO, setting individual goals, discussing
crave security. overall organizational goals, and obtaining the trust of
subordinates are steps that an MBO manager might
Answer (D) is incorrect because it is a characteristic take. However, a policy change as significant as
that McGregor's Theory Y and MBO managers changing from Theory X to MBO must be initiated by
attribute to their employees. verification that top management supports MBO.

Answer (D) is correct. When making a change of


[97] Source: Publisher policy as great as from Theory X to MBO,
ascertaining that top management is committed to the
Answer (A) is incorrect because, when an employee change is the essential first step. Establishing and
believes that (s)he is in control, (s)he feels more communicating organizational goals, obtaining the
responsible for achieving goals. trust of subordinates, educating employees about
MBO, assisting individuals in goal setting, ensuring
Answer (B) is correct. MBO is the process by which that employees have needed resources, conducting
a manager and his/her subordinate work together to informal reviews, making performance evaluations,
formulate the objectives and goals of the subordinate. and establishing new employee goals are additional
One of the weaknesses of MBO is that emphasis on steps an MBO manager might take.
quantitative factors may cause employees to focus on
ends rather than means. Thus, MBO may jeopardize
the quality of the organization's output. [100] Source: Publisher

Answer (C) is incorrect because employee Answer (A) is correct. There are many reasons for
participation in goal setting helps clarify goals and evaluating performance. Evaluations reinforce
motivate the employee. accomplishments, help in assessing employee
strengths and weaknesses, provide motivation, assist
Answer (D) is incorrect because an employee's trust in employee development, permit the organization to
in the team approach helps make the work assess its human resource needs, and serve as a basis
atmosphere more positive. for wage increases. Nondiscriminatory benefits are
given to everyone in the organization in equal amounts
regardless of title, pay, or achievement of objectives.
[98] Source: Publisher
Answer (B) is incorrect because it is a purpose of a
Answer (A) is incorrect because it is a characteristic performance evaluation.
that Theory X managers attribute to their employees.
Answer (C) is incorrect because it is a purpose of a
Answer (B) is incorrect because it is a characteristic performance evaluation.
that Theory X managers attribute to their employees.
Answer (D) is incorrect because it is a purpose of a
Answer (C) is incorrect because it is a characteristic performance evaluation.
that Theory X managers attribute to their employees.

Answer (D) is correct. MBO managers believe that [101] Source: Publisher
employees are committed to achieving objectives,
working hard to receive the rewards of achievement, Answer (A) is incorrect because a judgmental
and striving for self-actualization. The MBO view is evaluation is based on nonverifiable, subjective
that employees enjoy work, need little supervision, criteria.
seek responsibility, and are imaginative problem
solvers. Answer (B) is correct. The halo effect occurs when
the appraiser judges one or a few employee traits and
carries over this judgment to the evaluation of the
[99] Source: Publisher employee's other traits. The halo effect can be
positive or negative.
Answer (A) is incorrect because teaching employees
about MBO, setting individual goals, discussing Answer (C) is incorrect because projection is the
overall organizational goals, and obtaining the trust of process of attributing one's own traits to another
subordinates are steps that an MBO manager might person.
take. However, a policy change as significant as
changing from Theory X to MBO must be initiated by Answer (D) is incorrect because an objective
verification that top management supports MBO. evaluation uses verifiable, often quantitative criteria.

Answer (B) is incorrect because teaching employees


about MBO, setting individual goals, discussing [102] Source: Publisher
overall organizational goals, and obtaining the trust of
subordinates are steps that an MBO manager might Answer (A) is incorrect because an advantage of
take. However, a policy change as significant as separating appraisals from wage increases is that
changing from Theory X to MBO must be initiated by more emphasis is placed on long-term objectives and
verification that top management supports MBO. goals.

Answer (C) is incorrect because teaching employees Answer (B) is incorrect because this separation does
not deprive money of its motivational power, but it rules, and the manager has a Theory X approach.
does emphasize other rewards, such as feelings of
achievement and the recognition of superiors. Answer (C) is incorrect because this effect is likely
when the leadership style is to impose a set of strict
Answer (C) is correct. The employee may not be rules, and the manager has a Theory X approach.
motivated immediately by a good appraisal because
of the delay in receipt of any monetary reward. The Answer (D) is correct. When a manager works to
evaluation may also not be taken as seriously by the maintain a positive attitude among employees and
employee if compensation is not correlated with cooperates with them in problem solving, employees
performance. are likely to be more motivated, confident, and
creative. This leadership style should also improve
Answer (D) is incorrect because an advantage of communication and decrease absenteeism.
separating performance evaluations from
wage-increase decisions is that the employee's good
performance can be separated from the overall [106] Source: Publisher
company's bad financial performance.
Answer (A) is correct. The situational approach to
leadership (called path-goal theory) allows a manager
[103] Source: Publisher to choose one of four approaches for implementing
his/her leadership style. One of these is the directive
Answer (A) is correct. The personnel director should leader approach in which a manager provides close
tailor the discussion to the listener by describing how guidance to the employee through the use of specific
the employee's department evaluates performance rules, policies, and procedures.
and what is expected of him/her. The director should
also obtain feedback from the employee to determine Answer (B) is incorrect because this action would be
if everything is clearly understood. taken when implementing the achievement-oriented
leader approach.
Answer (B) is incorrect because this may confuse the
new employee. Answer (C) is incorrect because this action would be
taken when implementing the achievement-oriented
Answer (C) is incorrect because this may confuse the leader approach.
new employee.
Answer (D) is incorrect because this action would be
Answer (D) is incorrect because this may confuse the taken when implementing the achievement-oriented
new employee. leader approach.

[104] Source: Publisher [107] Source: Publisher

Answer (A) is incorrect because employee Answer (A) is incorrect because employee
participation is decreased by this leadership style. development is also enhanced under the directive
leader approach.
Answer (B) is correct. The characteristics of a
leadership style based on deadlines and strict rules Answer (B) is incorrect because this benefit results
include an emphasis on relatively inflexible from the directive leader approach. A structured
congruence with the organization's overall goals, environment is not a characteristic of the
perceptions related to McGregor's Theory X, and achievement-oriented leader approach.
limitations on interaction and communication among
employees. Answer (C) is incorrect because this benefit results
from the directive leader approach. Close supervision
Answer (C) is incorrect because organizational goals is not a characteristic of the achievement-oriented
are strongly emphasized, whereas individual leader approach.
employee goals are de-emphasized.
Answer (D) is correct. The benefits to the company
Answer (D) is incorrect because Theory Y is of the achievement-oriented leader approach include
optimistic about employees' motivation, ability, and greater employee confidence and commitment, more
self-discipline. In this situation, the manager's employee decision making, increased employee
perceptions are similar to Theory X, which suggests creativity, more challenging objectives, and reduced
that employees dislike work and need constant supervision for employees who work best
direction or coercion. independently.

[105] Source: Publisher [108] Source: Publisher

Answer (A) is incorrect because this effect is likely Answer (A) is correct. When an employee is aware
when the leadership style is to impose a set of strict that others at the same level are paid more because
rules, and the manager has a Theory X approach. of superior education rather than performance, the
perceived inequity is likely to have several effects;
Answer (B) is incorrect because this effect is likely e.g., (s)he is likely to become less productive, be less
when the leadership style is to impose a set of strict cooperative, have a negative effect on subordinates'
output, and be reluctant to help colleagues. should be visibly committed to the system, develop a
culture that emphasizes above-average performance,
Answer (B) is incorrect because the employee will implement a policy of following up on unacceptable
probably be reluctant to help colleagues since they performance reviews, and establish an employee
have higher salaries. appeal process for unfavorable reviews.

Answer (C) is incorrect because an employee whose Answer (C) is incorrect because a significant
leading attribute is long experience with the company difference in pay for different levels of performance
is more likely to reduce his/her productivity than to would motivate employees to improve performance.
seek possibly inferior employment elsewhere.
Answer (D) is incorrect because an employee is
Answer (D) is incorrect because an employee whose more motivated when his/her job description is
leading attribute is long experience with the company frequently reviewed and his/her opinion is heard.
is more likely to reduce his/her productivity than to
seek possibly inferior employment elsewhere.
[112] Source: CIA 0593 III-11

[109] Source: Publisher Answer (A) is incorrect because reward power is


based on a person's ability to grant benefits.
Answer (A) is incorrect because this behavior would
undermine the employee's chance for a promotion. Answer (B) is incorrect because coercive power is
rooted in the fear or threat of punishment.
Answer (B) is incorrect because this behavior would
undermine the employee's chance for a promotion. Answer (C) is correct. Referent power is based on
identification of subordinates with a superior. Thus,
Answer (C) is incorrect because this behavior would personal magnetism (charisma) may be a basis for
undermine the employee's chance for a promotion. influencing others to comply with a manager's
directives.
Answer (D) is correct. When an employee can earn a
desired salary increase only through a promotion, Answer (D) is incorrect because legitimate power is
(s)he is likely to be motivated to perform better. If based on a person's superior position.
this does not result in a promotion, (s)he will
probably look for another job.
[113] Source: CIA 0591 III-14

[110] Source: Publisher Answer (A) is incorrect because coercive power is


rooted in fear or threat of punishment.
Answer (A) is incorrect because the company should
administer a uniform system of salary increases. Answer (B) is correct. Referent power is based on
identification of subordinates with a superior. Thus,
Answer (B) is incorrect because the company could personal magnetism (charisma) may be a basis for
improve its system if it provided other compensation, influencing others to comply with a manager's
such as assisting with an employee's tuition or directives.
recognizing an employee who has given many years
of service. Answer (C) is incorrect because legitimate power is
based on formal authority or the organizational
Answer (C) is correct. A company with a position held by a leader.
compensation program that stops rewarding
above-average performance at the top of each salary Answer (D) is incorrect because reward power is
range and rewards education with higher based on a person's ability to grant benefits.
compensation could improve the system in several
ways. These include starting a bonus program for
above-average performance, refusing to hire a person [114] Source: CIA 0594 III-57
at the top of a salary range unless (s)he will be
promoted quickly, and equating experience and Answer (A) is incorrect because the rate of
education so that employees are treated fairly. absenteeism does not compare input and output.

Answer (D) is incorrect because the company would Answer (B) is incorrect because the goals of
please all employees by increasing this percentage. becoming a leading manufacturer or of increasing
market share concern effectiveness, not efficiency.

[111] Source: Publisher Answer (C) is correct. An organizational structure is


efficient if it facilitates the accomplishment of
Answer (A) is incorrect because an emphasis on organizational objectives with minimum resources and
currently attainable goals better motivates employees fewest unsought consequences. An efficient
under a merit pay system. organizational structure maximizes output for a given
amount of input. Thus, an efficiency measure
Answer (B) is correct. A merit pay system rewards compares input to output. Insurance claims
good conduct or behavior with salary increases. To processed per day relates output (claims processed)
motivate employees under this system, management to input (a day's work).
addition or availability of which will motivate
Answer (D) is incorrect because the goals of employees. Intrinsic to the work itself, these include
becoming a leading manufacturer or of increasing achievement, recognition, challenging work,
market share concern effectiveness, not efficiency. advancement, growth in the job, and responsibility.
Thus, increased planning and decision making enrich
the job and serve to increase satisfaction and
[115] Source: CIA 1193 III-11 motivation. A pay increase is not necessary to
achieve this positive effect, but a perceived
Answer (A) is incorrect because job enlargement inadequacy of compensation would act as a
provides task variety but not necessarily satisfaction dissatisfier.
and motivation.
Answer (B) is incorrect because public recognition is
Answer (B) is incorrect because job rotation is a satisfier.
similar to job enlargement and provides similar
results. Answer (C) is incorrect because poor working
conditions and poor interpersonal relations are
Answer (C) is correct. Job enrichment is an attempt dissatisfiers.
to apply the findings of Maslow, Likert, and
McGregor by allowing each worker to participate in Answer (D) is incorrect because poor working
planning and controlling. It includes encouraging conditions and poor interpersonal relations are
worker discretion and interaction among workers, dissatisfiers.
giving workers a feeling of personal responsibility,
and involving them in changes in the physical aspects
of the work environment. [118] Source: CIA 1192 III-13

Answer (D) is incorrect because job (or task) Answer (A) is correct. Frederick Herzberg's
significance is just one component of job enrichment. two-factor theory of human behavior postulates that
there are two classes of factors in the job situation.
Maintenance or hygiene factors are those whose
[116] Source: CIA 0592 III-15 presence will not especially motivate people but
whose absence will diminish performance. These
Answer (A) is incorrect because advancement is a factors are extrinsic to the work itself. They include
satisfier. supervision, working conditions, interpersonal
relations, salary, and status. Motivational factors are
Answer (B) is correct. Frederick Herzberg's those the absence of which will not diminish
two-factor theory of human behavior postulates that performance but the addition or availability of which
there are two classes of factors in the job situation. will motivate employees. Intrinsic to the work itself,
Maintenance of hygiene factors (dissatisfiers) are these include achievement, recognition, challenging
those the presence of which will not especially work, advancement, growth in the job, and
motivate people but the absence of which will responsibility.
diminish performance. These factors are extrinsic to
the work itself. They include supervision, working Answer (B) is incorrect because physiological needs
conditions, interpersonal relations, salary, and status. are at the base of Maslow's hierarchy of needs.
Motivational factors (satisfiers) are those the absence
of which will not diminish performance but the Answer (C) is incorrect because Maslow's theory is
addition or availability of which will motivate that higher needs emerge as lower needs are met.
employees. Intrinsic to the work itself, these include
achievement, recognition, challenging work, Answer (D) is incorrect because esteem and
advancement, growth in the job, and responsibility. satisfaction are almost synonymous.

Answer (C) is incorrect because challenging work is


a satisfier. [119] Source: CIA 0594 III-49

Answer (D) is incorrect because responsibility is a Answer (A) is correct. Herzberg's two-factor theory
satisfier. of human behavior postulates two classes of factors:
motivational and hygiene. Hygiene factors
(dissatisfiers) include those factors whose presence
[117] Source: CIA 0592 III-16 will not especially motivate people but whose
absence will lead to diminished motivation. These
Answer (A) is correct. Frederick Herzberg's factors are extrinsic to the work itself. They include
two-factor theory of human behavior postulates that status, interpersonal relations, and alternative work
there are two classes of factors in the job situation. schedules. Hygiene factors such as work schedules
Maintenance of hygiene factors (dissatisfiers) are need to be adequate so that workers will have little
those the presence of which will not especially dissatisfaction. The absence of motivational factors
motivate people but the absence of which will (satisfiers) will not diminish performance, but their
diminish performance. These factors are extrinsic to addition or availability will motivate employees.
the work itself. They include supervision, working Intrinsic to the work itself, they include achievement,
conditions, interpersonal relations, salary, and status. advancement, and recognition.
Motivational factors (satisfiers) are those the absence
of which will not diminish performance but the Answer (B) is incorrect because theory X assumes
that workers have to be coerced, controlled, or
threatened to achieve goals. Answer (C) is incorrect because it is more difficult to
identify proven performers from among outside
Answer (C) is incorrect because, according to equity candidates than internal candidates.
theory, individuals compare their inputs and outputs
with those of others. Answer (D) is correct. Hiring an internal candidate
can lead to social inbreeding. Many firms look to
Answer (D) is incorrect because, according to external candidates for certain jobs because they
cognitive evaluation theory, allocating extrinsic bring a fresh perspective to the organization's
rewards for behavior that had been previously problems and may have more up-to-date training or
intrinsically rewarded tends to decrease the overall education.
level of motivation.

[123] Source: Publisher


[120] Source: CIA 0594 III-48
Answer (A) is incorrect because recognition of
Answer (A) is incorrect because alternative work goal-specific performance is characteristic of a
schedules include compressed workweeks. reward system.

Answer (B) is incorrect because alternative work Answer (B) is incorrect because goal congruence is
schedules include flextime. the sharing of goals by supervisors and subordinates.

Answer (C) is correct. Job rotation is not a type of Answer (C) is incorrect because autonomy is the
alternative work schedule. Rather, it entails shifting a extent to which individuals have the authority to make
worker from one task to another. Thus, job rotation decisions.
adds to the worker's flexibility and gives a better
perspective of the total production process. Although
popular with managers, this technique may cause lack Answer (D) is correct. Motivation is the desire to
of continuity for both subordinates and superiors, and attain a specific goal (goal congruence) and the
confusion about the training process. commitment to accomplish the goal (managerial
effort). Managerial motivation is therefore a
Answer (D) is incorrect because alternative work combination of managerial effort and goal
schedules include telecommuting. congruence.

[121] Source: CIA 0591 III-11 [124] Source: CIA 0594 III-88

Answer (A) is incorrect because the inexperienced Answer (A) is correct. People with a high need for
applicant may have just finished his/her education. achievement prefer tasks of intermediate difficulty.
Thus, continuing education may not be relevant. They want success or failure to be the result of their
own actions. High achievers avoid easy tasks
Answer (B) is incorrect because the applicant is because of the lack of challenge and difficult tasks
inexperienced and thus may not have written reports because of the low probability of success.
in previous jobs.
Answer (B) is incorrect because personal
Answer (C) is correct. Sawyer's Internal Auditing responsibility is a prerequisite for recognition of
(IIA, 1988. p. 791) suggests the following questions achievement.
for inexperienced candidates:
Answer (C) is incorrect because sales positions
What is your concept of internal auditing? provide the individual autonomy, personal
How did you hear about it? responsibility, rapid feedback, and opportunity for
Why do you think you'd like it? moderate challenge needed by achievers.
What kinds of assignments would you like best?
What are your outside interests? Answer (D) is incorrect because frequent and
What are your personal goals? unambiguous feedback allows the high achiever to
determine whether (s)he is improving.
Answer (D) is incorrect because federal law prohibits
asking about marital status.
[125] Source: CIA 1194 II-30

[122] Source: CIA 1192 III-10 Answer (A) is correct. Studies have shown that
worker satisfaction does not necessarily lead to
Answer (A) is incorrect because internal promotions improved productivity. In fact, studies indicate that it
usually lead to increased motivation among is more likely that a productive worker is a happy
employees. worker.

Answer (B) is incorrect because internal promotions Answer (B) is incorrect because satisfaction is
are less expensive. The firm can avoid the expenses negatively correlated with turnover. However, level of
associated with an executive search and certain performance is a moderating factor on the
training costs. relationship. Superior performers have lower turnover
in part because management makes stronger efforts isolating the essential features of a situation is that
to retain them. important elements may be omitted.

Answer (C) is incorrect because job satisfaction is Answer (D) is incorrect because time limitations only
directly correlated with good health and longevity. serve to reduce the information available to the
decision maker and lead to decisions that are similar
Answer (D) is incorrect because a happy worker is to previous decisions.
not necessarily a more productive worker.

[129] Source: CIA 0594 III-50


[126] Source: CIA 1196 II-38
Answer (A) is incorrect because it is clearly an
Answer (A) is incorrect because values are stable advantage of teamwork.
and enduring.
Answer (B) is incorrect because it is clearly an
Answer (B) is incorrect because questioning values advantage of teamwork.
may result in their reinforcement.

Answer (C) is incorrect because values are relatively Answer (C) is correct. In a culture that strongly
fixed and change only slowly. emphasizes individual identity and competition, the
preference tends to be for a clear link between effort
Answer (D) is correct. Values are specific to each and outcome. However, teams tend to submerge
individual and involve moral and personal issues. individual identity and responsibility and therefore to
They tend to be learned in childhood from parents, blur the link between individual effort and its results.
friends, and others. Values can be modified
throughout life but ordinarily tend to stay the same. Answer (D) is incorrect because it is clearly an
advantage of teamwork.

[127] Source: CIA 0596 II-33


[130] Source: CIA 1196 II-26
Answer (A) is incorrect because personal beliefs
alone are not an appropriate basis for managerial Answer (A) is incorrect because an approach based
action. on pure power is an autocratic style of leadership, not
a supportive approach.
Answer (B) is incorrect because personal beliefs
alone are not an appropriate basis for managerial Answer (B) is incorrect because the custodial model
action. depends on material rewards for the worker. This
model is predicated on the belief that a happy worker
Answer (C) is correct. The only legitimate grounds is a productive worker.
on which the supervisor may take action is the
employee's behavior. Personal beliefs, such as those Answer (C) is correct. Supportive management
on religious and political matters, cannot be the basis techniques orient workers toward performance rather
of personnel actions. Discrimination on the basis of than obedience or happiness. The leader should have
personal beliefs could expose the organization to legal positive feelings for his/her employees and should
action. attempt to encourage participation and involvement.
This approach is effective when used with employees
Answer (D) is incorrect because personal beliefs who are motivated to work, improve themselves and
alone are not an appropriate basis for managerial their abilities, and accomplish goals.
action.
Answer (D) is incorrect because the manager's
beliefs are not sufficient. The workers must also
[128] Source: CIA 0595 II-35 believe in the system.

Answer (A) is incorrect because the most available


information may not be the best and most useful. [131] Source: CIA 1196 II-27

Answer (B) is incorrect because the limitations on Answer (A) is incorrect because the participative
rationality suggest that individuals will seek limited approach assumes that workers are positively
information that is readily available and familiar to motivated.
them, thereby reducing the effectiveness of decision
making. The auditor's report should present essential Answer (B) is incorrect because the presence of
information and recommendations as well as define dissatisfiers is not consistent with the participative
problems. approach.

Answer (C) is correct. Bounded rationality is the Answer (C) is correct. For a participative
concept that recognizes the limitations on an management approach to succeed, the parties must
individual's ability to process information. Thus, have sufficient time, the issues must be relevant to
presenting information in simpler forms assists the employees' interests, employees must have the
decision maker. However, the danger of constructing abilities (training and communication skills) to
models that attempt to manage complexity by participate, and the company culture should support
participation. Accordingly, a limitation of the each individual with a positive role model.
participative approach is that it is unlikely that all
employees are willing to participate in decision Answer (D) is incorrect because holding a lottery is
making. an intermittent positive reinforcement.

Answer (D) is incorrect because such conflicts arise


when the needs of individuals are not integrated with [135] Source: CIA 0594 III-89
the needs of the organization.
Answer (A) is incorrect because stating the
undesirable behavior clarifies for the employee the
[132] Source: CIA 1193 III-2 link between conduct and consequences.

Answer (A) is incorrect because, in an organizational Answer (B) is correct. Effective discipline requires
orientation session, employees receive formal immediate corrective action to eliminate the negative
information about objectives, the organizational chart, effects of the undesirable employee conduct and to
benefits, and procedures. establish and reinforce appropriate behavior. Delay
merely invites more serious consequences.
Answer (B) is correct. A manager's knowledge and Moreover, the punishment should be commensurate
skills are broadened by serving in a number of with the offense, and the employee should clearly
different capacities. To a significant degree, research perceive the relationship between the punishment and
shows that managers learn to manage by managing. the behavior.

Answer (C) is incorrect because role playing is Answer (C) is incorrect because, in U.S. legal
assuming different roles in different situations culture, the accused has the right to be heard in
according to the expectations of the group. his/her defense.

Answer (D) is incorrect because liaison committees Answer (D) is incorrect because focusing on the
facilitate understanding between management and offense rather than the offender is less likely to
employees. engender fear and resentment on the part of the
employee.

[133] Source: CIA 1194 II-28


[136] Source: CIA 1195 II-40
Answer (A) is incorrect because classroom training
permits the employee to learn to use the actual Answer (A) is incorrect because punishment only
job-related equipment in a training setting. leads to short-term suppression of the behavior and
may cause the staff member to avoid the manager,
Answer (B) is incorrect because videos provide the who is seen as punishing rather than helpful.
specific, technical details necessary to perform the
job and can be viewed until the employee is Answer (B) is incorrect because fixed-interval
comfortable with the material. reinforcement schedules do not clearly link
performance and rewards.
Answer (C) is incorrect because apprenticeships
place the employee with an experienced worker who Answer (C) is correct. Variable-interval schedules of
serves as a model for the understudy to observe. reinforcement lead to higher performance. Employees
are more alert because of the uncertainty involved,
Answer (D) is correct. According to Robbins and performance and reward are connected.
(Organizational Behavior, pg. 565), simulation
exercises, such as case analyses, role playing, Answer (D) is incorrect because 6 months is too long
experiential exercises, and group interaction sessions, an interval for linking performance and reward.
best serve in developing problem-solving and
interpersonal skills.
[137] Source: CIA 0596 II-36

[134] Source: CIA 0594 II-9 Answer (A) is incorrect because threats are not as
effective as positive reinforcement.
Answer (A) is incorrect because paying a bonus is a
positive reinforcement. Answer (B) is incorrect because negative comparison
with others is a demotivator.
Answer (B) is correct. Negative reinforcement
removes an unpleasant condition when the desired Answer (C) is incorrect because the auditor should
behavior occurs, whereas positive reinforcement try to provide conclusive proof that the findings are
rewards the desired behavior. Thus, attending class is correct and the conclusions are sound, but such proof
reinforced by the removal of something unpleasant, is not concerned directly with motivation.
i.e., the receipt of a written warning. Because a
warning is given after every other absence, the Answer (D) is correct. Positive reinforcement is a
reinforcement is intermittent, not continuous. behavior modification technique that provides
rewards for certain responses. It focuses on desirable
Answer (C) is incorrect because assigning a mentor is rather than undesirable behavior. Examples are the
a positive reinforcement. The firm is attempting to link awarding of merit-based salary bonuses or paying on
a sliding scale relative to production. Continuous participation, and workplace democracy.
reinforcement rewards every occurrence of a
desirable new behavior, and intermittent Answer (B) is incorrect because providing fair and
reinforcement provides occasional rewards for an equitable reward systems that are clearly linked to the
established behavior. Behavior modification theorists employees effort and performance is one of the keys
regard positive reinforcement as the most effective to motivation. However, it is merely one part of the
approach. domain of QWL.

Answer (C) is incorrect because focusing on


[138] Source: CIA 0595 III-25 employees higher-level needs relates to Maslow's
needs hierarchy, not QWL concepts.
Answer (A) is incorrect because high achievers
perform best given moderate risks. Answer (D) is incorrect because the emphasis of
QWL is not on job enrichment but on participation
Answer (B) is correct. High achievers wish to do and workplace democracy.
something better than it has been done before.
According to McClelland's theory of needs, high
achievers thrive when the job provides for personal [141] Source: CIA 1195 III-22
responsibility, feedback, and moderate risks. They
avoid very easy or very difficult tasks, and they do Answer (A) is incorrect because job rotation would
not like to succeed by chance. Accordingly, one high not adequately address the scheduling issue.
achiever should not be assigned a job when the
probability of its successful completion is very low. Answer (B) is incorrect because job enlargement
would not adequately address the scheduling issue.
Answer (C) is incorrect because high achievers
perform best given moderate risks. Answer (C) is correct. Job sharing and flextime allow
employees to adjust their work schedules and hours
Answer (D) is incorrect because high achievers to better achieve personal objectives. These
perform best given moderate risks. programs can increase worker loyalty and motivation.

Answer (D) is incorrect because job enrichment


[139] Source: CIA 1196 III-28 would not adequately address the scheduling issue.

Answer (A) is incorrect because focusing on


employees' higher-level needs in order to help them [142] Source: CIA 0596 III-33
achieve self-actualization is a recommendation based
on Maslow's hierarchy of needs that does not Answer (A) is incorrect because specific, difficult
address the job itself as a source of motivation. goals provide more motivation than generalized goals.

Answer (B) is incorrect because removing Answer (B) is incorrect because employee
dissatisfiers does not address the issue of involvement in goal setting provides better assurance
overspecialization, although it may remove some of that employees will be committed to the goals.
the obstacles to motivation.
Answer (C) is incorrect because specific, difficult
Answer (C) is incorrect because implementing an goals provide more motivation than generalized goals.
optimal organizational rewards systems and providing
extensive training to keep employees up to date do Answer (D) is correct. According to Edwin Locke's
not address the job and the issue of goal-setting theory, specific, difficult goals to which
overspecialization. the employee is committed provide the best
motivation tool. Performance improves when goals
Answer (D) is correct. Job design theories of are specific rather than general, difficult rather than
motivation specifically address the issue of easy, and self-set rather than imposed by others.
overspecialization. These theories focus on the match Feedback, especially self-generated feedback, also
between the person and the job as the key to improves performance compared with lack of
motivation. The recommendation for dealing with the feedback. Commitment to goals, that is, a
potential problems of overspecialization and boredom determination not to reduce or abandon them, and
is either to enrich the job or to move the employee to self-efficacy, that is, a belief in one's ability to
a job that provides the appropriate level of challenge. accomplish the task, are additional qualities that result
in better performance.

[140] Source: CIA 0596 III-34


[143] Source: CIA 1195 III-23
Answer (A) is correct. According to Robert Guest
(Harvard Business Review, Vol. 57, July-August Answer (A) is correct. The tasks and environment for
1979, pp. 76-77), the quality of work life (QWL) is both sets of employees are the same. Thus, the
"a process by which an organization attempts to individuals' perceptions are the key to the problem.
unlock the creative potential of its people by involving
them in decisions affecting their lives." The domain of Answer (B) is incorrect because both sets of
QWL includes pay, employee benefits, job security, employees have the same degree of autonomy.
alternative work schedules, occupational stress,
Answer (C) is incorrect because both sets of structure. The resulting trust engenders cohesiveness
employees exhibit the same skill variety. and a free exchange of information between group
members.
Answer (D) is incorrect because both sets of
employees perform the same tasks. Answer (C) is incorrect because norms are general
standards of conduct for group members. They are
important to group effectiveness, but violation of
[144] Source: CIA 1195 III-2 norms is not as immediately damaging as mistrust.

Answer (A) is incorrect because external pressure Answer (D) is incorrect because a role is a social
sometimes increases group cohesiveness and scheme for behavior in a specific position. All group
improves performance. However, these team members develop their roles as the group evolves.
members do not know each other well, and they may
perceive that the team will not respond adequately to
adversity. [147] Source: CIA 0596 III-4

Answer (B) is incorrect because group cohesiveness Answer (A) is correct. According to path-goal
has not had sufficient opportunity to develop. Starting theory, two groups of contingency factors affect the
over will be counterproductive. relationship between leadership behavior and
outcomes (performance and satisfaction):
Answer (C) is correct. As the team members work environmental factors beyond subordinates' control
and socialize with each other, cohesiveness will be (task structure, the formal authority system, and the
enhanced because of the opportunity to talk with work group) and subordinate factors. The latter
each other, to discover commonalities, and to share include the subordinate's locus of control, experience,
experiences. However, performance may or may not and perceived ability. A leadership style should be
improve as cohesiveness increases. Improvement is chosen that complements but does not duplicate the
also contingent on the group's performance norms. A factors in the environment and is consistent with
cohesive group enforces norms. Thus, if norms are subordinates' characteristics. A directive style is most
high, greater cohesiveness should result in better effective when the subordinate's locus of control is
performance. external, tasks are ambiguous or stressful, and
substantial conflict exists in the work group. Thus, a
Answer (D) is incorrect because cohesiveness will directive style is appropriate when subordinates do
probably improve over time, but the supervisor can not have high perceived ability or experience.
speed the process by encouraging social interaction.
Answer (B) is incorrect because subordinates who
are neither competent nor confident are best led using
[145] Source: CIA 0595 III-7 the directive style.

Answer (A) is incorrect because a favorable view Answer (C) is incorrect because subordinates with an
from the outside is a definition of attractiveness. internal locus of control need a leader with a
participative style.
Answer (B) is incorrect because the similarity of
members aids cohesiveness. Answer (D) is incorrect because
achievement-oriented leadership is appropriate when
Answer (C) is correct. Commitment to a group tasks are nonrepetitive and ambiguous and employee
depends on its attractiveness and cohesiveness. The competence is high.
latter quality is the tendency of members to adhere to
the group and resist outside influences. Factors that
enhance group attractiveness and cohesiveness are [148] Source: CIA 0596 III-5
prestige, status, a cooperative relationship, a high
degree of interaction, small size, the similarity of Answer (A) is correct. Directive leadership provides
members, a superior public image, and a common highest subordinate satisfaction when a team
threat in the environment. encounters substantive internal conflict. Thus,
directive leadership is the appropriate complement to
Answer (D) is incorrect because a relatively small the environmental factors. The leader should
size enhances cohesiveness. intervene to compensate for the stress and strife in the
workplace.

[146] Source: CIA 0595 III-8 Answer (B) is incorrect because supportive style is
best when tasks and authority relationships are highly
Answer (A) is incorrect because challenge and structured.
conflict within the group are threats to cohesion.
Answer (C) is incorrect because participative style is
Answer (B) is correct. Groups in the acceptance most useful when subordinates believe they control
stage of group development tend to be effective and their own destinies.
efficient. This stage is characterized by personal and
mutual understanding, tolerance of individual Answer (D) is incorrect because
differences, constructive conflict about substantive achievement-oriented leadership is appropriate when
matters, realistic expectations about group tasks are nonrepetitive and ambiguous and employee
performance, and acceptance of the authority competence is high.
because of the dispersal of accountability. Individual
decision making tends to be more conservative
[149] Source: CIA 0596 III-6 because accountability can be specifically assigned.

Answer (A) is incorrect because directive leadership Answer (B) is incorrect because a group has greater
provides highest subordinate satisfaction when a team resources of knowledge and experience than an
encounters substantive internal conflict, when tasks individual.
are ambiguous, and when subordinates' locus of
control is external. Answer (C) is incorrect because a group has a wider
diversity of views and should be able to offer a wider
Answer (B) is incorrect because supportive style is range of solutions.
best when tasks are highly structured and the
authority relationships are clear and bureaucratic. Answer (D) is incorrect because group decisions are
more likely to be accepted by those affected.
Answer (C) is correct. Participative style is most
useful when subordinates believe they control their
own destinies, that is, when they have an internal [153] Source: CIA 1196 II-34
locus of control. Such individuals may be resentful if
they are not consulted. Answer (A) is incorrect because suspending
employees is punishment.
Answer (D) is incorrect because
achievement-oriented style will increase subordinates' Answer (B) is correct. Positive reinforcement is a
expectations that high performance will result from
their best efforts. behavior modification technique that provides
rewards for certain responses. It focuses on desirable
rather than undesirable behavior. The practice of
[150] Source: CIA 0596 III-7 praising employees when the detected error rate in
their work stays below a predefined level
Answer (A) is incorrect because supportive style is demonstrates positive reinforcement.
best when tasks are highly structured and the
authority relationships are clear and bureaucratic. Answer (C) is incorrect because eliminating time
budgets is extinction, which is the elimination of
Answer (B) is correct. Supportive style is best when reinforcement that is maintaining a behavior.
tasks are highly structured and the authority
relationships are clear and bureaucratic. Answer (D) is incorrect because not requiring
employees to work overtime is negative
Answer (C) is incorrect because participative style is reinforcement, which is the elimination of something
most useful when subordinates believe they control unpleasant when a desired behavior occurs.
their own destinies.

Answer (D) is incorrect because [154] Source: CMA Samp Q1-10


achievement-oriented leadership is appropriate when
tasks are nonrepetitive and ambiguous and employee Answer (A) is incorrect because esteem is the need
competence is high. to be valued, including the need to be esteemed by
both one's self and others. These needs are satisfied
by power, prestige, status, and self-confidence.
[151] Source: CMA 0696 1-20
Answer (B) is incorrect because affiliation or
Answer (A) is incorrect because the act provides acceptance needs are the needs of people as social
protection for individuals who are aged 40 and older. beings to belong to groups and be accepted by
others.
Answer (B) is incorrect because the act provides
protection for individuals who are aged 40 and older. Answer (C) is correct. Self-actualization is the highest
need in the hierarchy. It is the desire to become what
Answer (C) is correct. The Age Discrimination in one is capable of becoming, to realize one's potential
Employment Act is designed to protect individuals and accomplish to the limit of one's ability.
aged 40 and older from employment discrimination.
The act has been amended to eliminate the Answer (D) is incorrect because security or safety
mandatory retirement age. However, certain needs are freedom from physical danger, or from loss
managerial employees are not protected by this of job, property, food, or shelter.
amendment.

Answer (D) is incorrect because the act provides [155] Source: Publisher
protection for individuals who are aged 40 and older.
Answer (A) is incorrect because each applies to
external auditors. The IMA Code of Ethics does not
[152] Source: CIA 0595 II-34 expressly use such language.

Answer (A) is correct. Group decision making is Answer (B) is correct. The preamble to the IMA
often characterized by greater acceptance of risk Code of Ethics states, "Practitioners of management
accounting and financial management have an from the top down to subordinates. The chain
obligation to the public, their profession, the communication network best summarizes these
organizations they serve, and themselves, to maintain characteristics.
the highest standards of ethical conduct. In
recognition of this obligation, the Institute of Answer (C) is incorrect because a military command
Management Accountants has promulgated the structure is mechanistic, has authority centralized at
following standards of ethical conduct for the top, and emphasizes one-way communications
practitioners of management accounting and financial from the top down to subordinates. The chain
management. Adherence to these standards, both communication network best summarizes these
domestically and internationally, is integral to characteristics.
achieving the Objectives of Management Accounting.
Practitioners of management accounting and financial Answer (D) is correct. The "chain of command" of
management shall not commit acts contrary to these the military is one in which authority is centralized at
standards nor shall they condone the commission of the top. It is autocratic or mechanistic and
such acts by others within their organizations." emphasizes the sending of one-way communications
from the top down to subordinates.
Answer (C) is incorrect because each applies to
external auditors. The IMA Code of Ethics does not
expressly use such language. [158] Source: Publisher

Answer (D) is incorrect because each applies to Answer (A) is incorrect because extensive
external auditors. The IMA Code of Ethics does not cross-communication is required to encourage
expressly use such language. creativity and innovation. The all-channel
communication network best supports
cross-communication.
[156] Source: CMA 3
Answer (B) is incorrect because extensive
Answer (A) is correct. One of the responsibilities of cross-communication is required to encourage
the financial manager/management accountant under creativity and innovation. The all-channel
the competence standard is to "maintain an communication network best supports
appropriate level of professional competence by cross-communication.
ongoing development of his/her knowledge and
skills." (S)he must also "perform professional duties in Answer (C) is correct. Extensive
accordance with relevant laws, regulations, and cross-communication is helpful in encouraging the
technical standards." The third requirement under this free flow of ideas and the resulting innovation. A
standard is to "prepare complete and clear reports group that must interact and is thus highly
and recommendations after appropriate analyses of interdependent in the performance of a complex
relevant and reliable information." common task would thus form an all-channel
network.
Answer (B) is incorrect because the confidentiality
standard concerns the financial manager/management Answer (D) is incorrect because extensive
accountant's responsibility not to disclose or use the cross-communication is required to encourage
firm's confidential information. creativity and innovation. The all-channel
communication network best supports
Answer (C) is incorrect because the integrity cross-communication.
standard pertains to conflicts of interest, refusal of
gifts, professional limitations, professional
communications, avoidance of acts discreditable to [159] Source: Publisher
the profession, and refraining from activities that
prejudice the ability to carry out duties ethically. Answer (A) is incorrect because the process also
includes symbols in which the message is encoded
Answer (D) is incorrect because objectivity is the and feedback.
fourth part of the IMA Code of Ethics. It requires
that information be communicated "fairly and Answer (B) is incorrect because effective
objectively," and that all information that could communication requires feedback.
reasonably influence users be fully disclosed.
Answer (C) is correct. The communication process
has five elements: the sender, the symbols in which
[157] Source: Publisher the message is encoded, the medium through which
the message flows, the receiver, and feedback.
Answer (A) is incorrect because a military command Because the effectiveness of communication can be
structure is mechanistic, has authority centralized at known only by its impact on the receiver and the
the top, and emphasizes one-way communications perceived change in the receiver's behavior, the
from the top down to subordinates. The chain received message is what is communicated. The sent
communication network best summarizes these message may be garbled in encoding, in transmission,
characteristics. or in the receiver's decoding.

Answer (B) is incorrect because a military command Answer (D) is incorrect because informal
structure is mechanistic, has authority centralized at communication (the grapevine) is often accurate, and
the top, and emphasizes one-way communications effective managers use it constructively.
mistakes and less clarity than other networks.

[160] Source: Publisher Answer (C) is incorrect because the chain


communication network emphasizes one-way,
Answer (A) is correct. The directions of top-down communication. Since there is no
communication are upward, downward, horizontal, mechanism for feedback, the chain results in more
and diagonal. Upward communication is from a mistakes and less clarity than other networks.
subordinate to a superior (morale surveys, grievance
procedures, interviews, conferences). It is usually Answer (D) is correct. The "chain of command" in
slower and must overcome more barriers than other the military was the model for the chain
directions of communication. Moreover, subordinates communication network stressed by classical
may be hesitant to communicate with superiors when management. Because its emphasis on one-way
interpersonal problems arise. communication from management down to
subordinates ignores feedback, it results in more
Answer (B) is incorrect because it is easier than mistakes and less clarity than other networks.
upward communication since it is initiated by the
superior.
[163] Source: Publisher
Answer (C) is incorrect because horizontal
communication is that among peers or across Answer (A) is correct. The needs, abilities, and skills
organizational channels, so status differences are less of the receiver must be taken into account by the
of a concern and the fear of criticism from a superior sender. The sender must fit the message to what the
is not involved. receiver can effectively receive or the message will be
useless.
Answer (D) is incorrect because sending a
communication diagonally may be more rapid than Answer (B) is incorrect because nonverbal cues
transmittal along a traditional vertical and then (body language) may be vital in transmitting a
horizontal path. An example would be a plant message and in receiving feedback.
manager's communicating a cost report directly to the
accounting department instead of first conveying it Answer (C) is incorrect because effective
vertically to the production vice president. communication changes behavior, not just attitudes.

Answer (D) is incorrect because the sender should


[161] Source: Publisher monitor the receiver's behavior or solicit feedback to
determine the effectiveness of the message.
Answer (A) is incorrect because management
involves more than communication skills.
[164] Source: Publisher
Answer (B) is correct. Because communication is the
process of conveying meaning or understanding from Answer (A) is incorrect because such information is
one person to another, managers must spend most of not necessary in the memorandum and may result in
their time communicating with subordinates, peers, information overload, thus clouding the reader's
and superiors. More of this communication is oral understanding of the main points of the memorandum.
and informal than written and formal. Managers
communicate lower-level results and problems Answer (B) is correct. The necessary details for a
upward, and coordinating information horizontally. memorandum announcing a mandatory meeting
Infrequently, communication may also be among include the purpose and goals of the meeting, a list of
peers or across organizational channels. who must attend, the information or reports required
for the meeting, the expected length of the meeting,
Answer (C) is incorrect because most communication and the person to contact regarding any questions or
is oral and informal. problems.

Answer (D) is incorrect because modern Answer (C) is incorrect because the memorandum
management theory stresses feedback and should cover only one subject, specifically the
multi-channel communication, not the sending of meeting it announces.
one-way messages down to subordinates.
Answer (D) is incorrect because an employee who
has not received the memorandum may not be aware
[162] Source: Publisher of it or need to receive it. In addition, (s)he could not
find out whom to contact without receiving a
Answer (A) is incorrect because the chain memorandum or reading someone else's copy.
communication network emphasizes one-way,
top-down communication. Since there is no
mechanism for feedback, the chain results in more [165] Source: Publisher
mistakes and less clarity than other networks.
Answer (A) is incorrect because approval should
Answer (B) is incorrect because the chain have been obtained prior to announcing the meeting.
communication network emphasizes one-way,
top-down communication. Since there is no Answer (B) is incorrect because the name of the
mechanism for feedback, the chain results in more person to contact about questions and problems
should have been included in the memorandum. form of communication. This communication form is
selected by the sender of the message. The device
Answer (C) is correct. There are several preliminary may be verbal, written, or nonverbal, such as facial
actions the writer/chair should take after announcing expressions or voice inflections. Computers are often
the meeting, including distributing the planned agenda, used as encoders.
providing copies of information that will prepare the
participants for the meeting, and calling the Answer (C) is incorrect because channels are media
participants to remind them of their required along which the message travels, e.g., telephone lines.
attendance.
Answer (D) is incorrect because a decoder is a
Answer (D) is incorrect because allowing each device used to translate the message into a form
person to choose the style of presentation may be meaningful to the receiver. The human auditory
preferable. In addition, the individual may not be able system is a decoder.
to distribute the information in advance.

[169] Source: Publisher


[166] Source: Publisher
Answer (A) is incorrect because expectancy is the
Answer (A) is correct. The chair could state its tendency to allow past experience to affect the
purpose and objectives at the beginning of the individual's perception of the other party.
meeting, maintain a list of assignments decided on
during the meeting, focus the discussion by giving Answer (B) is correct. Perception is the process used
direction, curtail repetition and triviality, encourage to adapt the message to the receiver during encoding
individual responses, and summarize and review the or to interpret the message from the sender when
discussion at the meeting's conclusion. decoding. Perceptual errors cause misinterpretation
of the intended message. Projection is the tendency
Answer (B) is incorrect because the chair would be for the sender to attribute his/her traits, values, and
too busy to take minutes and chair the meeting. (S)he emotions to the receiver and vice versa.
should assign this task to another participant.
Answer (C) is incorrect because the halo effect is the
Answer (C) is incorrect because the minutes for the extrapolation of a judgment about one matter to
last monthly meeting would normally be read at the other, possibly unrelated, issues.
next monthly meeting. This meeting was called for a
special purpose. Answer (D) is incorrect because stereotyping is the
attribution to another person of traits that are
Answer (D) is incorrect because the meeting will run commonly associated with a category or group to
more smoothly if a preestablished agenda is which that person belongs.
distributed to all participants and followed.

[170] Source: Publisher


[167] Source: Publisher
Answer (A) is incorrect because the sender should
Answer (A) is incorrect because the minutes, encourage feedback from the receiver through
attendance list, and documented information from the two-way communication.
meeting should be distributed to participants.
Answer (B) is incorrect because the sender encodes
Answer (B) is incorrect because it is unnecessary and the message; the receiver decodes the message.
costly to distribute such a list.
Answer (C) is incorrect because both parties should
Answer (C) is incorrect because, if at all possible, the make an effort to minimize noise.
participants should be informed of the time and
location of the follow-up meeting with plenty of time Answer (D) is correct. Perceptual errors can be
to adjust their schedules, prepare, and plan. minimized in several ways, including feedback from
the receiver concerning his/her perceptions and
Answer (D) is correct. There are several actions the interpretations of the message, understanding of the
chair should take after a meeting. A following-up of sender's perspective by the receiver, the sender's
special assignments made to individuals at the meeting sensitivity to the receiver's problems, and
is appropriate. A request for periodic reports on the implementation of a training program to improve
status of these assignments may be made or the communication skills throughout the company.
individuals can be contacted directly.

[171] Source: Publisher


[168] Source: Publisher
Answer (A) is incorrect because the perception that
Answer (A) is incorrect because a transmitter is a management values employee accomplishments is a
device used by the encoder to send the message benefit of recognition of employee achievements
toward the receiver. rather than negotiation.

Answer (B) is correct. The encoder is a device used Answer (B) is incorrect because a written
to translate the sender's message into an appropriate communication link and the reduced need for the
informal grapevine are benefits of a device such as a knowledge of the stated conclusions hinders the
periodic (e.g., monthly or quarterly) newsletter. compromise process, the company's image may be
distorted.
Answer (C) is correct. The benefits of negotiating
employee-supervisor differences include Answer (B) is correct. When public media are used
communicating both sides of an issue without as tools for resolving employee-management
litigation, recognizing employee concerns to indicate disputes, communication is hindered because the
that management values each subordinate's needs and issues are addressed to the public rather than
rights, and impartially treating tensions in the work discussed with the other party. Moreover,
environment while finding compromise solutions. compromise becomes difficult because the stated
conclusions are solidified by public opinion.
Answer (D) is incorrect because a written However, the need for union support is reduced
communication link and the reduced need for the because public pressure may help to resolve the
informal grapevine are benefits of a device such as a negotiations.
periodic (e.g., monthly or quarterly) newsletter.
Answer (C) is incorrect because, when issues are
presented to the public rather than discussed by the
[172] Source: Publisher two parties, communication is inhibited.

Answer (A) is correct. The benefits of a holiday party Answer (D) is incorrect because the pressure from
include establishing a controlled setting in which all the public to resolve the situation may eliminate the
levels of employees can interact equally, need for union support.
acknowledging that the company appreciates the
employees, and providing an opportunity for
discussions with management other than the
immediate supervisor. [175] Source: Publisher

Answer (B) is incorrect because increased employee Answer (A) is correct. The benefits of electronic
awareness of corporate objectives is more likely to communication include better control of information,
be a benefit of written communication, such as a more timely information, elimination of tedious tasks,
periodic newsletter. improvement of competitiveness due to improved
technology, standardization of procedures by
Answer (C) is incorrect because the opportunity to computer programs, assistance for strategic planning,
express concerns without repercussion is more likely and optimization of organizational resources to
to be a benefit of negotiating employee-supervisor improve productivity.
differences.
Answer (B) is incorrect because electronic
Answer (D) is incorrect because communication of communication eliminates redundant tasks.
outstanding performance is more likely to occur when
recognizing employee achievements, which may not Answer (C) is incorrect because procedures and
be done at a holiday party. operations are standardized by electronic
communication.

[173] Source: Publisher Answer (D) is incorrect because simulation is


enhanced by computerization.
Answer (A) is incorrect because when one
employee's dismissal is inadequately communicated,
the other employees are likely to perceive that their [176] Source: Publisher
jobs are also in jeopardy.
Answer (A) is incorrect because employees can
Answer (B) is incorrect because the other employees' participate in the system's design by serving on initial
feelings of inadequacy and helplessness encourage project teams and task forces.
them to unify.
Answer (B) is incorrect because employees can learn
Answer (C) is incorrect because when an employee how the system works and how to use it through their
believes that employment is in jeopardy, his/her access to systems manuals and designated user
insecurity increases. support systems.

Answer (D) is correct. When management fails to Answer (C) is incorrect because the MIS steering
adequately communicate the dismissal of an committee should handle ongoing issues and work
employee, other employees may believe that their with or be in contact with representatives of the
jobs are in jeopardy, and that they are inadequate employees.
and helpless. Speculation, mistrust, and paranoia can
be caused by the uncertainty arising from inadequate Answer (D) is correct. There are several actions a
communication. firm can take to ease the implementation of an
electronic communication system, including involving
employees in the design of the system, educating and
[174] Source: Publisher training employees in the appropriate use of the
system, creating an MIS steering committee to handle
Answer (A) is incorrect because, when public ongoing problems, and communicating information
about the system and its benefits to lessen the peers and subordinates. The informational roles
employees' perception that it is a threat. (nerve center, disseminator, spokesperson) reflect the
importance of information to organizational activity.
Managers must receive and transmit information to
[177] Source: CMA 0692 1-16 parties both within and outside the organization. The
decisional roles (entrepreneur, disturbance handler,
Answer (A) is incorrect because written resource allocator, negotiator) require managers to
communication does inhibit feedback. make choices and balance divergent interests.
Decisions involve developing strategies and
Answer (B) is incorrect because most managers do implementing them. The scanning of industry reports
spend more time involved with oral than with written to stay abreast of developments is an example of
communication. communication that fulfills the informational role of
disseminator.
Answer (C) is incorrect because written
communication does provide a permanent record of
the message. [180] Source: CMA 1292 1-28

Answer (D) is correct. Written communication is Answer (A) is correct. Noise in a communication
usually better when the message is routine and channel is an outside disruption that impedes the flow
impersonal. A non-routine message can be better of a message. It can vary from real noise, such as
communicated orally. Written communication also loud machines running and static on a phone line, to
offers the advantage of providing a permanent record disruptions such as phone calls during a face-to-face
of the message, but it inhibits immediate feedback conversation. Selective perception on the part of
because the two parties are not in direct contact. either the sender or the receiver of a message is not
Breakthroughs in electronic technology, such as noise because it is not an outside disruption.
computers that can recognize the human voice, may
in the future blur the distinction between oral and Answer (B) is incorrect because static on a phone
written communication. line is noise that might affect the quality of a
communication.

[178] Source: CMA 0692 1-21 Answer (C) is incorrect because a lost letter is a
random event that can cause a disruption in
Answer (A) is incorrect because this statement is communication.
straightforward in that it defines the technical term
labor standards. Answer (D) is incorrect because an interruption
during a conversation disrupts the communication.
Answer (B) is correct. The use of jargon leads to
communication problems when a specialist tries to
send a message to a nonspecialist. The use of the [181] Source: CMA 1292 1-29
phrase "expensed or capitalized" is a use of jargon in
that a nonaccountant might not understand the Answer (A) is incorrect because written
technical meaning of the terms. Thus, the recipient of communications are time consuming to prepare.
the message may not be able to provide a clear
answer. Answer (B) is incorrect because written
communications are not particularly difficult for most
Answer (C) is incorrect because the technical term is people.
described in layman's terms.
Answer (C) is incorrect because a permanent record
Answer (D) is incorrect because no technical is an advantage of written communication.
accounting terms are used.
Answer (D) is correct. Written communication offers
the advantage of providing a permanent record of the
[179] Source: CMA 1292 1-26 message. Written communication is also more
accurate, but it can be time consuming to prepare. Its
Answer (A) is incorrect because resolving conflict major disadvantage is that it inhibits feedback. The
involves the manager's decisional role as disturbance recipient of the message is not face-to-face with the
handler. sender.

Answer (B) is incorrect because encouraging


employee productivity involves the manager's [182] Source: CMA 1292 1-30
interpersonal role as leader.
Answer (A) is incorrect because telecommuters have
Answer (C) is incorrect because attending a tended to fall behind in their fields of specialization.
ribbon-cutting ceremony fulfills the manager's
interpersonal role as figurehead. Answer (B) is incorrect because telecommuters may
be unable to form normal manager-employee and
Answer (D) is correct. The interpersonal roles employee-employee relationships. Interaction with
(figurehead, leader, liaison) are necessary because, telecommuters poses obvious problems.
given his/her authority and status, a manager has
substantial interpersonal contacts, particularly with Answer (C) is incorrect because telecommuters
sometimes experience a loss of career opportunities
as a result of not being in the office on a day-to-day Answer (A) is incorrect because perception is the
basis. process of giving meaning to the environment, e.g., to

Answer (D) is correct. People who are computer determine the meaning of messages.
literate have in recent years begun working from their
homes via telecommunication devices. Problems Answer (B) is correct. Managers must always
include lack of reliable telephone lines, a potential consider the perceptions of a message recipient.
increase in management's work load, the loss of Recipients vary in their perception of messages
in-office contributions, a tendency to fall behind in because of background, language, education,
fields of specialization, a lack of strong working attitudes toward the sender (such as stereotyping)
relationships with other employees, a loss of career and job, etc. This variance can result in
opportunities, and inadequate socialization. The communication distortion at any stage of the
primary strength of these individuals, however, has communication process.
been their communication skills.
Answer (C) is incorrect because selectivity is the
necessary process of filtering external stimuli.
[183] Source: CMA 1293 1-13
Answer (D) is incorrect because stereotyping is a
Answer (A) is incorrect because the tone of voice way of organizing and interpreting experience.
can be a nonverbal communication. Unfortunately, it entails drawing inferences based on
inadequate information.
Answer (B) is incorrect because the way people
organize their offices can indicate a feeling of
openness or a feeling of separateness. [186] Source: CMA 1293 1-16

Answer (C) is incorrect because eye contact is Answer (A) is incorrect because basic purposes of
usually considered to be a positive communication an organization's internal communications network
tool. include establishing a common focus.

Answer (D) is correct. Nonverbal communication Answer (B) is correct. Because all managerial
consists of the nuances that accompany a verbal functions require communication, it is the secret to the
communication. For instance, the tone of a speaker's success of any manager. A manager's ability to
voice may communicate something totally different understand other people, and their ability to
from the words being spoken. Similarly, body understand the manager, are crucial to accomplishing
language (facial expressions, gestures, posture, and organizational objectives. Communication is the link
appearance) is a nonverbal communicator. Even the that ties an organization together and transforms a
positioning or nature of furniture in an office may diverse group of people into a cohesive whole. An
communicate something to a listener. A fax message, organization's internal communications network is
however, is a verbal communication to the extent that designed to facilitate decision making among
it contains words. managers, to promote goal congruence among
employees, integrate the efforts of all employees, and
build high morale and mutual trust. Informing potential
[184] Source: CMA 1293 1-14 investors about company operations is a purpose of
the external, not internal, communications network.
Answer (A) is incorrect because managers can
minimize the damage that a grapevine can cause by Answer (C) is incorrect because basic purposes of
transmitting accurate and timely information and an organization's internal communications network
maintaining open channels of communication. include aiding decision making.

Answer (B) is incorrect because attempts to eliminate Answer (D) is incorrect because basic purposes of
the grapevine are likely to strengthen it. an organization's internal communications network
include integrating employee efforts.
Answer (C) is incorrect because grapevines can exist
in several patterns. For example, one person may tell
one other person who tells one other person, etc.; [187] Source: CMA 1293 1-17
one person may tell all people in a group; individuals
may tell selected others; or individuals may tell others Answer (A) is incorrect because downward
at random. communication is from a superior to a subordinate.

Answer (D) is correct. A grapevine is the name of the Answer (B) is incorrect because upward
informal communication channel that exists in all communication is from a subordinate to a superior.
organizations. The grapevine exists wherever there
are people. Although the grapevine is usually
accurate, it can carry gossip and rumor. The effective Answer (C) is incorrect because informal
manager stays tuned into the grapevine and uses it communication operates outside the formal structural
constructively. channels of the organization; an example is the
grapevine.

[185] Source: CMA 1293 1-15 Answer (D) is correct. An interdepartmental


memorandum, as described, is a horizontal
communication in that no superior-subordinate Answer (A) is incorrect because generally accepted
relationship is involved. accounting principles concern external financial
reporting, not internal reporting.

[188] Source: CMA 1293 1-18 Answer (B) is incorrect because the Financial
Accounting Standards Board concerns external
Answer (A) is incorrect because an all-channel financial reporting, not internal reporting.
network is inappropriate. All members of this group
have no need to communicate with each other. Answer (C) is incorrect because the American
Institute of Certified Public Accountants concerns
Answer (B) is incorrect because a circle network and external financial reporting, not internal reporting.
a chain network are inappropriate. Each member of
the group must communicate with the controller. Answer (D) is correct. The responsibility for internal
reports is management's. Management may direct the
Answer (C) is incorrect because a circle network and accountant to provide a report in any format deemed
a chain network are inappropriate. Each member of suitable for the decision process. The accountant
the group must communicate with the controller. should work closely with management to make these
reports an effective communication device regarding
Answer (D) is correct. In the wheel form of the firm and its decisions.
communication network, the leader acts as a central
conduit for all communications. Because the
controller must process all payments, a wheel [192] Source: Publisher
network offers the most efficient and accurate
communication channel in these circumstances. Answer (A) is incorrect because evaluating solutions
and making the choice precede implementation,
which is an aspect of the follow-up to the decision
[189] Source: CMA 1291 3-10 choice.

Answer (A) is incorrect because a service center has Answer (B) is incorrect because defining the problem
no responsibility for developing markets or selling. and making the choice precede implementation,
which is an aspect of the follow-up to the decision
Answer (B) is incorrect because a production center choice.
is engaged in manufacturing.
Answer (C) is correct. A decision cannot be
Answer (C) is incorrect because a profit center can communicated to affected parties until it has been
choose its markets and sources of supply. made. Effective communication is vital to successful
implementation of the change resulting from the
Answer (D) is correct. A service center exists decision. Follow-up to evaluate the decision will
primarily and sometimes solely to provide specialized determine whether the decision was correct. One
support to other units within the organization. Service reason desired results may not be obtained is lack of
centers are usually operated as cost centers. effective communication.

[190] Source: CIA 0588 IV-12 Answer (D) is incorrect because identifying solutions
and making the choice precede implementation,
Answer (A) is correct. Responsibility accounting which is an aspect of the follow-up to the decision
stresses that managers should only be held choice.
responsible for factors under their control. Detailed
information is given to the lowest-level manager (the
foreman) who can control the costs. [193] Source: Publisher

Answer (B) is incorrect because the foreman, Answer (A) is incorrect because an interruption or
supervisor, and vice president would receive only distraction is noise.
summary cost data and exception reports.
Answer (B) is incorrect because an interruption or
Answer (C) is incorrect because the foreman, distraction is noise.
supervisor, vice president, and president would
receive only summary cost data and exception Answer (C) is incorrect because an interruption or
reports. The controller would not receive these distraction is noise.
reports.
Answer (D) is correct. Noise is a distraction that
Answer (D) is incorrect because the foreman, lessens the accuracy of the intended message making
supervisor, vice president, and president would it vulnerable to miscommunication. Interruption in the
receive only summary cost data and exception sender's preparation of the message or the message's
reports. The controller would not receive these trip along the channel of communication is noise.
reports. Noise also occurs when the receiver is distracted
from listening accurately to the message. The
tendency for the receiver to listen only to what (s)he
[191] Source: Publisher wants to hear is a perceptual error called selective
perception. Perceptual errors cause
misinterpretations of communications.
[197] Source: CIA 1192 III-20

[194] Source: Publisher Answer (A) is incorrect because, although the


performance of the junior auditors is addressed,
Answer (A) is incorrect because the initial capital directing the communication through the
expenditures for acquisition and set-up costs are auditor-in-charge follows normal protocol.
large.
Answer (B) is incorrect because no environmental
Answer (B) is correct. Implementation of an problems are apparent.
electronic communication system improves the
technology of a company. This increases the Answer (C) is correct. The method of communication
company's comparative advantage over smaller chosen offered no opportunity for feedback. It is
companies that cannot afford electronic likely that verbal communication in a face-to-face
communication systems. situation would have offered the best method to
communicate the desires of the supervisor. The
Answer (C) is incorrect because additional resources effectiveness of communication can be determined
must be allocated for implementation. only when the sender seeks feedback and observes
the impact of the communication on the receiver. The
Answer (D) is incorrect because no system can sender is obligated to solicit feedback to ensure that
positively ensure against data loss. Backup files and the communication process is complete. The receiver
data recovery systems are essential. should give feedback to the sender. The importance
of feedback to check the effectiveness of the
communication process indicates the limitations of
[195] Source: Publisher one-way communications (e.g., memos).

Answer (A) is incorrect because the hardware and Answer (D) is incorrect because nothing in the facts
software development tend to conflict with the users' indicates that the sender and receiver have
comfort. interpersonal problems.

Answer (B) is incorrect because employees are


resistant to and do not communicate with the MIS [198] Source: CIA 1190 III-20
department because they are usually not permitted to
participate in the system's design. Answer (A) is correct. Electronic mail is an
application of office automation. It is a
Answer (C) is correct. When an electronic computer-based message system (software) that
communication system is implemented, employees permits transfer, receipt, and storage of messages
tend to resist the change because they feel within or between computer systems via telephone
threatened, lack understanding, and usually do not lines. The "mail" consists of electronically transmitted
participate in design. They are also likely to believe messages. A user's "mailbox" is the storage allocated
that its development conflicts with their comfort. for messages. The advantages of electronic mail are
high-speed transmission, reduction of message
Answer (D) is incorrect because employees resist preparation costs, and the possibility of sending or
changing to the new system because they believe it reading messages at a convenient time. Moreover,
threatens their job security. electronic mail can be read wherever the recipient
may be, provided (s)he has access to a terminal and
a telephone link. The typical system includes the listed
[196] Source: CIA 1192 III-19 features as well as COMPOSE, DELETE, FILE,
SCAN, MOVE, RETRIEVE, etc.
Answer (A) is incorrect because the facts do not
suggest that the environment of the communication Answer (B) is incorrect because this system lacks
was poor. Read and Print capability.

Answer (B) is correct. A semantic barrier arises Answer (C) is incorrect because, of the features
when people assign different meanings to words. One listed, desktop publishing provides only EDIT and
obvious cause of a semantic barrier is the difference PRINT capabilities.
in technical training between the sender and recipient.
In these circumstances, particular care must be taken Answer (D) is incorrect because it refers to a method
in the encoding process. Only someone with a of transmission.
thorough understanding of audit or statistical jargon
would be able to understand the finding as presented.
[199] Source: CIA 0594 II-EX9
Answer (C) is incorrect because verbal
communication is a good method for presenting Answer (A) is incorrect because these methods may
preliminary findings. slow down a meeting.

Answer (D) is incorrect because nothing in the facts Answer (B) is correct. Listening entails decoding and
indicates that the timing of the communication was understanding the first message sent. The sender then
poor. becomes a listener with respect to the feedback.
Hence, listening is necessary at both ends of the
communication channel. Other aids to effective
listening are using body language to encourage the [202] Source: CIA 1191 III-13
speaker, showing appropriate emotion to signify
sympathy, understanding and correcting for one's Answer (A) is correct. Selectivity is the process of
biases, avoiding making premature judgments, and allowing only particular stimuli to form a perception.
briefly summarizing what has been said. Organization is the meaningful arrangement of
perceived stimuli. Interpretation is the process by
Answer (C) is incorrect because these methods may which different people perceive situations differently.
or may not help depending on the purpose of the
interview. Answer (B) is incorrect because sending, receiving,
and feedback are parts of the communication chain.
Answer (D) is incorrect because only paraphrasing
relates to feedback. Answer (C) is incorrect because listening, writing,
and speaking are methods of communication.

[200] Source: CIA 0594 II-46 Answer (D) is incorrect because sending is part of
the communication chain. Listening is a method of
Answer (A) is incorrect because listening tends to be communication. Psychosocial is a barrier to effective
more difficult than talking. Most people prefer to communication.
express their own ideas rather than listen.

Answer (B) is incorrect because a good listener does [203] Source: CIA 1191 III-12
not interrupt and makes smooth transitions between
listening and speaking. Answer (A) is incorrect because the order
information was repeated back correctly to the
Answer (C) is correct. Questions can communicate a sender, so it was encoded properly.
high attention level by the listener. Also, questions
asked while the speaker is talking may provide Answer (B) is incorrect because the order
needed clarifications. information was repeated back correctly to the
sender, so it was decoded properly.
Answer (D) is incorrect because using appropriate
nonverbal cues characteristic of attentive listening will Answer (C) is correct. In the communication
tend to put the speaker at ease and enhance the process, the medium is the channel through which the
communication process. communication flows. The failure in this case was
caused by the choice of a medium that did not create
a permanent record of the facts of the
[201] Source: CIA 1192 III-18 communication.

Answer (A) is incorrect because the environment Answer (D) is incorrect because the supplier's clerk
may affect the perception of a communication, but it gave accurate verbal feedback on the essentials of
has only limited impact upon selectivity, organization, the order.
and interpretation.

Answer (B) is incorrect because objectives focus [204] Source: CIA 0593 III-29
upon the purpose of the communication, whereas
selectivity, organization, and interpretation focus upon Answer (A) is correct. Noise in the communication
message receipt and interpretation (perception). channel refers to any disruption that impedes the
encoding, sending, or receipt of a message, such as
Answer (C) is incorrect because noise is a distraction being interrupted by several telephone calls while
in the message decoding process. issuing instructions.

Answer (D) is correct. Perception is the process Answer (B) is incorrect because nonverbal feedback,
through which someone gives meaning to the or body language, encompasses the facial
surrounding environment. This process is composed expressions, gestures, and posture that send various
of three subprocesses. These subprocesses serve as messages.
obstacles to proper understanding of the various
external stimuli. Selectivity is a sensory screening Answer (C) is incorrect because closure is the
process that permits one to ignore certain details in process of filling in the blanks of an incomplete
the surrounding environment. Without selectivity, an message.
individual or an organization would be overwhelmed
by stimuli. Organization is the process of ordering Answer (D) is incorrect because selectivity is the
otherwise meaningless and disorganized stimuli. process of allowing only selected stimuli to form a
Grouping, figure-ground, and closure are means of perception.
perceptual organization. Interpretation is the meaning
attributed to a given situation based on personal
experience. For example, a large rush order from a [205] Source: CIA 0594 II-26
customer will be perceived differently by a
salesperson who obtained the order and by the Answer (A) is correct. The effectiveness of
production manager who must fill it. communication can be determined only by the
sender's seeking feedback and observing the impact
of the communication on the receiver. The sender is Answer (A) is incorrect because a receiver who
obligated to solicit feedback to ensure that the understands a message may change attitude but may
communication process is complete. The receiver not necessarily change behavior.
should give feedback to the sender. The importance
of feedback to check the effectiveness of the Answer (B) is incorrect because, although the clarity
communication process indicates the limitations of of the message is a receiver perception necessary to
one-way communications (e.g., memos). understanding (believing) the message, the receiver
Effectiveness can only be measured when the sender must respond appropriately before the message is
perceives a change in the receiver's behavior. Thus, effective.
the parts manager (the sender) should have sought
and the supplier (the receiver) should have provided Answer (C) is incorrect because a receiver who
feedback. understands a message may change attitude but may
not necessarily change behavior.
Answer (B) is incorrect because the facts do not
suggest that the language used was confusing. Answer (D) is correct. The best indicator of the
effectiveness of a communication on the receiver is
Answer (C) is incorrect because the mail is an the change in the receiver's behavior in the direction
acceptable medium of transmission. requested or required by the communication. The
sender has the responsibility to solicit feedback (or
Answer (D) is incorrect because the supplier had no observe results) to determine the communication's
effectiveness.
reason to ignore (selectively screen out) an order. A
supplier's perceptual selection obviously includes
rather than excludes customer orders. [209] Source: CIA 0580 III-11

Answer (A) is incorrect because management can


[206] Source: CIA 0591 III-17 use a grapevine or informal communication network
to supplement the formal communication process, to
Answer (A) is incorrect because a meeting is correct misinformation, and to transmit information
logistically too difficult. not appropriate for formal communication channels.

Answer (B) is incorrect because a front-page Answer (B) is correct. Suppressing grapevine
message in the monthly company newsletter is not networks is difficult, if not impossible. Recognizing
timely. that the grapevine exists and can be used effectively
for several kinds of messages makes better
Answer (C) is correct. A memo is clear, concise, and managerial sense.
permanent. It can be communicated effectively to
Answer (C) is incorrect because management can
each employee and will give all employees the same use a grapevine or informal communication network
message in a timely fashion. to supplement the formal communication process, to
correct misinformation, and to transmit information
Answer (D) is incorrect because a board meeting not appropriate for formal communication channels.
does not reach employees.
Answer (D) is incorrect because management can
use a grapevine or informal communication network
[207] Source: CIA 0578 III-12 to supplement the formal communication process, to
correct misinformation, and to transmit information
Answer (A) is incorrect because this is an example of not appropriate for formal communication channels.
downward communication. Official changes in
procedures or benefits can be announced by notices
on bulletin boards. [210] Source: CIA 0580 III-19

Answer (B) is correct. Grievance actions are a formal Answer (A) is correct. Effective communication is
means of bringing employee dissatisfaction to the likely to have the least amount of distortion when the
attention of management, i.e., from the bottom sender and the receiver share similar frames of
upward. reference. If both sender and receiver understand the
symbols used to communicate and the underlying
Answer (C) is incorrect because this is an example of assumptions concerning the problem, the message
downward communication. Official changes in will be easier to write, to send, and to understand.
procedures or benefits can be announced by notices
on bulletin boards. Answer (B) is incorrect because stating a message in
general terms will not create effective communication
Answer (D) is incorrect because this is an example of if the message concerns a specific problem.
downward communication. Official changes in
procedures or benefits can be announced by notices Answer (C) is incorrect because haste can make
on bulletin boards. waste. A message cannot be effective if it is coded
too quickly or if some is lost in transit.

[208] Source: CIA 0579 III-26 Answer (D) is incorrect because the sender should
not assume that the receiver will recall all underlying change in the receiver's behavior, the received
assumptions. If both ends of the message share a message must govern the definition. The sent message
frame of reference, underlying assumptions need not may be garbled in encoding, in transmission, or in the
be spelled out, but they should not be ignored. receiver's decoding.

Answer (C) is incorrect because an organization is,


[211] Source: CIA 0582 III-11 by definition, two or more people gathered together
for a common purpose. These people agree on
Answer (A) is incorrect because selective reception organizational goals via communicating their
(perceptual defense), the tendency for people to hear objectives, and management spends the majority of
what they want or expect to hear, is filtering by the its time influencing the achievement of goals by
recipient. communicating with other members of the
organization.
Answer (B) is correct. Communication within an
organization must be clear, appropriate, and properly Answer (D) is incorrect because an organization is,
transmitted. Distortion can be unintentional (e.g., a by definition, two or more people gathered together
phone line going dead), or it may follow from for a common purpose. These people agree on
deliberate filtering either by the sender or an organizational goals via communicating their
intermediary. The auditor should watch for indications objectives, and management spends the majority of
that first-line or lower-level management is "filtering" its time influencing the achievement of goals by
out bad news or covering up irregularities. communicating with other members of the
organization.
Answer (C) is incorrect because regulating
information flow deals more with volume than
content. [214] Source: CIA 0594 II-1

Answer (D) is incorrect because selective reception Answer (A) is incorrect because trust, competence,
(perceptual defense), the tendency for people to hear objectivity, and high ethical standards are important in
what they want or expect to hear, is filtering by the changing attitudes.
recipient.
Answer (B) is correct. Presenting many different
issues in as short a time as possible will confuse the
[212] Source: CIA 0582 III-24 listener and cause the message to be lost or
disregarded. To convey a persuasive message
Answer (A) is incorrect because one of the problems effectively, the communicator should make a clear
within management is the inability of many managers presentation that focuses on the ultimate objective.
to clearly and concisely communicate ideas, The argument should be stated one idea at a time,
concepts, directives, policies, results, etc. and unrelated subjects and jumping from issue to
issue should be avoided. The presentation should
Answer (B) is correct. Because communication is the guide the recipient of the communication directly to
process of conveying meaning or understanding from the desired conclusion.
one person to another, managers must spend most of
their time communicating with subordinates, peers, Answer (C) is incorrect because effective persuasion
and superiors. They communicate organizational demands flexibility so that the arguments presented
goals and plans downward, lower-level results and have a better chance of changing the person's
problems upward, and coordinating information attitudes.
horizontally (among peers or across organizational
channels). Answer (D) is incorrect because, to convey a
persuasive message effectively, the communicator
Answer (C) is incorrect because managers spend should make a clear presentation that focuses on the
more time in oral than in written communication. ultimate objective.

Answer (D) is incorrect because good management


requires more than just effective communication. If a [215] Source: CIA 0594 II-45
manager cannot motivate subordinates, even clearly
communicated information will be ineffective to Answer (A) is incorrect because nonverbal
achieve organizational objectives. communication is heavily influenced by culture. For
example, a nod of the head may have opposite
meanings in different cultures.
[213] Source: CIA 1182 III-25
Answer (B) is correct. Nonverbal communication
Answer (A) is incorrect because communication (body language) consists of facial expressions, vocal
involves at least a sender and a receiver. intonations, posture, gestures, and appearance, and
physical distance. Thus, by its nature, nonverbal
Answer (B) is correct. The communication process communication is much less precise than verbal
has five elements: the sender, the symbols in which communication.
the message is encoded, the medium through which
the message flows, the receiver, and feedback. Since
the effectiveness of communication can be known Answer (C) is incorrect because nonverbal
only by its impact on the receiver and the perceived communication is not necessarily more truthful.
for systems development as well as understanding the
Answer (D) is incorrect because nonverbal internal control structure. A flowchart is a pictorial
communication can sometimes convey more diagram of the definition, analysis, or solution of a
information. problem in which symbols are used to represent
operations, data flow, equipment, etc. A systems
flowchart provides an overall view of the inputs,
[216] Source: CIA 0594 II-12 processes, and outputs of a system, such as a set of
interacting departments.
Answer (A) is incorrect because use of a computer
database may not be necessary. Answer (B) is incorrect because a vertical flowchart
does not highlight the interaction between
Answer (B) is correct. Proper presentation of departments.
research material requires clear documentation of
sources. The purpose is to allow others to verify the Answer (C) is incorrect because a Gantt chart is not
information and conclusions given. In financial a tool for documenting procedures. Gantt charts
reporting, documentation is especially important typically are used in industry as a method of
because of the necessity to adhere to GAAP. recording progress toward goals for employees and
machinery.
Answer (C) is incorrect because presenting all
relevant and material facts is desirable. Answer (D) is incorrect because an internal control
questionnaire does not highlight the interaction
Answer (D) is incorrect because presenting all between departments.
relevant and material facts is desirable.

[220] Source: CIA 0594 II-11


[217] Source: CIA 0594 II-13
Answer (A) is incorrect because memos are usually
Answer (A) is incorrect because the validity and short, but some can be lengthy reports.
reliability of each question are extremely important.
Bias and ambiguity must be avoided. Answer (B) is incorrect because passive voice is
often wordier and more awkward than active voice,
Answer (B) is incorrect because the validity and which is stronger and more vivid.
reliability of each question are extremely important.
Bias and ambiguity must be avoided. Answer (C) is correct. Memos can vary considerably
in tone, depending on what they are about and how
Answer (C) is correct. Many types of questions can they will be circulated. Some are quite formal, while
be used. Questions can be multiple-choice, others are informal, depending on many factors (i.e.,
checklists, fill-in-the-blank, essay, Likert scales, items message, circulation, etc.).
(options indicating degrees of agreement or
disagreement), etc. Answer (D) is incorrect because memos should have
the characteristics of good writing. They should avoid
Answer (D) is incorrect because questions can be unnecessary verbiage, and readers should grasp the
multiple-choice, checklists, fill-in-the-blank, essay, meaning quickly and easily.
Likert scales, items (options indicating degrees of
agreement or disagreement), etc.
[221] Source: CIA 0594 II-EX10

[218] Source: CIA 0594 II-16 Answer (A) is incorrect because the firm's only
interest is in maintaining a good relationship with a
Answer (A) is incorrect because a checklist may omit long-term supplier. Without important substantive
factors the importance of which could not be goals, there is no basis for collaboration.
foreseen.
Answer (B) is incorrect because the desire to
Answer (B) is incorrect because each item will not be maintain good relations with the supplier makes
of equal significance. competition inappropriate.

Answer (C) is correct. Checklists increase the Answer (C) is correct. The supplier should be
uniformity of data acquisition. They ensure that a allowed to take the lead in making proposals. The
standard approach to assessing audit risk is taken firm should go along with anything reasonable.
and minimize the possibility of omitting consideration
of factors that can be anticipated. Answer (D) is incorrect because the supplier needs
the negotiations, so they should not be avoided.
Answer (D) is incorrect because a checklist does not
substitute for the sound professional judgment needed
to understand the process of assessing audit risk. [222] Source: CIA 0594 II-EX11

Answer (A) is correct. Distributive bargaining occurs


[219] Source: CIA 0594 II-43 in zero-sum conditions; that is, what one side gains,
the other loses. The parties are in fundamental
Answer (A) is correct. Flowcharting is a useful tool opposition to each other, and their relationship tends
to be short-term. [226] Source: Publisher

Answer (B) is incorrect because both parties may Answer (A) is incorrect because encouraging
gain in integrative bargaining. employee productivity involves the manager's
interpersonal role.
Answer (C) is incorrect because attitudes are not
substantive here. Answer (B) is correct. The decisional role requires
managers to make choices and balance differing
Answer (D) is incorrect because intraorganizational interests. Therefore, resolving a conflict between two
bargaining is done through representatives. subordinates falls into the decisional category.

Answer (C) is incorrect because scanning industry


reports to stay current fulfills the informational role.
[223] Source: Publisher
Answer (D) is incorrect because attending a
Answer (A) is incorrect because written ribbon-cutting ceremony fulfills the manager's
communication tends to be more accurate than oral interpersonal role as figurehead.
communication.

Answer (B) is correct. Oral communication promotes [227] Source: Publisher


immediate feedback so that the sender knows that
the message has been clearly received by the Answer (A) is incorrect because projection is the
receiver. tendency for the sender to attribute his/her traits,
values, and emotions to the receiver and vice versa.
Answer (C) is incorrect because although oral
communication is less time-consuming than written Answer (B) is correct. Perceptual errors cause
communication, immediate feedback is a greater misinterpretation of the intended message. The halo
advantage. effect is the extrapolation of a judgment about one
matter to other, possibly unrelated issues.
Answer (D) is incorrect because providing a
permanent record is advantageous. Answer (C) is incorrect because expectancy is the
tendency to allow past experience to influence the
individual's perception of the other party.
[224] Source: Publisher
Answer (D) is incorrect because stereotyping is the
Answer (A) is incorrect because organizational status attribution to another person of traits which are
and power differences are characteristics that may commonly associated with a group to which that
inhibit effective communication. person belongs.

Answer (B) is incorrect because lack of formal


channels is a characteristic that may inhibit effective [228] Source: Publisher
communication.
Answer (A) is incorrect because encoding and
Answer (C) is correct. Listening problems are decoding are parts of the communication process that
personal characteristics, not organizational can be disrupted by noise.
characteristics that may cause communication
problems. Answer (B) is incorrect because sending is a part of
the communication process that can be impeded by
Answer (D) is incorrect because departmental needs noise.
and goals are characteristics that may inhibit effective
communication. Answer (C) is correct. Interpretation is a subprocess
of perception. Noise is a disruption that impedes the
communication process.
[225] Source: Publisher
Answer (D) is incorrect because encoding and
Answer (A) is incorrect because managers spend decoding are parts of the communication process that
more time communicating orally than they do can be disrupted by noise.
communicating in writing.

Answer (B) is incorrect because written [229] Source: Publisher


communication inhibits feedback.
Answer (A) is incorrect because selectivity is a
Answer (C) is correct. Breakthroughs in electronic perception subprocess by which one screens out
technology may lead to oral messages being recorded certain stimuli to focus on details. Without selectivity,
and subsequently written out. one would be overwhelmed by sensory overload.

Answer (D) is incorrect because the grapevine Answer (B) is incorrect because organization is a
provides accurate information more often than not. perception subprocess by which disorganized stimuli

are grouped to give meaning to otherwise


meaningless information. accounting department and sent to production
departments is a formal horizontal communication
Answer (C) is correct. Perception is the process because it occurs among peers laterally through an
through which someone gives meaning to the officially established channel.
surrounding environment. Perception consists of three
subprocesses: selectivity, organization, and
interpretation. Objectives focus upon the purpose of [233] Source: CMA 0695 1-26
communication rather than the receipt and
interpretation (perception). Answer (A) is incorrect because no communication
among peers occurred.
Answer (D) is incorrect because interpretation is a
perception subprocess by which meaning is given to a Answer (B) is incorrect because both forms of
set of stimuli based on the individual's experience. communication were within the formal organizational
structure.

[230] Source: Publisher Answer (C) is incorrect because both forms of


communication were within the formal organizational
Answer (A) is correct. The communication process structure.
has five elements: the sender, the symbols in which
the message is encoded, the medium through which Answer (D) is correct. Communication can be
the message is sent, the receiver, and feedback. downward (from superior to subordinate), upward
(from subordinate to superior), or horizontal (from
Answer (B) is incorrect because two directional one peer to another). Bell's communication of
communication is usually most effective. variance reports to his subordinates is downward
communication. The subordinates' replies are upward
Answer (C) is incorrect because communication is communications.
the message received.

Answer (D) is incorrect because effective managers [234] Source: CMA 0695 1-27
do use informal communications.
Answer (A) is incorrect because input for the yearly
budget is a form of communication that is both
[231] Source: Publisher generated internally and received by a person within
the organization.
Answer (A) is incorrect because inattention or
disinterest in the message is an example of Answer (B) is correct. A formal communication is
nonreception in which the receiver fails to receive any conducted through the formal structure of the
communication. organization. Informal communication operates
outside of officially established channels. An internal
Answer (B) is correct. If body language or tone of communication is one that is both generated and
voice send a message different from the spoken received within the organization. An environmental
words, the receiver will not be clear about the impact statement is generated within the organization,
meaning of the message. but the recipient (a governmental body) is outside the
organization.
Answer (C) is incorrect because gesturing to
someone who cannot see the gesture is an example of Answer (C) is incorrect because expense reports are
faulty channel selection. a form of communication that is both generated
internally and received by a person within the
Answer (D) is incorrect because the sender's dislike organization.
of the receiver is an example of interpersonal
problems. Answer (D) is incorrect because safety bulletins are a
form of communication that is both generated

[232] Source: CMA 0695 1-25 internally and received by a person within the
organization.
Answer (A) is incorrect because downward
communication is from a superior to a subordinate.
[235] Source: CMA 0695 1-28
Answer (B) is incorrect because hierarchical
communication is either upward or downward. Answer (A) is incorrect because communication can
be unconscious, e.g., body language.
Answer (C) is incorrect because informal
communication operates outside of formal structural Answer (B) is incorrect because accounting terms
channels; a grapevine is an example. can mean different things to different people; that is
why some companies use an accounting manual to
Answer (D) is correct. Communication can be promote consistent treatment of similar items.
downward (from superior to subordinate), upward
(from subordinate to superior), or horizontal (from Answer (C) is incorrect because accounting reports
one peer to another). It may also be formal or can add to information overload, particularly for
informal. A variance report prepared in the people who do not understand them.
example of the passive voice because the verb ("will
Answer (D) is correct. Communication between be conducted") is passive.
departments is sometimes affected by the level of
functional specialization within those departments. Answer (B) is incorrect because the verb ("make") is
Noise of any sort can inhibit communication, and that active.
noise can be physical or nonphysical. An example of
nonphysical noise is the difference in expertise Answer (C) is incorrect because the verb ("reports")
between the sender and the recipient. A sender must is active.
establish a climate that encourages the elimination of
interpersonal barriers to communication. Answer (D) is incorrect because the verb ("cannot
recommend") is active.

[236] Source: CMA 0695 1-29


[239] Source: CIA 0594 II-49
Answer (A) is correct. Formal communication is
conducted through the officially established structure Answer (A) is incorrect because "make an
of the organization. Informal communication operates acquisition" should be replaced by "acquire."

outside officially established structural channels. The Answer (B) is incorrect because the better sentence
grapevine is an example. Although the grapevine is is "The security department eliminated overtime."
usually accurate, it can also carry gossip and rumor.
Answer (C) is incorrect because "provide assistance
Answer (B) is incorrect because variance analysis is a to" should be replaced by "assist."
formal means of communication.
Answer (D) is correct. This answer is correct
Answer (C) is incorrect because the performance because it is the only sentence without a concealed
evaluation system is a formal means of verb. Each of the other sentences includes a wordy
communication. phrase in which a noun effectively replaces the verb.

Answer (D) is incorrect because a budget is a formal


means of communication. [240] Source: CIA 1196 II-19

Answer (A) is correct. Diffusion temporarily leaves


[237] Source: CMA 0695 1-30 the conflict unresolved. Smoothing (downplaying
differences and emphasizing common interests) and
Answer (A) is incorrect because interdepartmental compromise (requiring each party to make
task forces are an excellent means of overcoming concessions) are diffusion approaches to conflict
communication problems between departments. management. The disadvantage is that the underlying
problems remain unresolved while the less
Answer (B) is incorrect because cross training and controversial issues are being addressed first.
job rotation are an excellent means of overcoming
communication problems between departments. Answer (B) is incorrect because differences are
downplayed while using a diffusion approach.
Answer (C) is incorrect because organization-wide
social events are an excellent means of overcoming Answer (C) is incorrect because diffusion addresses
communication problems between departments. issues of conflict.

Answer (D) is correct. Horizontal communication Answer (D) is incorrect because directly addressing
between departments is often subject to more the conflict is a confrontational approach.
problems than either upward or downward
communication within a single department. Examples
of these problems include the use of technical terms [241] Source: CIA 1194 II-25
understood by only one side in the communication
process, faulty channel selection, and interpersonal Answer (A) is correct. Good business writing style is
problems. To overcome these difficulties, the sender concise, clear, coherent, correct, credible, natural,
must encode the message in the context of the positive, interesting, and readable. It treats receivers
receiver's perceptions; that is, the sender must with respect by sending a courteous message. It is
understand the receiver. The sender must establish an also suitable to the medium of presentation and
interpersonal climate that encourages the elimination delivery. Good business writing style provides clear,
of interpersonal barriers to communication. A developed ideas but avoids overstatement. It also
performance appraisal prepared by the accountant's conveys the message unobtrusively. The writing
supervisor will not help the accountant understand the should not call attention to itself.
perceptions of people in other departments.
Answer (B) is incorrect because treating all receivers
with respect is a characteristic of good business style.
[238] Source: CIA 0594 II-48
Answer (C) is incorrect because using a suitable
Answer (A) is correct. In a sentence using the writing style for the method of presentation and
passive voice, the grammatical subject is also the delivery is a characteristic of good business style.
object of the verb's action. This sentence is an
Answer (D) is incorrect because developing ideas Answer (D) is incorrect because the action matched
the committee's understanding of the message.
without overstating is a characteristic of good
business style.
[245] Source: CIA 1196 II-31

[242] Source: CIA 1195 II-39 Answer (A) is incorrect because no formal
communication was sent by management.
Answer (A) is correct. A typical sequence of
arguments in a persuasive message places strong Answer (B) is incorrect because no formal
arguments first and last, with weaker arguments and communication was sent by management.
refutations of opposing arguments in the middle. The
reasons for this ordering are the primary and recency Answer (C) is correct. Management's lack of formal
principles. People tend to remember best what they communication regarding possible downsizing caused
read or hear first and last. The middle section the employees to draw their own negative
includes weaker arguments. It also is the appropriate conclusions based on a manager's actions.
location of counterarguments. Selective choice of the Management should formally communicate the
opponent's arguments that can be successfully reasons for eliminating the use of temporary
rebutted can be very effective. However, mention of employees or refute the rumor about downsizing.
the opponent's arguments that cannot be refuted will
weaken the persuader's case. Answer (D) is incorrect because no formal
communication was sent by management.
Answer (B) is incorrect because attacking strong
arguments may help the opponent. Refuting weaker
arguments shows that the opposing position has [246] Source: CIA 0593 III-29
weaknesses.
Answer (A) is correct. Noise in the communication
Answer (C) is incorrect because an attack on the channel refers to any disruption that impedes the
character of the opponent is not a rational business encoding, sending, or receipt of a message, such as
strategy. It may hurt the persuader more than it will being interrupted by several telephone calls while
help. issuing instructions.

Answer (D) is incorrect because the last part of the Answer (B) is incorrect because nonverbal feedback,
message should move readers or listeners to action. It or body language, encompasses the facial
is a part of the message the audience will remember. expressions, gestures, and posture that send various
messages.

[243] Source: CIA 0592 III-19 Answer (C) is incorrect because semantics is the
study of meaning in words.
Answer (A) is incorrect because the message was
received exactly as transmitted. Answer (D) is incorrect because closure is the
process of filling in the blanks of an incomplete
Answer (B) is incorrect because sender had the message.
correct perception of the message as it was actually
encoded.
[247] Source: CIA 0593 III-27
Answer (C) is correct. Encoding is the sender's
packaging of an idea for better understanding. It Answer (A) is incorrect because the buyer will
entails translating the message into symbols that can readily understand that (s)he has been fired.
be transmitted through the chosen medium of
communication and then decoded by the recipient. In Answer (B) is correct. In the communications
this example, the sender's wording of the message process, the medium is the channel through which the
was misleading. communication flows. The defect in this case was the
channel chosen to inform the employee of his/her loss
Answer (D) is incorrect because no transmission of job. The supervisor should have spoken directly
errors occurred. with the employee.

Answer (C) is incorrect because the buyer's


[244] Source: CIA 0592 III-20 supervisor should have conveyed the news of the
termination.
Answer (A) is incorrect because words were
properly decoded. Answer (D) is incorrect because there is no noise
(interference) in the communication channel.
Answer (B) is incorrect because the medium was
capable of completing the exchange.
[248] Source: CIA 0593 III-28
Answer (C) is correct. Because of faulty encoding,
the message was open to two different Answer (A) is incorrect because, although the
interpretations. The committee chose the wrong one. message should also be routed through the personnel
department, the notification of a termination
appropriately comes from the direct superior.
[251] Source: CIA 0594 II-EX9
Answer (B) is incorrect because the memorandum
included the reasons for the termination. Answer (A) is incorrect because these methods may
slow down a meeting.
Answer (C) is correct. The communications process
has five elements: (1) the sender of the message, (2) Answer (B) is correct. Listening entails decoding and
symbols used to encode the message, (3) the medium understanding the first message sent. The sender then
chosen to send the message, (4) the receiver of the becomes a listener with respect to the feedback.
message, and (5) feedback acknowledging Hence, listening is necessary at both ends of the
interpretation of the message by the receiver. communication channel. Other aids to effective
Because the supervisor and buyer have conflicting listening are using body language to encourage the
vacation schedules, no possibility exists for feedback, speaker, showing appropriate emotion to signify
and the buyer is not referred to anyone else in the sympathy, understanding and correcting for one's
organization for any additional information. biases, avoiding making premature judgments, and
briefly summarizing what has been said.
Answer (D) is incorrect because communication of a
direct job-related impact should be transmitted by the Answer (C) is incorrect because these methods may
employee's immediate superior. or may not help depending on the purpose of the
interview.

[249] Source: CIA 1196 II-33 Answer (D) is incorrect because only paraphrasing
relates to feedback.
Answer (A) is correct. The conflict management
technique that involves face-to-face meetings is
problem solving. Problem solving is a means of [252] Source: CIA 1196 II-20
confronting the conflict and removing its causes. The
emphasis is on facts and solutions, not personalities Answer (A) is correct. Concentrating on what the
and assignment of blame. speaker is saying is critical to effective listening. This
result is best achieved by resisting internal and
Answer (B) is incorrect because expansion of external distractions. Physical distractions such as
resources addresses conflicts that arise from scarcity. noise, a tendency to be overly aware of the speaker's
physical and other differences from the listener,
Answer (C) is incorrect because the auditor is not focusing on interesting details at the expense of major
using formal authority. points, or emotional reactions to a statement with
which the listener disagrees should be avoided.
Answer (D) is incorrect because the auditor is not
using behavioral techniques to change attitudes and Answer (B) is incorrect because, given that a person
behavior. listens faster than a speaker talks, (s)he can review
the key concepts silently without waiting for the
speaker to conclude. This process helps the listener
[250] Source: CIA 0596 II-3 remember them better without notes.

Answer (A) is incorrect because trust, competence, Answer (C) is incorrect because seemingly unrelated
objectivity, and high ethical standards are important in information may be important.
changing attitudes.
Answer (D) is incorrect because the listener should
Answer (B) is correct. Presenting many different concentrate on the information while listening. Later,
issues in as short a time as possible will confuse the that person can allow for bias on both the listener's
listener and cause the message to be lost or part and the speaker's part.
disregarded. To convey a persuasive message
effectively, the communicator should make a clear
presentation that focuses on the ultimate objective. [253] Source: CIA 0596 II-15
The argument should be stated one idea at a time,
and unrelated subjects and jumping from issue to Answer (A) is incorrect because looking away is
issue should be avoided. The presentation should discouraging.
guide the recipient of the communication directly to
the desired conclusion. Answer (B) is incorrect because interruptions devalue
the speaker and the speaker's message.
Answer (C) is incorrect because, although rational
and objective evidence is important, one also should Answer (C) is correct. An effective listener enhances
try to understand a person's loves, hates, fears, and the communication process by sending appropriate
frustrations. This information can then be used to nonverbal signals to the speaker. Thus, even though a
design what to say and how to say it to effectively person can probably listen and do some routine
change the person's attitudes. work, a listener who wishes to convey a positive and
encouraging message should stop other activities and
Answer (D) is incorrect because effective persuasion focus complete attention on the speaker.
demands flexibility so that arguments presented have
a better chance of changing the person's attitudes. Answer (D) is incorrect because complete silence
may appear disapproving.
Answer (B) is incorrect because the physical distance
[254] Source: CIA 0596 II-19 between the sender and the receiver and the facial
expressions used when speaking are also nonverbal
Answer (A) is incorrect because the most effective signals.
meeting participants come to meetings prepared. The
agenda and other materials should be read in Answer (C) is incorrect because the speaker's
advance. unconscious actions are part of nonverbal
communication.
Answer (B) is incorrect because, unless the speaker
is certain of others' opinions (or is the most powerful Answer (D) is correct. Physical distance and
person in the organization), (s)he should not commit positioning convey many nonverbal messages that
to a position until the degree of support for that view depend on cultural differences. For example,
can be estimated. Americans tend to prefer a large personal space.
Facial expressions provide almost limitless variations
Answer (C) is incorrect because ideas should be of meaning thanks to the dozens of facial muscles and
researched in advance of the meeting so that the the possibilities created by different contexts,
participant appears to be prepared and productive. cultures, and individual personalities. In addition to
facial expressions, other unconscious actions of the
Answer (D) is correct. Analyzing the audience assists speaker affect the message sent. They include
a speaker to gather the right information for the gestures, posture, movement, touch, mode of dress,
meeting. Moreover, understanding the other surroundings, and voice characteristics.
participants' opinions and needs enables the speaker
to express his/her ideas in the way best calculated to
be persuasive. [258] Source: CIA 0596 III-17

Answer (A) is incorrect because a mediator is a


[255] Source: CIA 0594 II-EX10 neutral third party who facilitates a negotiated solution
by using persuasion and offering solutions. However,
Answer (A) is incorrect because the firm's only the mediator has no authority to make a decision.
interest is in maintaining a good relationship with a
long-term supplier. Without important substantive Answer (B) is correct. An arbitrator has the authority
goals, there is no basis for collaboration. to impose an agreement. Arbitration may be
requested by the parties or may be imposed by law
Answer (B) is incorrect because the desire to or by the terms of a contract.
maintain good relations with the supplier makes
competition inappropriate. Answer (C) is incorrect because a consultant is
skilled in facilitation and communication skills but
Answer (C) is correct. The supplier should be does not have authority to make a decision. A
allowed to take the lead in making proposals. The consultant helps improve relations between the two
firm should go along with anything reasonable. disagreeing parties but does not offer specific
solutions.
Answer (D) is incorrect because the supplier needs
the negotiations, so they should not be avoided. Answer (D) is incorrect because a conciliator
provides an informal communication link between the
two parties but does not have authority to make a
[256] Source: CIA 1194 II-26 decision.

Answer (A) is incorrect because superordinate goals


are shared goals that can be achieved only through [259] Source: CIA 1196 III-11
cooperation.
Answer (A) is incorrect because, when using a
Answer (B) is correct. Smoothing is a conflict distributive-bargaining approach, the manager should
resolution technique in which differences are negotiate a price that both the manager and the
deemphasized and common interests of the parties supplier can agree on.
are emphasized. It has the disadvantage of not solving
the underlying problems that created the conflict. Answer (B) is incorrect because a mediator is not
used when the distributive-bargaining approach is
Answer (C) is incorrect because problem solving used to resolve a conflict.
involves identifying and correcting the source of the
conflict. Answer (C) is correct. When using a
distributive-bargaining approach, the negotiator
Answer (D) is incorrect because compromise operates with a maximum desired result (target point)
requires each party to give up something. and a minimum acceptable result (resistance point) in
mind. If the ranges of feasible outcomes (aspiration
ranges) overlap, an agreement is possible.
[257] Source: CIA 0596 II-18
Answer (D) is incorrect because the manager should
Answer (A) is incorrect because e-mail is electronic not reveal the resistance point (the minimum
communication, not a nonverbal communication. acceptable result). Instead, (s)he should negotiate to
induce the supplier to agree to an amount closer to superordinate goals is a conflict resolution technique,
the target point. not a conflict trigger.

Answer (C) is incorrect because personality clashes


[260] Source: CIA 1196 III-12 between an auditor and auditee can destroy the
auditor's usefulness if allowed to continue.
Answer (A) is correct. The best approach to
negotiating a large purchase, assuming that both Answer (D) is incorrect because some auditees may
parties follow the same approach, is to review not respect a staff auditor if a significant disparity in
previous settlements, demands, and concessions to organization status exists.
determine what can be achieved. The history of past
practices and interactions tends to define current
standards of fairness in negotiations. [263] Source: CIA 0595 III-4

Answer (A) is incorrect because authoritative


Answer (B) is incorrect because research indicates command is a conflict resolution technique.
that personality traits have no material direct effect on
the outcome of negotiation. Answer (B) is correct. The interactionist view is that
conflict may be constructive as well as destructive
Answer (C) is incorrect because negotiators should because it encourages self-criticism, creativity, and
be prepared. They should know what they wish to necessary change. Accordingly, managers may
gain and what their resistance point is. decide to stimulate controlled conflict. Techniques for
this purpose may include ambiguous or threatening
Answer (D) is incorrect because a skilled negotiator communications; hiring outsiders with different values,
asks many questions, is a good listener, is not managerial styles, attitudes, and backgrounds;
defensive, focuses arguments well, and avoids designating an individual to argue against the majority
irritating the opponent. opinions of the group; and restructuring the
organization to disrupt the status quo.

[261] Source: CIA 1196 III-9 Answer (C) is incorrect because the expansion of
resources is a conflict resolution technique.
Answer (A) is correct. Accommodating is a conflict
handling intention. The dimensions of conflict handling Answer (D) is incorrect because the creation of
intentions are assertiveness and cooperation. An superordinate goals are a conflict resolution
intention is what mediates between one's actual technique.
behavior and one's emotions and perceptions.
Accommodating entails placing another person's
interests above one's own. It represents the minimum [264] Source: CIA 0596 III-10
of assertiveness and the maximum of cooperation.
For example, the seller should accommodate the Answer (A) is correct. Problem solving is a means of
customer by providing the product the customer confronting the conflict and removing its causes. The
wants. emphasis is on facts and solutions, not personalities
and assignment of blame. The disadvantage is that
Answer (B) is incorrect because, in a compromise, problem solving takes time. In this situation, a
both parties give up something to reach accord. It three-party negotiation stressing a win-win attitude is
represents moderate assertiveness and cooperation. appropriate. Negotiation should be feasible because
the parties have overlapping interests.
Answer (C) is incorrect because, in a competitive
situation, each party seeks to satisfy his/her own Answer (B) is incorrect because forcing does not
needs without regard to the other. It represents the resolve the basis for the conflict and can cause
maximum of assertiveness and the minimum of resentment to grow.
cooperation.
Answer (C) is incorrect because superordinate goals
Answer (D) is incorrect because challenging is not a can be achieved only by bringing conflicting parties
conflict handling intention. together. This disagreement is over the allocation of
scarce resources, office space. One party may not be
inclined to cooperate unless the supervisor requires
[262] Source: CIA 0595 III-12 some form of compromise.

Answer (A) is incorrect because communication Answer (D) is incorrect because avoidance merely
breakdowns are conflict triggers. Two-way postpones the inevitable face-to-face resolution of
communication is a complex process that is fraught this disagreement.
with maintenance problems.

Answer (B) is correct. Conflict triggers include [265] Source: CIA 1195 III-8
ambiguous jurisdictions (unclear job boundaries);
competition for scarce resources; status differentials; Answer (A) is incorrect because emphasizing
time pressures; personality clashes; unreasonable common ground may resolve the conflict but does not
standards, rules, etc.; communication breakdowns; allow each party to get what (s)he wants.
and unrealized expectations. However, an appeal to
Answer (B) is incorrect because altering attitudes and promotes cooperative, positive attitudes that transfer
behaviors that cause conflict does not create a to other organizational behaviors. Hence, optimizing
win-win situation. may be worth the expenditure of more resources than
other strategies because it improves the future
Answer (C) is incorrect because compromise forces relationship of the parties.
each side to give up something of value.
Answer (C) is incorrect because, to optimize, people
Answer (D) is correct. Expanding the pool of scarce need to think outside of established habits to find new
resources, in this case, the money available for capital benefits to divide.
projects, permits both managers to achieve his/her
objectives without having to give up anything of value. Answer (D) is incorrect because optimizing takes
Thus, each side wins. more time and energy than other conflict resolution
strategies.

[266] Source: CIA 1195 II-16


[269] Source: CIA 0596 III-8
Answer (A) is incorrect because accommodation is a
conflict resolution technique. Answer (A) is incorrect because not all conflict is
dysfunctional.
Answer (B) is correct. Competition is a
conflict-handling intention characterized by Answer (B) is incorrect because some conflict is
considerable assertiveness (the degree to which the dysfunctional.
party seeks to achieve his/her goals) and a low
degree of cooperativeness (the degree to which the Answer (C) is incorrect because ignoring the problem
party attempts to satisfy the concerns of others). will not solve it.
Encouraging competition stimulates conflict.
Answer (D) is correct. The interactionist view is that
Answer (C) is incorrect because altering the conflict may be constructive as well as destructive
structural variables is a conflict resolution technique. because it encourages self-criticism, creativity, and
necessary change. Accordingly, managers may
Answer (D) is incorrect because compromise is a decide to stimulate controlled conflict. Techniques for
conflict resolution technique. this purpose may include ambiguous or threatening
communications; hiring outsiders with different values,
managerial styles, attitudes, and backgrounds;
[267] Source: CIA 0594 III-81 designating an individual to argue against the majority
opinions of the group; and restructuring the
Answer (A) is incorrect because conflict triggers organization to disrupt the status quo. Thus, bringing
include ambiguous jurisdictions (unclear job in outside managers may stimulate conflict but may
boundaries); time pressure; personality clashes; also eliminate complacency and improve creativity.
unreasonable standards, rules, etc.; communication
breakdowns; and unrealized expectations.
[270] Source: CIA 0595 II-31
Answer (B) is incorrect because conflict triggers
include competition for scarce resources; time Answer (A) is incorrect because whether conflict is
pressure; personality clashes; unreasonable functional depends on many variables. Hence,
standards, rules, etc.; communication breakdowns; generalizations about the effect of organizational
and unrealized expectations. structure are untenable. However, one hypothesis is
that conflict is more likely to be constructive when
Answer (C) is incorrect because conflict triggers creative or unstructured decisions must be made.
include status differentials; time pressure; personality
clashes; unreasonable standards, rules, etc.; Answer (B) is incorrect because conflict avoidance
communication breakdowns; and unrealized may be necessary in the short run but does not
expectations. resolve the underlying problem. Removing the causes
of dysfunctional conflict is the best long-term
Answer (D) is correct. An appeal to superordinate approach.
goals is a means of resolving conflict. Superordinate
goals are overriding, common goals of the parties. Answer (C) is correct. The interactionist view holds
The conflict can be resolved if the individuals involved that constructive or functional conflict is necessary for
understand that it is preventing them from achieving an organization to perform effectively. Functional
more important, mutually held goals. conflict fosters self-criticism, innovation, and the
ability to respond successfully to changing
circumstances. It should be promoted, not
[268] Source: CIA 0596 II-22 suppressed.

Answer (A) is incorrect because optimizing can Answer (D) is incorrect because conflict that points
usually discover more benefits to divide. out differences in goals and objectives facilitates
discussion and eventual growth and is therefore
Answer (B) is correct. Optimizing or problem solving beneficial.
entails addressing the source of conflict and finding
alternative strategies that benefit all parties. It
[271] Source: CIA 0596 II-24 [274] Source: CIA 1196 II-23

Answer (A) is correct. Conflict arises when one party Answer (A) is incorrect because referring the matter
perceives that another party has negatively affected, to the auditee's superior will serve only to alienate the
or will negatively affect, its interests. Hence, auditee.
management actions conflict with the educational
interests of the staff. Answer (B) is incorrect because waiting until late in
the day is unlikely to work. A tired individual is less
Answer (B) is incorrect because objective criteria are likely to listen and cooperate.
identified for a performance evaluation.
Answer (C) is incorrect because the auditee has
Answer (C) is incorrect because the division of staff decided that (s)he is right and is not likely to be
into separate groups does not represent conflict. receptive to a logical argument.

Answer (D) is incorrect because pre-assigning Answer (D) is correct. The most effective way for an
standard audit programs does not represent conflict. auditor to gain consensus is to find a point on which
the auditor and auditee agree. This point may be an
opening wedge. The auditor should also practice
[272] Source: CIA 0596 II-25 genuine listening and understanding and emphasize
persuasion, not threats.
Answer (A) is incorrect because all of the answers
are examples of the traditional view of motivation that
the new director needs to overcome. [275] Source: Publisher

Answer (B) is incorrect because all of the answers Answer (A) is incorrect because the board would be
are examples of the traditional view of motivation that consulted initially only if the immediate superior is the
the new director needs to overcome. chief executive officer and that person is involved in
the ethical conflict.
Answer (C) is incorrect because all of the answers
are examples of the traditional view of motivation that Answer (B) is correct. The Standards of Ethical
the new director needs to overcome. Conduct for Practitioners of Management Accounting
and Financial Management state that the financial
Answer (D) is correct. One of the traditional theories manager/management accountant should first discuss
emphasizes an autocratic view of management: an ethical problem with his/her immediate superior. If
employees do not like to work and must be coerced the superior is involved, the problem should be taken
and controlled; employees are principally motivated initially to the next higher managerial level.
by economic concerns. Hence, pre-set audit
programs and specific rules for attendance and Answer (C) is incorrect because unless "legally
assignment completion reflect the emphasis on close prescribed, communication of such problems to
control. Standardizing compensation is another authorities or individuals not employed or engaged by
aspect of the traditional view. the organization is not considered appropriate."

Answer (D) is incorrect because resignation is a last


[273] Source: CIA 0596 II-26 resort.

Answer (A) is correct. Communication, structure,


and personal variables are the broad categories of [276] Source: Publisher
conditions that may result in conflict. Structural
conditions include the size of the work group, Answer (A) is incorrect because each standard is
specialization of tasks, the clarity of lines of authority, violated by a financial manager/management
leadership practices, compensation schemes, and the accountant who fails to act upon discovering unethical
interdependence of groups. Thus, communication conduct.
within the department is not a structural issue.
Answer (B) is incorrect because each standard is
Answer (B) is incorrect because conflict of the violated by a financial manager/management
accountant who fails to act upon discovering unethical
structural variety tends to be greatest when group conduct.
members are younger and turnover is high. Thus,
tenure in the job and conflict are inversely related. Answer (C) is incorrect because each standard is
violated by a financial manager/management
Answer (C) is incorrect because reward systems are accountant who fails to act upon discovering unethical
an element of structure. They may stimulate conflict if conduct.
individuals believe they are being unfairly
compensated. Answer (D) is correct. A financial
manager/management accountant displays his/her
Answer (D) is incorrect because size of the group competence and objectivity and maintains integrity by
and degree of specialization are two elements of taking the appropriate action within the organization
structure in an organization. to resolve an ethical problem. Failure to act would
condone wrongful acts, breach the duty to convey
unfavorable as well as favorable information,
undermine the organization's legitimate aims, discredit standard of confidentiality. Thus, the financial
the profession, and violate the duty of objectivity manager/management accountant should "refrain from
owed to users of the subordinate's work product. disclosing confidential information acquired in the
course of his/her work except when authorized,
unless legally obligated to do so."
[277] Source: Publisher
Answer (C) is incorrect because the financial
Answer (A) is incorrect because this course of action manager/management accountant should "inform
would be appropriate only for the chief executive subordinates as appropriate regarding the
officer or for his/her immediate subordinate when the confidentiality of information acquired in the course of
CEO is involved in the conflict. their work and monitor their activities to assure the
maintenance of that confidentiality."
Answer (B) is incorrect because the proper action
would be to present the matter to the next higher Answer (D) is incorrect because the financial
managerial level. manager/management accountant is required to
"refrain from using or appearing to use confidential
Answer (C) is incorrect because such action is information acquired in the course of his/her work for
inappropriate unless legally prescribed. unethical or illegal advantage either personally or
through third parties."
Answer (D) is correct. In these circumstances, the
problem should be discussed with the immediate
superior unless (s)he is involved. In that case, initial [280] Source: Publisher
presentation should be to the next higher managerial
level. If the problem is not satisfactorily resolved after Answer (A) is correct. One of the responsibilities of
initial presentation, the question should be submitted the financial manager/management accountant under
to the next higher level. the integrity standard is to "recognize and

communicate professional limitations or other


[278] Source: Publisher constraints that would preclude responsible judgment
or successful performance of an activity."
Answer (A) is incorrect because, in this situation, the
chief executive officer is the next higher managerial Answer (B) is incorrect because the objectivity
level. standard requires the financial manager/management
accountant to "disclose fully all relevant information
Answer (B) is incorrect because the immediate that could reasonably be expected to influence an
superior has promised or taken action toward intended user's understanding of the reports,
satisfactory resolution. comments, and recommendations presented."

Answer (C) is incorrect because the immediate Answer (C) is incorrect because the confidentiality
superior has promised or taken action toward standard requires the financial manager/management
satisfactory resolution. accountant to "refrain from disclosing confidential
information acquired in the course of his/her work
Answer (D) is correct. According to the IMA Code except when authorized, unless legally obligated to do
of Ethics, the financial manager/management so."
accountant should "discuss such problems with the
immediate superior except when it appears that the Answer (D) is incorrect because the integrity
superior is involved, in which case the problem standard requires the financial manager/ management
should be presented initially to the next higher accountant to "refuse any gift, favor, or hospitality
managerial level. If satisfactory resolution cannot be that would influence or would appear to influence
achieved when the problem is initially presented, his/her actions."
submit the issues to the next higher managerial level.

If the immediate superior is the chief executive officer,


or equivalent, the acceptable reviewing authority may
be a group such as the audit committee, executive
committee, board of directors, board of trustees, or
owners."

[279] Source: Publisher

Answer (A) is incorrect because the IMA Code of


Ethics states that "except where legally prescribed,
communication of such [ethical conflict] problems to
authorities or individuals not employed or engaged by
the organization is not considered appropriate."

Answer (B) is correct. According to the IMA Code


of Ethics, financial managers/management
accountants are responsible for observing the

You might also like

pFad - Phonifier reborn

Pfad - The Proxy pFad of © 2024 Garber Painting. All rights reserved.

Note: This service is not intended for secure transactions such as banking, social media, email, or purchasing. Use at your own risk. We assume no liability whatsoever for broken pages.


Alternative Proxies:

Alternative Proxy

pFad Proxy

pFad v3 Proxy

pFad v4 Proxy